Download as pdf or txt
Download as pdf or txt
You are on page 1of 203

fiziks

Institute for NET/JRF, GATE, IIT‐JAM, M.Sc. Entrance, JEST, TIFR and GRE in Physics

MATHEMATICAL PHYSICS SOLUTIONS


GATE-2010
Q1. Consider an anti-symmetric tensor Pij with indices i and j running from 1 to 5. The

number of independent components of the tensor is


(a) 3 (b) 10 (c) 9 (d) 6
Ans: (b)
Solution: The number of independent components of the tensor

= N  N   1 25  5  10
1 2
 N  5 
2 2
e z sin  z 
Q2. The value of the integral  dz , where the contour C is the unit circle: z  2  1 ,
C
z 2

is
(a) 2 i (b) 4 i (c)  i (d) 0
Ans: (d)
Solution:  z  2  1  1  z  3 i.e. the pole z  0 does not lie inside the contour.

e z sin z
 C z 2 dz  2 i  0  0 .
2 3 0
 
Q3. The eigenvalues of the matrix  3 2 0  are
0 1 
 0

(a) 5, 2, -2 (b) -5, -1, -1 (c) 5, 1, -1 (d) -5, 1, 1


Ans: (c)
Solution: The characteristic equation of the matrix A , A  I  0

2 3 0
 A  I  2  0  1     2     9   0    1, 2    3
2
3 0
 
0 0 1 

   5,1,  1

0 for x  3,
Q4. If f  x    then the Laplace transform of f(x) is
x  3 for x  3

(a) s 2 e3s (b) s 2 e3s (c) s 2 (d) s 2 e 3s

H.No. 40-D, Ground Floor, Jia Sarai, Near IIT, Hauz Khas, New Delhi-110016
Phone: 011-26865455/+91-9871145498
Website: www.physicsbyfiziks.com | Email: fiziks.physics@gmail.com
1
fiziks
Institute for NET/JRF, GATE, IIT‐JAM, M.Sc. Entrance, JEST, TIFR and GRE in Physics

Ans: (d)
 3  
Solution: L f  x    e  sx
f  x  dx   e s x
f  x  dx   e  sx
f  x  dx    x  3 e  sx dx
0 0 3 3

   
e  sx  e sx  1  e sx 
L  f  x    x  3
1  sx
  1   dx  0   e dx     s 2 e3 s
s 3 3  s  s3 s  s 3

d2y
Q5. The solution of the differential equation for y t  :  y  2 cosh(t ) , subject to the
dt 2
dy
initial conditions y 0   0 and  0 , is
dt t  0

cosh t   t sinh t  (b)  sinh t   t cosh t 


1
(a)
2
(c) t cosh t  (d) t sinh t 
Ans: (d)
Solution: For C.F D 2  1y  0  m  1  C.F .  C1e t  C 2 e t

 e t  e t 
P.I .  2
1
2 cosh t   2 2
1
  2
1
et  2
1
 
t t
e t  e t   e t    
D 1 D 1  2  D 1 D 1 2 2

t t t t
 y  C1e t  C 2 e t  e  e
2 2
As, y  0   0  C1  C2  0............ 1

dy t 1 t 1
 C1e t  C 2 e t  e t  e t  e t  e t
dt 2 2 2 2
dy 1 1
Also,  0  C1  C2  0   0   0  C1  C2  0...........  2 
dt t 0 2 2
From equation (1) and (2),
C1  0, C2  0 .

t t
Thus y  et  e  t  y  t sinh t
2 2

H.No. 40-D, Ground Floor, Jia Sarai, Near IIT, Hauz Khas, New Delhi-110016
Phone: 011-26865455/+91-9871145498
Website: www.physicsbyfiziks.com | Email: fiziks.physics@gmail.com
2
fiziks
Institute for NET/JRF, GATE, IIT‐JAM, M.Sc. Entrance, JEST, TIFR and GRE in Physics

GATE-2011
Q6. Two matrices A and B are said to be similar if B = P-1AP for some invertible matrix P.
Which of the following statements is NOT TRUE?
(a) Det A = Det B (b) Trace of A = Trace of B
(c) A and B have the same eigenvectors (d) A and B have the same eigenvalues
Ans: (c)
Solution: If A and B be square matrices of the same type and if P be invertible matrix, then
matrices A and B = P-1AP have the same characteristic roots.
Then, B  I  P 1 AP  P 1IP  P 1  A  I P where I is identity matrix.

B  I  P 1  A  I P  P 1 A  I P  A  I P 1 P  A  I PP 1  A  I

Thus, the matrices A and B (= P-1AP) have the same characteristic equation and hence
same characteristic roots or eigen values. Since, the sum of the eigen values of a matrix
and product of eigen values of a matrix is equal to the determinant of matrix, hence third
alternative is incorrect.

Q7. If a force F is derivable from a potential function V(r), where r is the distance from the
origin of the coordinate system, it follows that

(a)   F  0 (b)   F  0 (c)  V  0 (d)  2 V  0


Ans: (a)
 
Solution: Since, F is derivative of potential V(r) and F  V r 

 
   F    V  0 .
Q8. A 33 matrix has elements such that its trace is 11 and its determinant is 36. The
eigenvalues of the matrix are all known to be positive integers. The largest eigenvalues of
the matrix is
(a) 18 (b) 12 (c) 9 (d) 6
Ans: (d)
Solution: We know that for any matrix
1. The product of eigenvalues is equals to the determinant of that matrix.
2. 1   2  3  .......  Trace of matrix
1   2  3  11 and 12 3  36 . Hence, the largest eigen value of the matrix is 6.

H.No. 40-D, Ground Floor, Jia Sarai, Near IIT, Hauz Khas, New Delhi-110016
Phone: 011-26865455/+91-9871145498
Website: www.physicsbyfiziks.com | Email: fiziks.physics@gmail.com
3
fiziks
Institute for NET/JRF, GATE, IIT‐JAM, M.Sc. Entrance, JEST, TIFR and GRE in Physics

Q9. The unit vector normal to the surface x2 + y2 – z = 1 at the point P(1, 1, 1) is
iˆ  ˆj  kˆ 2iˆ  ˆj  kˆ iˆ  2 ˆj  kˆ 2iˆ  2 ˆj  kˆ
(a) (b) (c) (d)
3 6 6 3

Ans: (d)
Solution: The equation of the system is f x, y, z   x 2  y 2  z  1  0
   ˆ  ˆ 2
f   iˆ  j  k x  y 2  z  1  2 xiˆ  2 yjˆ  kˆ
 x y z 

f 2iˆ  2 ˆj  kˆ
Hence, unit normal vector at (1, 1, 1)    .
f 3

Q10. Consider a cylinder of height h and radius a, closed at both ends, centered at the origin.

Let r  iˆx  ˆjy  kˆz be the position vector and n̂ be a unit vector normal to the surface.

The surface integral  r  nˆ ds over the closed surface of the cylinder is


S
z

O y

x
2 2
(a) 2πa (a + h) (b) 3πa h (c) 2 πa2h (d) zero
Ans: (b)

 
Solution:  r.nˆ ds   .r d  3 d  3a 2 h
S V V

dy x
Q11. The solutions to the differential equation  are a family of
dx y 1

(a) circles with different radii


(b) circles with different centres
(c) straight lines with different slopes
(d) straight lines with different intercepts on the y-axis
Ans: (a)
dy x x2 y2
Solution:   xdx  ydy  dy  0    y  C1  x 2  y 2  2 y  2C1
dx y 1 2 2

H.No. 40-D, Ground Floor, Jia Sarai, Near IIT, Hauz Khas, New Delhi-110016
Phone: 011-26865455/+91-9871145498
Website: www.physicsbyfiziks.com | Email: fiziks.physics@gmail.com
4
fiziks
Institute for NET/JRF, GATE, IIT‐JAM, M.Sc. Entrance, JEST, TIFR and GRE in Physics

  x  0    y  1  2C1  1  C
2 2

which is a family of circles with different radii.


z sin z
Q12. Which of the following statements is TRUE for the function f  z   ?
 z   2
(a) f  z  is analytic everywhere in the complex plane

(b) f  z  has a zero at z  

(c) f  z  has a pole of order 2 at z  

(d) f  z  has a simple pole at z  

Ans: (c)
z sin z
Solution: f  z   has a pole of order 2 at z  
z  
2

z sin z
Q13. Consider a counterclockwise circular contour z  1 about the origin. Let f  z   ,
 z   2
then the integral  f z dz over this contour is
(a) –iπ (b) zero (c) iπ (d) 2iπ
Ans: (b)
Solution: Since, pole z   does not lie inside the contour, hence

 f  z  dz  0
GATE-2012
 
Q14. Identify the correct statement for the following vectors a  3iˆ  2 ˆj and b  iˆ  2 ˆj
 
(a) The vectors a and b are linearly independent
 
(b) The vectors a and b are linearly dependent
 
(c) The vectors a and b are orthogonal
 
(d) The vectors a and b are normalized
Ans: (a)
 
Solution: If a  3iˆ  2 ˆj, b  iˆ  2 ˆj are linearly dependent, then

H.No. 40-D, Ground Floor, Jia Sarai, Near IIT, Hauz Khas, New Delhi-110016
Phone: 011-26865455/+91-9871145498
Website: www.physicsbyfiziks.com | Email: fiziks.physics@gmail.com
5
fiziks
Institute for NET/JRF, GATE, IIT‐JAM, M.Sc. Entrance, JEST, TIFR and GRE in Physics

 
a  mb  0, for some values of m but here,
3 + m = 0 and 2 + 2m = 0 , do not have any solution. So, they are linearly independent.
   
a  b  0 (Not orthogonal); a  b  0 (Not normalized)
Q15. The number of independent components of the symmetric tensor Aij with indices
i, j  1, 2,3 is
(a) 1 (b) 3 (c) 6 (d) 9
Ans: (c)
 A11 A12 A13 
Solution: For symmetric tensor, Aij   A21 A22 A23 

 A31 A32 A33 

 A12  A21 , A23  A32 , A13  A31 , hence there are six independent components.

0 1 0
Q16. The eigenvalues of the matrix 1 0 1  are
0 0 
 1

(a) 0, 1, 1 (b) 0, 2 , 2


1 1
(c) , ,0 (d) 2 , 2 ,0
2 2
Ans: (b)
 1 0
Solution: A  I  0  1   
1  0   2  1    0    0,  2 ,  2
0 1 

GATE-2013
    
Q17. If A and B are constant vectors, then  A  B  r is  
    
(a) A  B (b) A  B (c) r (d) zero
Ans: (d)

Solution: Let A  A0  xˆ  yˆ  zˆ  , B  B0  xˆ  yˆ  zˆ  and r  xxˆ  yyˆ  zzˆ .
  
 
 
B  r  xˆ  z  y B0  yˆ  z  x B0  zˆ  y  x B0   A  B  r  0 . 

H.No. 40-D, Ground Floor, Jia Sarai, Near IIT, Hauz Khas, New Delhi-110016
Phone: 011-26865455/+91-9871145498
Website: www.physicsbyfiziks.com | Email: fiziks.physics@gmail.com
6
fiziks
Institute for NET/JRF, GATE, IIT‐JAM, M.Sc. Entrance, JEST, TIFR and GRE in Physics

16 z
Q18. For the function f  z   , the residue at the pole z  1 is (your answer
z  3z  12
should be an integer) ____________.
Ans: 3

1 d 21   z  1 16 z 
2

Solution: At z  1 , pole is of order 2. So, residue is   =3.


2  1 dz 21   z  3 z  12 
  z 1

 4  1  1
Q19. The degenerate eigenvalue of the matrix  1 4  1 is (your answer should be an
 1  1 4 

integer) ____________
Ans: 2,5,5

4   1 1   1 1 1 
 1 4  
1   0  (2   )  0 5   0  = (2   )(5   )2  0    2,5,5 .

 1 1 4     0 0 5   

Q20. The number of distinct ways of placing four indistinguishable balls into five
distinguishable boxes is ___________.
Ans: 120
Solution: 4  C 45 =120 ways

GATE-2014
Q21. The unit vector perpendicular to the surface x 2  y 2  z 2  3 at the point (1, 1, 1) is
xˆ  yˆ  zˆ xˆ  yˆ  zˆ xˆ  yˆ  zˆ xˆ  yˆ  zˆ
(a) (b) (c) (d)
3 3 3 3
Ans: (d)

Solution: Let, f  x 2  y 2  z 2  3  0   f  2 xxˆ  2 yyˆ  2 zzˆ

f 2 xˆ  2 yˆ  2 zˆ xˆ  yˆ  zˆ
 nˆ   at 1,1,1  
f 12 3

H.No. 40-D, Ground Floor, Jia Sarai, Near IIT, Hauz Khas, New Delhi-110016
Phone: 011-26865455/+91-9871145498
Website: www.physicsbyfiziks.com | Email: fiziks.physics@gmail.com
7
fiziks
Institute for NET/JRF, GATE, IIT‐JAM, M.Sc. Entrance, JEST, TIFR and GRE in Physics

Q22. The matrix


1  1 1  i
A   is
3 1  i  1 
(a) orthogonal (b) symmetric (c) anti-symmetric (d) Unitary
Ans. : (d)
Solution: Unitary A† A  I
Q23. The value of the integral
z2
 z dz
C e 1

where C is the circle z  4 , is

(a) 2 i (b) 2 2 i (c) 4 3i (d) 4 2 i


Ans. : (c)
Solution: Pole e z  1  e z  ei 2 m 1 where m  0,1, 2,3.....

  z 2
For z  i , Res  lim   i   2
z i    z  e

Similarly, for z  i , Res   2

I  2 i  2   2   4 3i

d2y
Q24. The solution of the differential equation  y  0 , subject to the boundary conditions
d t2

y 0   1 and y    0 is
(a) cos t  sin t (b) cosh t  sinh t
(c) cos t  sin t (d) cosh t  sinh t
Ans: (d)
Soluiton:
D 2  1  0  D  1  y  t   c1et  c2 e  t

Applying boundary condition,


y 0   1  1  c1  c2 and y     0  0  c1e  c2 e   c1  0, c2  1

 y  t   e t  y  t   cosh t  sinh t

H.No. 40-D, Ground Floor, Jia Sarai, Near IIT, Hauz Khas, New Delhi-110016
Phone: 011-26865455/+91-9871145498
Website: www.physicsbyfiziks.com | Email: fiziks.physics@gmail.com
8
fiziks
Institute for NET/JRF, GATE, IIT‐JAM, M.Sc. Entrance, JEST, TIFR and GRE in Physics

GATE-2015
1
Q25. Consider a complex function f  z   . Which one of the following
 1
z  z   cos  z 
 2
statements is correct?
(a) f  z  has simple poles at z  0 and z  
1
2
(b) f  z  has second order pole at z  
1
2
(c) f  z  has infinite number of second order poles
(d) f  z  has all simple poles
Ans.: (a)
1
Solution: f  z  
 1
z  z   cos  z 
 2

For nth order pole, Res.  lim  z  a  f  z   finite


n

z a

At z  0 , lim zf  z   finite  z  0 is a simple pole.


z 0
2
 1  1
z  z 
1
At z   , lim  2
 lim 
2
 lim
1
2 z  1  1 z  z cos z z  1.cos z  z.   sin z 
1 1
2 z z   cos z 2 2
 2
1 1 2
 lim1     finite
z  cos z  z sin z  
2 
2
1
 f  z  has second order pole at z  
2
3
Q26. The value of  t 2 3t  6 dt is_______________ (upto one decimal place)
0

Ans.: 1.33
3 3 3
1 4
Solution:  t   3t  6  dt   t  3  t  2   dt   t 2  t  2  dt 
2 2

0 0
30 3

H.No. 40-D, Ground Floor, Jia Sarai, Near IIT, Hauz Khas, New Delhi-110016
Phone: 011-26865455/+91-9871145498
Website: www.physicsbyfiziks.com | Email: fiziks.physics@gmail.com
9
fiziks
Institute for NET/JRF, GATE, IIT‐JAM, M.Sc. Entrance, JEST, TIFR and GRE in Physics

If f  x   e  x and g  x   x e  x , then
2 2
Q27.

(a) f and g are differentiable everywhere


(b) f is differentiable everywhere but g is not
(c) g is differentiable everywhere but f is not
(d) g is discontinuous at x  0
Ans. (b)

Solution: f ( x)  e  x is differentiable but g ( x)  x e  x is not differentiable.


2 2

 xe  x ; x  0
2

g ( x)  
 x2
 xe ; x  0
2
 x h
Left hand Limit lim g  x  h     x  h  e
h 0

2
 xh
Right hand Limit lim g  x  h    x  h  e
h 0

 lim g  x  h   lim g  x  h 
h 0 h 0

Q28. Consider w  f  z   u  x, y   iv x, y  to be an analytic function in a domain D . Which


one of the following options is NOT correct?
(a) u  x, y  satisfies Laplace equation in D
(b) v x, y  satisfies Laplace equation in D
z2

(c)  f z dz is dependent on the choice of the contour between z and z


z1
1 2 in D

(d) f  z  can be Taylor expended in D


Ans.: (c)
z2

Solution: w  f ( z )  u  x, y   iv  x, y  to be an analytic function in a domain D ,  f  z  dz


z1
is

independent of the choice of the contour between z1 and z2 in D .

H.No. 40-D, Ground Floor, Jia Sarai, Near IIT, Hauz Khas, New Delhi-110016
Phone: 011-26865455/+91-9871145498
Website: www.physicsbyfiziks.com | Email: fiziks.physics@gmail.com
10
fiziks
Institute for NET/JRF, GATE, IIT‐JAM, M.Sc. Entrance, JEST, TIFR and GRE in Physics

1, for t  0
Q29. The Heaviside function is defined as H  t    and its Fourier transform
1, for t  0

is given by  2i /  . The Fourier transform of


1
H t  1 / 2  H t  1 / 2 is
2
   
sin   cos 
2 2  
(a) (b) (c) sin   (d) 0
  2
2 2
Ans.: (a)

2i
Solution: H  f    H t  e
 i 2 ft
dt , for a function H  t  , H  f   


For H  t  t0  , Fourier Transform is e  i 2 ft0
Hf

Shifting Theorem
 
1  1  1  1  i 2  i  2i 1  i 2 
 i  2i
For  H  t    H  t     e  e  2
 e  e 2  i
2   2  2  2    2i   
 
sin  
1
The Fourier transform of  H  t  1/ 2   H  t  1/ 2    2 .
2 
2
m2
A function y  z  satisfies the ordinary differential equation y 
1
Q30. y  2 y  0, where
z z
m  0, 1, 2, 3, ..... Consider the four statements P, Q, R, S as given below.

P: z m and z  m are linearly independent solutions for all values of m


Q: z m and z  m are linearly independent solutions for all values of m  0
R: ln z and 1 are linearly independent solutions for m  0
S: z m and ln z are linearly independent solutions for all values of m
The correct option for the combination of valid statements is
(a) P, R and S only (b) P and R only (c) Q and R only (d) R and S only
Ans.: (c)
1 m2 d
Solution: y   y   2 y  0  z 2 y  zy  m 2 y  0 , m  0,1, 2,3,..., z  ex , D 
z z dx

H.No. 40-D, Ground Floor, Jia Sarai, Near IIT, Hauz Khas, New Delhi-110016
Phone: 011-26865455/+91-9871145498
Website: www.physicsbyfiziks.com | Email: fiziks.physics@gmail.com
11
fiziks
Institute for NET/JRF, GATE, IIT‐JAM, M.Sc. Entrance, JEST, TIFR and GRE in Physics

If m  0 ; z 2 y  zy  0 ,  D  D  1  D  y  0   D 2  D  D  y  0

D 2 y  0  y  c1  c2 x  y  c1  c2 ln z ( R is correct)

And if m  0, m  0 , then m  0 , then D 2  m 2 y  0  D   m 


y  c1e mx  c2 e  mx  c1e m log z  c2 e  m log z  c1 z m  c2 z  m

or if m  0, m  0 , then

y  c1 cosh  m log  z    ic2 sinh  m log  x   , m  0

GATE-2016
dy
Q31. Consider the linear differential equation  xy . If y  2 at x  0 , then the value of y at
dx
x  2 is given by
(a) e 2 (b) 2e 2 (c) e 2 (d) 2e 2
Ans.: (d)
dy 1 x2
 xy  dy  xdx  ln y   ln c  y  ce x / 2
2
Solution:
dx y 2

If y  2 at x  0  c  2  y  2e x
2
/2
.

The value of y at x  2 is given by y  2e 2


Q32. Which of the following is an analytic function of z everywhere in the complex plane?

(a) z 2  
(b) z *
2
(c) z
2
(d) z

Ans.: (a)

Solution: z 2   x  iy   x 2  y 2  i  2 xy   u  x 2  y 2 and v  2 xy
2

u v v u
Cauchy Riemann equations   2 x,   2 y satisfies.
x y x y

The direction of f for a scalar field f  x, y, z   x 2  xy  z 2 at the point P1,1,2  is
1 1
Q33.
2 2

(a)
 ˆj  2kˆ (b)
 ˆj  2kˆ (c)
 ˆj  2kˆ (d)
 ˆj  2kˆ
5 5 5 5
Ans.: (b)

H.No. 40-D, Ground Floor, Jia Sarai, Near IIT, Hauz Khas, New Delhi-110016
Phone: 011-26865455/+91-9871145498
Website: www.physicsbyfiziks.com | Email: fiziks.physics@gmail.com
12
fiziks
Institute for NET/JRF, GATE, IIT‐JAM, M.Sc. Entrance, JEST, TIFR and GRE in Physics

  
 f  ˆj  2kˆ
Solution: f   x  y  iˆ  xjˆ  zkˆ  nˆ     
 f  5
 1,1,2

Q34. A periodic function f  x  of period 2 is defined in the interval    x   

 1,    x  0
f x   
 1, 0 x 

The appropriate Fourier series expansion for f  x  is

4
(a) f  x    sin x  sin 3x  / 3  sin 5 x  / 5  ...
 
4
(b) f  x    sin x  sin 3x  / 3  sin 5 x  / 5  ..
 
4
(c) f  x    cos x  cos 3x  / 3  cos 5 x  / 5  ...
 
4
(d) f  x    cos x  cos 3x  / 3  cos 5 x  / 5  ...
 
Ans.: (a)
 1,    x  0
Solution: f  x   
 1, 0 x 

Let f  x   a0    an cos nx  bn sin nx 
n 1

1 
 a0 
2   f  x dx

 1  0 
1 dx  
1 1 
f  x dx       x    x 0   0

 a0      
0
1 dx
2  
2    0 
 2  

This can also be seen without integration, since the area under the curve of f  x  between
 to  is zero.
1 
f  x  cos nxdx
 
 an 


1   sin nx   sin nx  
0
1 0 
  
 an 
  
1 cos nxdx  0  
1 cos nxdx  
    
    n   n 0 
 0

H.No. 40-D, Ground Floor, Jia Sarai, Near IIT, Hauz Khas, New Delhi-110016
Phone: 011-26865455/+91-9871145498
Website: www.physicsbyfiziks.com | Email: fiziks.physics@gmail.com
13
fiziks
Institute for NET/JRF, GATE, IIT‐JAM, M.Sc. Entrance, JEST, TIFR and GRE in Physics

1 
 bn 
   f  x  sin nxdx

1 0 
 bn    1 sin nxdx   1 sin nxdx 
   0 

 cos nx   1  1  1  1 1  1  2 2  1n 



1   cos nx 
0 n n

 bn               
   n   n 0    n n n n    n n 

0; n  even

 bn   4
 n ; n  odd

4 1 1 
Thus, Fourier series is f  x    sin x  sin 3 x  sin 5 x  ...
 3 5 

GATE-2017
dz
Q35. The contour integral  1  z 2
evaluated along a contour going from  to  along the

real axis and closed in the lower half-plane circle is equal to………….. (up to two
decimal places).
Ans. : 

1 1 1
Solution: C 1  z 2 dz   1  x 2 dx  C 1  z 2 dz
Poles, 1  z 2  0  z  i , z  i is inside C
1 1 1
 Res  z  i   lim  z  i   
z  i  z  i  z  i  i  i 2i

1 1
 dx    2 i  

1 x 2
2i
(Since, here we use lower half plane i.e., we traversed in clockwise direction, hence we
have to take 2 i )
Q36. The coefficient of eikx in the Fourier expansion of u  x   A sin 2  x  for k  2 is

A A A A
(a) (b) (c) (d)
4 4 2 2
Ans.: (b)

H.No. 40-D, Ground Floor, Jia Sarai, Near IIT, Hauz Khas, New Delhi-110016
Phone: 011-26865455/+91-9871145498
Website: www.physicsbyfiziks.com | Email: fiziks.physics@gmail.com
14
fiziks
Institute for NET/JRF, GATE, IIT‐JAM, M.Sc. Entrance, JEST, TIFR and GRE in Physics

ei x  e  i x ei 2 x  2  e 2i x
Solution: Since, sin  x    sin 2  x  
2i  4 
e  ikx  2  eikx
Since, 2   k , hence sin 2  x  
 4 
   
A A   ikx ikx 
 sin  x  dx     
 ikx  ikx ikx
Hence, ck  2
e e dx  2 e dx  e e dx 
2 
8     
  
A  2ikx 
  e dx  2  e dx   dx 
 ikx

8     
The first two integrals are zero and the third integral has the value 2 .
Thus,
A A
ck    2   
8 4
Q37. The imaginary part of an analytic complex function is v  x, y   2 xy  3 y . The real part of

the function is zero at the origin. The value of the real part of the function at 1  i
is ……………... (up to two decimal places)
Ans. : 3
Solution: The imaginary part of the given analytic function is v  x , y   2 xy  3 y . From the

Cauchy – Riemann condition


v u
  2x  3
y x
Integrating partially gives
u  x , y   x 2  3x  g  y 

From the second Cauchy – Riemann condition


u v u
 , we obtain  2 y,   x, y    y 2  g  x 
y x y
dg  y 
 2 y  g  y    y 2  c
dy

Hence, u  x , y   x 2  3 x  y 2  c

Since, the real part of the analytic function is zero at the origin.

H.No. 40-D, Ground Floor, Jia Sarai, Near IIT, Hauz Khas, New Delhi-110016
Phone: 011-26865455/+91-9871145498
Website: www.physicsbyfiziks.com | Email: fiziks.physics@gmail.com
15
fiziks
Institute for NET/JRF, GATE, IIT‐JAM, M.Sc. Entrance, JEST, TIFR and GRE in Physics

Hence, 0  0  0  0  c  c  0

Thus, u  x , y   x 2  3 x  y 2

 f  z    x 2  3 x  y 2   i  2 xy  3 y 

Thus, the value of real part when

z  1  i ,i.e. x  1 and y  1 is u  x, y   1  3 1  1  3 .


2

Q38. Let X be a column vector of dimension n  1 with at least one non-zero entry. The
number of non-zero eigenvalues of the matrix M  XX T is
(a) 0 (b) n (c) 1 (d) n  1
Ans. : (c)
0
0
 
a 
Solution: Let X    , then X T   0 0 a... 0
0
0
 
 0 

Here, X is an n  1 column vector with the entry in the i th row equal to a. X T is a row

vector having entry in the i th column equal to a. Then, XX T is an n  1 matrix having

the entry in the i th row and i th column equal to a 2 .


Hence,
0 0 0...0...0 0 
0 0 0...0...0 0 
 ith row
XX  0 0 0...0...0 0 
T
.....................
..................... 
0 0 0...0...0 0 

ith row
Since this matrix is diagonal, its eigenvalues are a 2 , 0, 0.....0 . Hence, the number of non

zero eigenvalues of the matrix XX T is 1 .

dy  
Q39. Consider the differential equation  y tan  x   cos  x  . If y  0   0, y  
dx 3
is …………... (up to two decimal places)

H.No. 40-D, Ground Floor, Jia Sarai, Near IIT, Hauz Khas, New Delhi-110016
Phone: 011-26865455/+91-9871145498
Website: www.physicsbyfiziks.com | Email: fiziks.physics@gmail.com
16
fiziks
Institute for NET/JRF, GATE, IIT‐JAM, M.Sc. Entrance, JEST, TIFR and GRE in Physics

Ans.: 0.52
Solution: The given differential equation is a linear differential equation of the form
dy
 p  x  y  cos x
dx

Integrating factor  e 
p  x  dx

Thus integrating factor  e 


tan x dx

 I  F  eln sec x  sec x


Thus the general solution of the given differential equation is
y  sec x   sec x  cos xdx  c
 y sec x  x  c -(i)

It is given that y  0   0  0  sec 0  0  c  c  0

Thus the solution satisfying the given condition is


x
y sec x  x  y 
sec x
 
Thus the value of y   is
3
 /3  /3 
y    0  52
sec  / 3 2 6

H.No. 40-D, Ground Floor, Jia Sarai, Near IIT, Hauz Khas, New Delhi-110016
Phone: 011-26865455/+91-9871145498
Website: www.physicsbyfiziks.com | Email: fiziks.physics@gmail.com
17
fiziks
Institute for NET/JRF, GATE, IIT‐JAM, M.Sc. Entrance, JEST, TIFR and GRE in Physics

GATE-2018
Q40. The eigenvalues of a Hermitian matrix are all
(a) real (b) imaginary (c) of modulus one (d) real and positive
Ans. : (a)
Solution: Eigenvalue of Hermitian matrix must be real.
Q41. In spherical polar coordinates  r ,  ,   , the unit vector ˆ at 10,  / 4,  / 2  is

(a) k̂ (b)
2

1 ˆ ˆ
jk  (c)
1
2

 ˆj  kˆ  (d)
2

1 ˆ ˆ
jk 
Ans. : (d)
z
Solution: ˆ  cos 45 ˆj  sin 45 kˆ
0 0

 /4
 ˆ 
1 ˆ ˆ
2
jk   ˆ
y
Q42. The scale factors corresponding to the covariant metric tensor gi j in spherical polar

coordinates are
(a) 1, r 2 , r 2 sin 2  (b) 1, r 2 ,sin 2  (c) 1,1,1 (d) 1, r , r sin 
Ans. : (d)
     
Q43. Given V1  iˆ  ˆj and V2  2iˆ  3 ˆj  2kˆ , which one of the following V3 makes V1 ,V2 , V3  
a complete set for a three dimensional real linear vector space?
 
(a) V3  iˆ  ˆj  4kˆ (b) V3  2iˆ  ˆj  2kˆ
 
(c) V3  iˆ  2 ˆj  6kˆ (d) V3  2iˆ  ˆj  4kˆ
Ans. : (d)
  
Solution: Let A be the matrix formed by taking V1 , V2 and V3 as column matrix i.e.,

 1 2 2 
A  V1 V2 V3    1 3 1   A  2 . Here V3  2iˆ  ˆj  4kˆ  
 0 2 4 
  
Since, A  0 , hence, V1 , V2 and V3 form a three dimensional real vector space.

Hence, option (d) is correct.

H.No. 40-D, Ground Floor, Jia Sarai, Near IIT, Hauz Khas, New Delhi-110016
Phone: 011-26865455/+91-9871145498
Website: www.physicsbyfiziks.com | Email: fiziks.physics@gmail.com
18
fiziks
Institute for NET/JRF, GATE, IIT‐JAM, M.Sc. Entrance, JEST, TIFR and GRE in Physics

Q44. Given
d 2 f  x df  x 
2  f  x  0 ,
dx 2 dx
and boundary conditions f  0   1 and f 1  0 , the value of f  0.5  is __________ (up

to two decimal places) .


Ans. : 0.81
d 2 f  x df  x 
Solution: 2  f  x  0
dx 2
dx
Auxiliary equation is,

m 2

 2m  1  0   m  1  0  m  1,1
2

Hence, the solution is


f  x    c1  c2 x  e x

using boundary condition,


f  0   c1e0  c1  1 (i)

f 1   c1  c2  e  0 (ii)

From (i) and (ii), c2  1

Hence, f  x   1  x  e x  f  0.5   1  0.5  e0.5  0.81

Q45. The absolute value of the integral


5 z 3  3z 2
 z 2  4 dz ,
over the circle z  1.5  1 in complex plane, is __________ (up to two decimal places).

Ans. : 81.64
5 z 3  3z 2
Solution: f  z  
 z  2  z  2 
Pole, z  2, 2
z  2 is outside the center
2  1.5  1 So, will not be considered

H.No. 40-D, Ground Floor, Jia Sarai, Near IIT, Hauz Khas, New Delhi-110016
Phone: 011-26865455/+91-9871145498
Website: www.physicsbyfiziks.com | Email: fiziks.physics@gmail.com
19
fiziks
Institute for NET/JRF, GATE, IIT‐JAM, M.Sc. Entrance, JEST, TIFR and GRE in Physics

Now, Re s  2   lim  z  2 
 5z  3z   52 3 2 3
 322 40  12
  13
z 2 z  2  z  2  4 4

I  2 i  residue  2 i  13  26  3.14  I  81.64

GATE-2019
d2y y
Q46. For the differential equation  n  n  1 2  0 , where n is a constant, the product of
dx 2
x
its two independent solutions is
1 1
(a) (b) x (c) x n (d)
x x n 1
Ans. : (b)
Q47. During a rotation, vectors along the axis of rotation remain unchanged. For the rotation
0 1 0
 
matrix  0 0 1 , the vector along the axis of rotation is
 1 0 0 
 

(a)
3

1 ˆ ˆ
2i  j  2kˆ  (b)
1 ˆ ˆ ˆ
3

i  jk 
(c)
1 ˆ ˆ ˆ
3

i  jk  (d)
1 ˆ
3

2i  2 ˆj  kˆ 
Ans. : (b)
Q48. The pole of the function f  z   cot z at z  0 is

(a) a removable pole (b) an essential singularity


(c) a simple pole (d) a second order pole
Ans. : (c)
Solution: f  z   cot z at z  0

1 1 1 2 
f  z  z  0 is a simple pole f  z   1  z  ....
tan z z 3 

cos  kx 
Q49. The value of the integral 

x2  a2
dx , where k  0 and a  0 , is

 2  ka  3  ka
(a) e  ka (b) e (c) e  ka (d) e
a a 2a 2a
H.No. 40-D, Ground Floor, Jia Sarai, Near IIT, Hauz Khas, New Delhi-110016
Phone: 011-26865455/+91-9871145498
Website: www.physicsbyfiziks.com | Email: fiziks.physics@gmail.com
20
fiziks
Institute for NET/JRF, GATE, IIT‐JAM, M.Sc. Entrance, JEST, TIFR and GRE in Physics

Ans. : (a)

cos kx
Solution: x

2
 a2
dx

eikx eikz
f  z  
z 2  a 2  z  ia  z  ia 

eik  ia   e  ka
I  Re.2 i  
2ia a
Q50. Let  be a variable in the range      . Now consider a function
  
1 for  
     2 2
0 otherwise

if its Fourier-series is written as      m  Cm e  im , then the value of C3


 2

(rounded off to three decimal places) is__________

H.No. 40-D, Ground Floor, Jia Sarai, Near IIT, Hauz Khas, New Delhi-110016
Phone: 011-26865455/+91-9871145498
Website: www.physicsbyfiziks.com | Email: fiziks.physics@gmail.com
21
fiziks
Institute for NET/JRF, GATE, IIT‐JAM, M.Sc. Entrance, JEST, TIFR and GRE in Physics

CLASSICAL MECHANICS SOLUTIONS


GATE- 2010
Q1. For the set of all Lorentz transformations with velocities along the x -axis consider the
two statements given below:
P: If L is a Lorentz transformation, then, L1 is also a Lorentz transformation.
Q: If L1 and L2 are Lorentz transformations, then L1 L2 is necessarily a Lorentz

transformation.
Choose the correct option
(a) P is true and Q is false (b) Both P and Q are true
(c) Both P and Q are false (d) P is false and Q is true
Ans: (b)
1 2  3
Q2. A particle is placed in a region with the potential V x   kx  x , where k ,   0 .
2 3
Then,
k
(a) x  0 and x  are points of stable equilibrium

k
(b) x  0 is a point of stable equilibrium and x  is a point of unstable equilibrium

k
(c) x  0 and x  are points of unstable equilibrium

(d) There are no points of stable or unstable equilibrium
Ans: (b)
1 2 x 3 V k
Solution: V  kx    kx  x 2  0  x  0, x  .
2 3 x 
 2V
  k  2x
x 2
 2V k  2V
 At x  0,   ve (Stable) and at x  ,  ve (unstable)
x 2  x 2

H.No. 40-D, Ground Floor, Jia Sarai, Near IIT, Hauz Khas, New Delhi-110016
Phone: 011-26865455/+91-9871145498
Website: www.physicsbyfiziks.com | Email: fiziks.physics@gmail.com
1
fiziks
Institute for NET/JRF, GATE, IIT‐JAM, M.Sc. Entrance, JEST, TIFR and GRE in Physics

Q3. A  0 meson at rest decays into two photons, which moves along the x -axis. They are
both detected simultaneously after a time, t  10 s . In an inertial frame moving with a
velocity v  0.6 c in the direction of one of the photons, the time interval between the two
detections is
(a) 15c (b) 0 s (c) 10 s (d) 20 s
Ans: (a)
Solution:
v v
1 1
t1  t 0 c  10 1  0.6  10  2  20sec t  t c  10 1  0.6  10  1  5sec
,
1  0.6 1  0.6
2 0
v v 2
1 1
c c
 t1  t 2  15sec

Statement for Linked Answer Questions 4 and 5:


ml   mgl 1  cos  
1 2 2
The Lagrangian for a simple pendulum is given by L 
2
Q4. Hamilton’s equations are then given by
p p
(a) p   mgl sin  ;   2 (b) p   mgl sin  ;   2
ml ml
p g p
(c) p   m;    (d) p     ;  
m l ml
Ans: (a)
P2 H H P
Solution: H   mgl 1  cos      P  P  mgl sin  ;       2 .
2ml 2
 P ml

Q5. The Poisson bracket between  and  is

 
(a)  ,  1   1
(b)  ,  2
ml

 
(c)  , 
1
m
 
(d)  , 
g
l
Ans: (b)

   P  P 1   
Solution:  ,   ,  2  where    2  2  
 P  1 1
  1  2  0  2 .
 ml  ml ml   P P   ml ml

H.No. 40-D, Ground Floor, Jia Sarai, Near IIT, Hauz Khas, New Delhi-110016
Phone: 011-26865455/+91-9871145498
Website: www.physicsbyfiziks.com | Email: fiziks.physics@gmail.com
2
fiziks
Institute for NET/JRF, GATE, IIT‐JAM, M.Sc. Entrance, JEST, TIFR and GRE in Physics

GATE- 2011
1  q
Q6. A particle is moving under the action of a generalized potential V q, q   . The
q2

magnitude of the generalized force is


21  q  21  q  2 q
(a) (b) (c) (d)
q3 q3 q3 q3

Ans: (c)
d  V  V 2
Solution:    Fq  Fq  3
dt  q  q q
Q7. Two bodies of mass m and 2m are connected by a spring constant k . The frequency of
the normal mode is
(a) 3k / 2m (b) k/m (c) 2k / 3m (d) k / 2m

Ans: (a)
Solution: m 2m
k

k k 3k 2mm 2m
   where reduce mass    .
 2m 2m 2m  m 3
3
Q8. Let  p, q  and  P, Q  be two pairs of canonical variables. The transformation

Q  q  cosp  , P  q  sinp 

is canonical for
1 1
(a)   2,   (b)   2,   2 (c)   1,   1 (d)   ,   2
2 2
Ans: (d)
Q P Q P
Solution:    1
q p p q

 q  1 cosp   q   cosp   q    sin p   q  1 sin p   1

q 2 1  cos 2 p  sin 2 p   1  q 2 1  1    ,   2 .


1
2

H.No. 40-D, Ground Floor, Jia Sarai, Near IIT, Hauz Khas, New Delhi-110016
Phone: 011-26865455/+91-9871145498
Website: www.physicsbyfiziks.com | Email: fiziks.physics@gmail.com
3
fiziks
Institute for NET/JRF, GATE, IIT‐JAM, M.Sc. Entrance, JEST, TIFR and GRE in Physics

Q9. Two particles each of rest mass m collide head-on and stick together. Before collision, the
speed of each mass was 0.6 times the speed of light in free space. The mass of the final
entity is
(a) 5m / 4 (b) 2m (c) 5m / 2 (d) 25m / 8
Ans: (c)
Solution: From conservation of energy
mc 2 mc 2 2mc 2
  m1c 2   m1c 2
2 2 2
v v v
1 1 1
c2 c2 c2

Since v  0.6c  m1  5m / 2

GATE- 2012
Q10. In a central force field, the trajectory of a particle of mass m and angular momentum L in
plane polar coordinates is given by,
1 m
 1   cos  
r l2
where,  is the eccentricity of the particle’s motion. Which one of the following choice
for  gives rise to a parabolic trajectory?
(a)   0 (b)   1 (c) 0    1 (d)   1
Ans: (b)
1 m
Solution:  1   cos  
r l2
For parabolic trajectory   1 .
Q11. A particle of unit mass moves along the x-axis under the influence of a potential,
V  x   x x  2 . The particle is found to be in stable equilibrium at the point x  2 . The
2

time period of oscillation of the particle is


 3
(a) (b)  (c) (d) 2
2 2
Ans: (b)
V 2
Solution: V  x   x  x  2     x  2   2 x  x  2   0  x  2, x 
2 2

x 3

H.No. 40-D, Ground Floor, Jia Sarai, Near IIT, Hauz Khas, New Delhi-110016
Phone: 011-26865455/+91-9871145498
Website: www.physicsbyfiziks.com | Email: fiziks.physics@gmail.com
4
fiziks
Institute for NET/JRF, GATE, IIT‐JAM, M.Sc. Entrance, JEST, TIFR and GRE in Physics

 2V  2V
 2  x  2  
 2 x  2   2 x   2 2  4
x 2 x 2 x2

 2V 2
     2 T 
x 2 x 2
T

Q12. A rod of proper length l0 oriented parallel to the x-axis moves with speed 2c / 3 along the

x -axis in the S -frame, where c is the speed of light in free space. The observer is also
moving along the x -axis with speed c / 2 with respect to the S -frame. The length of the
rod as measured by the observer is
(a) 0.35l0 (b) 0.48l0 (c) 0.87 l0 (d) 0.97 l0
Ans: (d)

u2x
Solution: l  l0 1   0.97 l0
c2
Q13. A particle of mass m is attached to a fixed point O by a weightless z
inextensible string of length a . It is rotating under the gravity as
 O
shown in the figure. The Lagrangian of the particle is
a
 
L ,    ma 2  2  sin 2  2  mga cos  where  and  are the polar
1
2 m

angles. The Hamiltonian of the particles is g

1  2 p2  1  2 p2 
(a) H   p    mga cos  (b) H   p    mga cos
2ma 2  sin 2
  2ma 2  sin 2
 
   

(c) H 
1
2 ma 2
 
p2  p2  mga cos  (d) H 
1
2ma 2
 
p2  p2  mga cos 

Ans: (b)
1
 
Solution: H  P  P  L  P   P   ma 2  2  sin 2  2  mga cos 
2
L P L P
 P  ma 2  P     2 and P    ma 2 sin 2    
 ma  ma sin 2 
2

Put the value of  and 

H.No. 40-D, Ground Floor, Jia Sarai, Near IIT, Hauz Khas, New Delhi-110016
Phone: 011-26865455/+91-9871145498
Website: www.physicsbyfiziks.com | Email: fiziks.physics@gmail.com
5
fiziks
Institute for NET/JRF, GATE, IIT‐JAM, M.Sc. Entrance, JEST, TIFR and GRE in Physics

1 2   P   
2

2
P P P
H  P  2  P  2 2  ma   2   sin   2 2    mga cos 
2

ma ma sin  2   ma   ma sin   

P2 P2 P2 P2


H     mga cos
ma 2 2ma 2 ma 2 sin 2  2ma 2 sin 2 

1  2 P2 
H  P  2   mga cos 
2ma 2  sin  

Statement for Linked Answer Questions 14 and 15:


Q14. A particle of mass m slides under the gravity without friction along the parabolic path
y  ax 2 , as shown in the figure. Here a is a constant.
y

x
The Lagrangian for this particle is given by

(a) L 
1
2
mx 2  mgax 2 (b) L 
1
2
 
m 1  4a 2 x 2 x 2  mgax 2

(c) L 
1
2
mx 2  mgax 2 (d) L 
1
2
 
m 1  4a 2 x 2 x 2  mgax 2

Ans: (b)

m  x 2  y 2 
1
Solution: Equation of constrain is given by y  ax 2 , K .E., T 
2

m  x 2  4a 2 x 2 x 2   mx 2 1  4ax 2 
1 1
y  2axx  T 
2 2

V  mgy  mgax 2 .

m 1  4a 2 x 2  x 2  mgax 2
1
 L  T V  L 
2

H.No. 40-D, Ground Floor, Jia Sarai, Near IIT, Hauz Khas, New Delhi-110016
Phone: 011-26865455/+91-9871145498
Website: www.physicsbyfiziks.com | Email: fiziks.physics@gmail.com
6
fiziks
Institute for NET/JRF, GATE, IIT‐JAM, M.Sc. Entrance, JEST, TIFR and GRE in Physics

Q15. The Lagrange’s equation of motion of the particle for above question is given by
(a) x  2 gax (b) m 1  4a 2 x 2  
x  2mgax  4ma 2 xx 2

 
(c) m 1  4a 2 x 2 x  2mgax  4ma 2 xx 2 (d) x  2 gax
Ans: (b)
d  dL  dL
Solution:    mx 1  4a 2 x 2   4ma 2 xx 2  2mgax
dt  dx  dx

GATE- 2013
Q16. In the most general case, which one of the following quantities is NOT a second order
tensor?
(a) Stress (b) Strain
(c) Moment of inertia (d) Pressure
Ans: (b)
Solution: Strain is not a tensor.
Q17. An electron is moving with a velocity of 0.85c in the same direction as that of a moving
photon. The relative velocity of the electron with respect to photon is
(a) c (b)  c (c) 0.15c (d)  0.15c
Ans: (b)
Q18. The Lagrangian of a system with one degree of freedom q is given by L  q 2  q 2 ,
where  and  are non-zero constants. If p q denotes the canonical momentum

conjugate to q then which one of the following statements is CORRECT?


(a) p q  2  q and it is a conserved quantity.

(b) p q  2  q and it is not a conserved quantity.

(c) p q  2 q and it is a conserved quantity.

(d) p q  2q and it is not a conserved quantity.

Ans: (d)
L L
Solution: As,  pq but  0 . Thus, it is not a conserved quantity.
q q

H.No. 40-D, Ground Floor, Jia Sarai, Near IIT, Hauz Khas, New Delhi-110016
Phone: 011-26865455/+91-9871145498
Website: www.physicsbyfiziks.com | Email: fiziks.physics@gmail.com
7
fiziks
Institute for NET/JRF, GATE, IIT‐JAM, M.Sc. Entrance, JEST, TIFR and GRE in Physics

Q19. The relativistic form of Newton’s second law of motion is

mc dv m c 2  v 2 dv
(a) F  (b) F 
c 2  v 2 dt c dt

mc 2 dv c 2  v 2 dv
(c) F  (d) F  m
c 2  v 2 dt c2 dt
Ans: (c)
mv dP dv 1  1 1 2v dv
Solution: P  F m   mv      2
 2   v2 
3/ 2
v2 dt dt v2 c dt
1 1 2  1  2 
c2 c  c 

   
 v 2 
2  v2 2 
1 
F m
dv 1  1
1 c   m dv  2c 
 
v  2 1  v   dt   v 2  3 2
2
dt 2

1  2   
2 
c   c   1  c 2  
  
  1  v2 / c2  
1/ 2
2
dv    mc dv
m
dt  1  v 2 / c 2 1  v 2 / c 2    c  v  dt
1/ 2 2 2
 
Q20. Consider two small blocks, each of mass M, attached to two identical springs. One of the
springs is attached to the wall, as shown in the figure. The spring constant of each spring
is k . The masses slide along the surface and the friction is negligible. The frequency of
one of the normal modes of the system is,

3 2 k
(a)
2 M

3 3 k
(b)
2 M
k k
3 5 k M M
(c)
2 M

3 6 k
(d)
2 M
Ans: (c)

H.No. 40-D, Ground Floor, Jia Sarai, Near IIT, Hauz Khas, New Delhi-110016
Phone: 011-26865455/+91-9871145498
Website: www.physicsbyfiziks.com | Email: fiziks.physics@gmail.com
8
fiziks
Institute for NET/JRF, GATE, IIT‐JAM, M.Sc. Entrance, JEST, TIFR and GRE in Physics

1 2 1 2
Solution: T  mx1  mx 2 ,
2 2

V 
2 2
2

2 2
  1
2

kx1  k  x 2  x1   kx12  k x 22  x12  2 x 2 x1  k 2 x12  x22  2 x2 x1
1 2 1 1 1

m 0   2k k 
T  ; V   
 0 m  k k 

2k   2 m k 3 5
 0   2 k   2 m  k   2 m   k 2  0   
k
k k  m2
2 m

GATE- 2014
Q21. If the half-life of an elementary particle moving with speed 0.9c in the laboratory frame is

5  10 8 s, then the proper half-life is _______________ 10 8 s. c  3  10 8 m / s 
Ans: 2.18
t0 v2
Solution: t  , t0  t  1  2
 5 108  0.19  2.18 108 s
v2 c
1
c2
Q22. Two masses m and 3m are attached to the two ends of a massless spring with force
constant K . If m  100 g and K  0.3 N / m , then the natural angular frequency of
oscillation is ________ Hz .
Ans: 0.318
1 k m .m 3m.m 3m 4k
Solution: f  ,  1 2   ,   2  f  0.318 Hz
2  m1  m2 4m 4 3m
Q23. The Hamilton’s canonical equation of motion in terms of Poisson Brackets are
(a) q  q, H ; p  p, H  (b) q  H , q; p  H , p

(c) q  H , p; p  H , p (d) q  p, H ; p  q, H 


Ans: (a)
df f q f p f
Solution:  .  . 
dt q t p t t
df f H f H f df f
 .  .     f , H 
dt q p p q t dt t
dq dp
 q, H  and   p, H 
dt dt

H.No. 40-D, Ground Floor, Jia Sarai, Near IIT, Hauz Khas, New Delhi-110016
Phone: 011-26865455/+91-9871145498
Website: www.physicsbyfiziks.com | Email: fiziks.physics@gmail.com
9
fiziks
Institute for NET/JRF, GATE, IIT‐JAM, M.Sc. Entrance, JEST, TIFR and GRE in Physics

Q24. A bead of mass m can slide without friction along a massless rod kept at 45 o with the
vertical as shown in the figure. The rod is rotating about the vertical axis with a constant
angular speed  . At any instant r is the distance of the bead from the origin. The

momentum conjugate to r is

(a) mr
1
(b) mr
2
1 m
(c) mr
2
45 o
(d) 2mr r
Ans: (a) x̂
1
Solution: L  m(r 2  r 2 2  r 2 sin 2  2 )  mgr cos 
2

Equation of constrain is   and it is given   
4
1 1 1
L m(r 2  r 2 2 )  mgr
2 2 2
L
Thus the momentum conjugate to r is p r   pr  mr
r
Q25. A particle of mass m is in a potential given by
a ar 2
V  r     03
r 3r
where a and r0 are positive constants. When disturbed slightly from its stable
equilibrium position it undergoes a simple harmonic oscillation. The time period of
oscillation is
3 3
mr03 m r0 2m r0 mr03
(a) 2 (b) 2 (c) 2 (d) 4
2a a a a
Ans: (a)
a ar 2
Solution: V  r     03 ,
r 3r
V a 3ar02
For equilibrium    0 , r   r0
r r 2 3r 4
 2V 2a 4ar02 2a 4ar02 2a
      5  3
r 2 r3 r5 r r03 r0 r0
0

H.No. 40-D, Ground Floor, Jia Sarai, Near IIT, Hauz Khas, New Delhi-110016
Phone: 011-26865455/+91-9871145498
Website: www.physicsbyfiziks.com | Email: fiziks.physics@gmail.com
10
fiziks
Institute for NET/JRF, GATE, IIT‐JAM, M.Sc. Entrance, JEST, TIFR and GRE in Physics

 2V
r 2 mr03
  T  2
r0

m 2a
Q26. A planet of mass m moves in a circular orbit of radius r0 in the gravitational potential

V r    , where k is a positive constant. The orbit angular momentum of the planet is


k
r
(a) 2r0 km (b) 2r0 km (c) r0 km (d) r0 km

Ans: (d)
J2 k dVeffect J2 k
Solution: Veffctive  2
    3
 2  0 at r  r0
2mr r dr mr r
so J  r0 km

Q27. Given that the linear transformation of a generalized coordinate q and the corresponding
momentum p , Q  q  4ap, P  q  2 p is canonical, the value of the constant a is ____
Ans: 0.25
Q P Q P
Solution: .  .  1  1 2  4a  1  1  a  0.25
q p p q

p2  q2
Q28. The Hamiltonian of particle of mass m is given by H   . Which one of the
2m 2
following figure describes the motion of the particle in phase space?
(a) p (b) p

q q

(c) p (d) p

q q

H.No. 40-D, Ground Floor, Jia Sarai, Near IIT, Hauz Khas, New Delhi-110016
Phone: 011-26865455/+91-9871145498
Website: www.physicsbyfiziks.com | Email: fiziks.physics@gmail.com
11
fiziks
Institute for NET/JRF, GATE, IIT‐JAM, M.Sc. Entrance, JEST, TIFR and GRE in Physics

Ans: (d)
GATE- 2015
Q29. A satellite is moving in a circular orbit around the Earth. If T ,V and E are its average
kinetic, average potential and total energies, respectively, then which one of the
following options is correct?
(a) V  2T ; E  T (b) V  T ; E  0
T T  3T T
(c) V   ;E  (d) V  ;E 
2 2 2 2
Ans.: (a)
n 1
Solution: From Virial theorem T  V where V  r n 1
2
k 1
V   V   n  2  V  2 T
r r

Q30. In an inertial frame S , two events A and B take place at ct A  0, rA  0  and
ct B  0, rB  2 yˆ  , respectively. The times at which these events take place in a frame
S  moving with a velocity 0.6c yˆ with respect to S are given by
3
(a) ct A  0; ct B   (b) ct A  0; ct B  0
2
3 1
(c) ct A  0; ct B  (d) ct A  0; ct B 
2 2
Ans.: (a)
Solution: Velocity of S ' with respect to S is v  0.6c
v
tA  y
t A'  c2
v2
1 2
c
For event A, t A  0, y  0 . So ct A'  0

v
tB  y
t B'  c2
v2
1 2
c

H.No. 40-D, Ground Floor, Jia Sarai, Near IIT, Hauz Khas, New Delhi-110016
Phone: 011-26865455/+91-9871145498
Website: www.physicsbyfiziks.com | Email: fiziks.physics@gmail.com
12
fiziks
Institute for NET/JRF, GATE, IIT‐JAM, M.Sc. Entrance, JEST, TIFR and GRE in Physics

3
For event B, t B  0, y  2 . So ct B'  
2

 
Q31. The Lagrangian for a particle of mass m at a position r moving with a velocity v is given
m  
by L  v 2  Cr .v  V r  , where V r  is a potential and C is a constant. If pc is the
2
canonical momentum, then its Hamiltonian is given by
1 
(a)  pc  Cr 2  V r  (b)
1 
 pc  Cr 2  V r 
2m 2m
pc2
 V r  pc  C 2 r 2  V r 
1 2
(c) (d)
2m 2m
Ans.: (b)
m 2 
Solution: L  v  Cr.v  V  r  where v  r
2
H   r pc  L  rp
 c L

L p  Cr
  pc   mr  Cr   r  c
r m

 p  Cr  m  pc  Cr   pc  Cr 
2

 H  c  pc     cr   V r 
 m  2 m   m 

 p  Cr  m  pc  Cr 
2

 H  c   pc  Cr     V r 
 m  2 m 

 p  Cr   p  Cr 
2 2
1
H  c  c V r  H   pc  Cr   V  r 
2

m 2m 2m
 
Q32. The Hamiltonian for a system of two particles of masses m1 and m2 at r1 and r2 having
  1 1 C  
velocities v1 and v2 is given by H  m1v12  m2 v22    2 zˆ   r1  r2  , where C is
2 2  r1  r2 
constant. Which one of the following statements is correct?
(a) The total energy and total momentum are conserved
(b) Only the total energy is conserved
(c) The total energy and the z - component of the total angular momentum are conserved
(d) The total energy and total angular momentum are conserved
Ans.: (c)

H.No. 40-D, Ground Floor, Jia Sarai, Near IIT, Hauz Khas, New Delhi-110016
Phone: 011-26865455/+91-9871145498
Website: www.physicsbyfiziks.com | Email: fiziks.physics@gmail.com
13
fiziks
Institute for NET/JRF, GATE, IIT‐JAM, M.Sc. Entrance, JEST, TIFR and GRE in Physics
 
Solution: Lagrangian is not a function of time, so energy is conserved and component of  r1  r2 

is only in ẑ direction means potential is symmetric under  , so Lz is conserved.

Q33. A particle of mass 0.01 kg falls freely in the earth’s gravitational field with an initial

velocity 0  10ms 1 . If the air exerts a frictional force of the form, f   kv , then for

k  0.05 Nm 1 s , the velocity (in ms 1 ) at time t  0.2 s is _________ (upto two decimal

places). (use g  10 ms 2 and e  2.72 )


Ans.: 4.94
u 0.2
dv dv k dv dv
Solution: m  mg  kv  g v   dt     dt
dt dt m k k
g v 10 g v 0
m m

m   10k  
u
m  k  k  
   ln  g  v    t 0    ln  g  u    ln  g 
0.2
   0.2
k   m  10 k   m   m 

m  0.05   .05  
 ln 10  u   ln 10  10     0.2
k  0.01   .01  


m
k
ln 10  5u   ln  40   0.2
 8  0.2k  8  0.2k 8
ln    ln    e
u2 m u2 m u2
8
 u   2  4.94 m / s
e
Q34. Consider the motion of the Sun with respect to the rotation of the Earth about its axis. If
 
Fc and FCo denote the centrifugal and the Coriolis forces, respectively, acting on the Sun,

then
     
(a) Fc is radially outward and FCo  Fc (b) Fc is radially inward and FCo  2 Fc
     
(c) Fc is radially outward and FCo  2 Fc (d) Fc is radially outward and FCo  2 Fc
Ans.: (b)
Q35. A particle with rest mass M is at rest and decays into two particles of equal rest masses
3
M which move along the z axis. Their velocities are given by
10

H.No. 40-D, Ground Floor, Jia Sarai, Near IIT, Hauz Khas, New Delhi-110016
Phone: 011-26865455/+91-9871145498
Website: www.physicsbyfiziks.com | Email: fiziks.physics@gmail.com
14
fiziks
Institute for NET/JRF, GATE, IIT‐JAM, M.Sc. Entrance, JEST, TIFR and GRE in Physics
   
(a) v1  v 2  0.8c zˆ (b) v1  v 2  0.8c zˆ
   
(c) v1  v 2  0.6c zˆ (d) v1  0.6c zˆ; v 2   0.8c zˆ
Ans.: (b)
3 3
Solution: M M M
10 10
From momentum conservation
   
0  P1  P 2  P1   P 2  P1  P2
From energy conservation
E  E1  E2

3 Mc 2 3 Mc 2 3 Mc 2
 Mc 2    Mc 2 
10 v 2 10 v2 5 v2
1 2 1 2 1 2
c c c
 v2  9 v 2 16
 1  2 
    v  0.8c
 c  25 c 2 25

GATE-2016
Q36. The kinetic energy of a particle of rest mass m0 is equal to its rest mass energy. Its

momentum in units of m0 c , where c is the speed of light in vacuum, is _______.


(Give your answer upto two decimal places)
Ans. : 1.73
Solution: m0c 2  E  m0c 2  E  2m0 c 2

m0 c 2 3
  2m0c 2  v  c
v2 2
1
c2

 E 2  p 2 c 2  m02 c 4  4m02 c 4  m02 c 4  p 2 c 2  p  3m0 c  1.732m0 c

Q37. In an inertial frame of reference S , an observer finds two events occurring at the same
time at coordinates x1  0 and x 2  d . A different inertial frame S  moves with velocity
v with respect to S along the positive x -axis. An observer in S  also notices these two

H.No. 40-D, Ground Floor, Jia Sarai, Near IIT, Hauz Khas, New Delhi-110016
Phone: 011-26865455/+91-9871145498
Website: www.physicsbyfiziks.com | Email: fiziks.physics@gmail.com
15
fiziks
Institute for NET/JRF, GATE, IIT‐JAM, M.Sc. Entrance, JEST, TIFR and GRE in Physics

events and finds them to occur at times t1 and t 2 and at positions x1 and x2 respectively.
1
If t   t 2  t1 , x   x 2  x1 and   , which of the following statements is true?
v2
1 2
c
d
(a) t   0, x   d (b) t   0, x  

 vd  vd d
(c) t   , x   d (d) t   , x  
c2 c 2

Ans.: (c)
 vx   vx 
 t2  22   t1  21 
Solution: t2  t1   c  c   t   t   vx
 v2   v2  c2
 1  2   1  2 
 c   c 
It is given, t  0, x  d
 vx  vd
 t    2

c c2
   
   
x  vt2    x1  vt1
x2  x1   2
    x    x  vt 
 v2   v2 
 1  2   1  2 
 c   c 
 x   d .
Q38. The Lagrangian of a system is given by

L
1 2 2
2
 
ml   sin 2  2  mgl cos  , where m, l and g are constants.

Which of the following is conserved?


 
(a)  sin 2  (b)  sin  (c) (d)
sin  sin 2 
Ans.: (a)
L
Solution: As  is cyclic coordinate, so  p  ml 2 sin 2  , is a constant since m, l and g are


constants. Thus  sin 2  is conserved.

H.No. 40-D, Ground Floor, Jia Sarai, Near IIT, Hauz Khas, New Delhi-110016
Phone: 011-26865455/+91-9871145498
Website: www.physicsbyfiziks.com | Email: fiziks.physics@gmail.com
16
fiziks
Institute for NET/JRF, GATE, IIT‐JAM, M.Sc. Entrance, JEST, TIFR and GRE in Physics

Q39. A particle of rest mass M is moving along the positive x -direction. It decays into two
photons  1 and  2 as shown in the figure. The energy of  1 is 1 GeV and the energy of
GeV
 2 is 0.82 GeV . The value of M (in units of ) is ________. (Give your answer
c2
1
upto two decimal places)

M 450
600

2
Ans.: 1.44

Solution: p 2 c 2  M 2 c 4  E1  E2  1.82GeV

E1 E 1GeV 1 0.82GeV 1 1.11GeV


p cos 1  2 cos  2   
c c c 2 c 2 c

 p 2 c 2  m 2c 4  3.312  m 2 c 4  3.312  1.23  2.08

 m  2.076  1.44
GATE- 2017
2
1  dq  1  dq 
Q40. If the Lagrangian L0  m    m 2 q 2 is modified to L  L0   q   , which one
2  dt  2  dt 
of the following is TRUE?
(a) Both the canonical momentum and equation of motion do not change
(b) Canonical momentum changes, equation of motion does not change
(c) Canonical momentum does not change, equation of motion changes
(d) Both the canonical momentum and equation of motion change
Ans. : (b)
2
1  dq  1
Solution: For Lagrangian L0  m    m 2 q 2 canonical momentum is p  mq and
2  dt  2

d  L   L 
       0  mq  m q  0
2
equation of motion is given by
dt  q   q 

H.No. 40-D, Ground Floor, Jia Sarai, Near IIT, Hauz Khas, New Delhi-110016
Phone: 011-26865455/+91-9871145498
Website: www.physicsbyfiziks.com | Email: fiziks.physics@gmail.com
17
fiziks
Institute for NET/JRF, GATE, IIT‐JAM, M.Sc. Entrance, JEST, TIFR and GRE in Physics

2
 dq  1  dq  1
For Lagrangian L  L0   q    L  m    m 2 q 2   qq
 dt  2  dt  2
Canonical momentum is p  mq   q
Equation of motion is,
d  L   L 
      0  mq  m q  0
2
dt  q   q 
Q41. Two identical masses of 10 gm each are connected by a massless spring of spring
constant 1 N / m . The non-zero angular eigenfrequency of the system is…………rad/s.
(up to two decimal places)
Ans. : 14.14

k m 10 1
Solution:   , where     and k  1N / m ,   14.14
 2 2 1000 200
Q42. The phase space trajectory of an otherwise free particle bouncing between two hard walls
elastically in one dimension is a
(a) straight line (b) parabola (c) rectangle (d) circle
Ans. : (c)
p2
Solution: E  , p   2mE
2m
Q43. The Poisson bracket  x, xp y  ypx  is equal to

(a)  x (b) y (c) 2 px (d) p y

Ans. : (b)
Solution:  x, xp y  ypx    x, xp y    x, ypx   0  y  x, px   y

c
Q44. An object travels along the x -direction with velocity
in a frame O . An observer in a
2
c
frame O sees the same object travelling with velocity . The relative velocity of O
4
with respect to O in units of c is…………….. (up to two decimal places).
Ans. : 0.28

H.No. 40-D, Ground Floor, Jia Sarai, Near IIT, Hauz Khas, New Delhi-110016
Phone: 011-26865455/+91-9871145498
Website: www.physicsbyfiziks.com | Email: fiziks.physics@gmail.com
18
fiziks
Institute for NET/JRF, GATE, IIT‐JAM, M.Sc. Entrance, JEST, TIFR and GRE in Physics

c c
u '
 v 
c c 2c
Solution: u x'  , v  , u x  x '  2 4   0.28c
2 4 uxv 1  c . c . 1 7
1 2
c 2 4 c2
Q45. A uniform solid cylinder is released on a horizontal surface with speed 5 m / s without
any rotation (slipping without rolling). The cylinder eventually starts rolling without
slipping. If the mass and radius of the cylinder are 10 gm and 1cm respectively, the final
linear velocity of the cylinder is…………… m / s . (up to two decimal places).
Ans. : 3.33
1 v 3 2 10
Solution: mvr  mvcm r  I cm  mvcm r  mr 2 cm  v  vcm  vcm  v   3.33m / sec
2 r 2 3 3
Q46. A person weighs wp at Earth’s north pole and we at the equator. Treating the Earth as a

perfect sphere of radius 6400 km , the value 100 


w p  we 
is………….. (up to two
wp
decimal places). (Take g  10 ms 2 ).
Ans. : 0.33
w  we  2 R
Solution: g p  g , ge  g   2 R  100  p 
wp g
Now, g  10 m / sec2 and R  6400 103 m
2 2
  
T 24  3600

wp  we
Then 100   0.33
wp

H.No. 40-D, Ground Floor, Jia Sarai, Near IIT, Hauz Khas, New Delhi-110016
Phone: 011-26865455/+91-9871145498
Website: www.physicsbyfiziks.com | Email: fiziks.physics@gmail.com
19
fiziks
Institute for NET/JRF, GATE, IIT‐JAM, M.Sc. Entrance, JEST, TIFR and GRE in Physics

GATE - 2018
Q47. In the context of small oscillations, which one of the following does NOT apply to the
normal coordinates?
(a) Each normal coordinate has an eigen-frequency associated with it
(b) The normal coordinates are orthogonal to one another
(c) The normal coordinates are all independent
(d) The potential energy of the system is a sum of squares of the normal coordinates with
constant coefficients
Ans. : (b)
Solution: Normal co-ordinate must be independent. It is not necessary that it should orthogonal.
Q48. A spaceship is travelling with a velocity of 0.7 c away from a space station. The
spaceship ejects a probe with a velocity 0.59 c opposite to its own velocity. A person in
the space station would see the probe moving at a speed Xc , where the value of X is
___________ (up to three decimal places).
Ans.: 0.187 c
Spacestation
Spaceship
Solution:   0  7c , ux  0  59 c , Prob
  0  7c
ux  u x  0.59 c
ux 
u
1 x
c2
0  59c  0  7c 0.11c 0.11c
ux     0.187 c
1  0  7  0  59 1  0.413 0.587
Q49. An interstellar object has speed v at the point of its shortest distance R from a star of
much larger mass M . Given v 2  2GM / R , the trajectory of the object is
(a) circle (b) ellipse (c) parabola (d) hyperbola
Ans. : (c)
J2 GMm
Solution: At shortest distance E  2

2mR R
Since, mvR  J  J 2  m 2 v 2 R 2
2GM
Now, J 2  m2 2GMR  2GMm2 R (Given that v 2  )
R

H.No. 40-D, Ground Floor, Jia Sarai, Near IIT, Hauz Khas, New Delhi-110016
Phone: 011-26865455/+91-9871145498
Website: www.physicsbyfiziks.com | Email: fiziks.physics@gmail.com
20
fiziks
Institute for NET/JRF, GATE, IIT‐JAM, M.Sc. Entrance, JEST, TIFR and GRE in Physics

2GMm 2 R GMm GMm GMm


E    0
2mR 2 R R R
For Kepler’s potential, if energy is zero, then the shape is parabola.
Q50. A particle moves in one dimension under a potential V  x    x with some non-zero

total energy. Which one of the following best describes the particle trajectory in the phase
space?
p p

(a) (b)
x x

p p

(c) (d)
x x

Ans.: (a)
p2 V  x
Solution: E   x
2m
p2 x
For x  0 , E   x
2m
 p 2  2m  E   x 
px
p2
For x  0 , E   x x
2m
 p 2  2m  E   x 

Q51. If H is the Hamiltonian for a free particle with mass m , the commutator  x,  x, H  is

(a)  2 / m (b)   2 / m (c)   2 /  2m  (d)  2 /  2m 

Ans. : (b)
Solution: For free particle, potential is zero.
Px2
H 
2m

H.No. 40-D, Ground Floor, Jia Sarai, Near IIT, Hauz Khas, New Delhi-110016
Phone: 011-26865455/+91-9871145498
Website: www.physicsbyfiziks.com | Email: fiziks.physics@gmail.com
21
fiziks
Institute for NET/JRF, GATE, IIT‐JAM, M.Sc. Entrance, JEST, TIFR and GRE in Physics

 P 2  2i
Now,  x, H    x, x   Px
 2 m  2 m

2i i 2
 x,  x, H    x  
x , P  i   
2m m m
Q52. For the transformation
Q  2q e 1 2 cos p, P  2q e  1 sin p

(where  is a constant) to be canonical, the value of  is _________.


Ans. : 2
Solution: Q  2qe 1 2 cos p, P  2qe  1.sin p

Since, Q, P   1

Q P Q P
  1
q p p q

1 
 
1
 2  12  .1
 2q 2 .e 1 2 cos p  2qe  1 cos p  2qe 1 2   sin p  . q e sin p  1
2  2

 e  2 . cos 2 p  sin 2 p   1  e0

  2
Q53. A uniform circular disc of mass m and radius R is rotating with angular 
speed  about an axis passing through its centre and making an angle

  300 with the axis of the disc. If the kinetic energy of the disc is
 m 2 R 2 , the value of  is__________ (up to two decimal places).
Ans. : 0.21
Solution: The kinetic energy of the disc is,
1 
T L 
2

Where L is angular momentum and  is angular velocity

1   1 3 1  mR 2  2 3
T  L  cos 30  I   
0
   
2 2 2 2 2  2
3
T m 2 R 2  0.21 m 2 R 2   m 2 R 2  0.21 m 2 R 2
8
Hence,   0.21

H.No. 40-D, Ground Floor, Jia Sarai, Near IIT, Hauz Khas, New Delhi-110016
Phone: 011-26865455/+91-9871145498
Website: www.physicsbyfiziks.com | Email: fiziks.physics@gmail.com
22
fiziks
Institute for NET/JRF, GATE, IIT‐JAM, M.Sc. Entrance, JEST, TIFR and GRE in Physics

GATE-2019
Q54. Consider a transformation from one set of generalized coordinate and momentum ( q, p )
to another set ( Q, P ) denoted by,

Q  pq s ; P  qr
where s and r are constants. The transformation is canonical if
(a) s  0 and r  1 (b) s  2 and r  1
(c) s  0 and r  1 (d) s  2 and r  1
Ans. : (b)
Q P Q P
Solution: .  .  1  0  q s rq r 1  1
q p p q

rq r  s 1  1  s  2 and r  1

p2
Q55. The Hamiltonian for a particle of mass m is H   kqt where q and p are the
2m
generalized coordinate and momentum, respectively, t is time and k is a constant. For
the initial condition, q  0 and p  0 at t  0, q  t   t  . The value of  is ________

Ans. : 3
H p
Solution:  q  ....(1)
p m

H kt 2
  p  kt  p   ….(2)
q 2

dq kt 2 kt 3
 q q  t3 so   3
dt 2 6

H.No. 40-D, Ground Floor, Jia Sarai, Near IIT, Hauz Khas, New Delhi-110016
Phone: 011-26865455/+91-9871145498
Website: www.physicsbyfiziks.com | Email: fiziks.physics@gmail.com
23
fiziks
Institute for NET/JRF, GATE, IIT‐JAM, M.Sc. Entrance, JEST, TIFR and GRE in Physics

Q56. A ball bouncing of a rigid floor is described by the potential energy function
mgx for x  0
V  x  
 for x  0
Which of the following schematic diagrams best represents the phase space plot of the
ball?
(a) (b)
 2mE  2mE
E E
mg mg
x x

 2mE  2mE

(c)
 2mE
(d)  2mE
E E E

mg mg mg
x x

 2mE
 2mE
Ans. : (b)
p2
Solution: E   mgx  p 2  2m  E  mgx  which is equation of parabola
2m
ap 2 q 4 
Q57. Consider the Hamiltonian H  q, p    2 , where  and  are parameters with
2 q
appropriate dimensions, and q and p are the generalized coordinate and momentum,

respectively. The corresponding Lagrangian L  q, q  is

1 q 2  1 q 2  1 q 2  1 q 2 
(a)  (b)  (c)  (d)  
2 q 4 q 2 2 q 4 q 2  q4 q2 2 q 4 q 2
Ans. : (a)
ap 2 q 4 
Solution: L  pq  H  pq   2 from Hamiltonian equation of motion
2 q
H q
 q  p  4
p aq
H.No. 40-D, Ground Floor, Jia Sarai, Near IIT, Hauz Khas, New Delhi-110016
Phone: 011-26865455/+91-9871145498
Website: www.physicsbyfiziks.com | Email: fiziks.physics@gmail.com
24
fiziks
Institute for NET/JRF, GATE, IIT‐JAM, M.Sc. Entrance, JEST, TIFR and GRE in Physics

1 q 2 
L 
2 q 4 q 2

Q58. A projectile of mass 1kg is launched at an angle of 300 from the horizontal direction at

t  0 and takes time T before hitting the ground. If its initial speed is 10 ms 1 , the value

of the action integral for the entire flight in the units of kgm 2 s 1 (round off to one

decimal place) is___________. [Take g  10 ms 2 ]


Ans. : 33.3
2v sin 
Solution: T   1sec
g

L
1
2

m x 2  y 2  mgy
x  v cos   5 3ms 1 y  v sin   gt  5  10t
1 1 1 1
y  ut  gt 2  v sin  t  gt 2  10. t  10t 2  5t  5t 2
2 2 2 2

L  1 5 3
1
  
  5  10t    1 10  5t  5t 2 
2 2

2  

L  100t 2  100t  50

 
T 1
A   Ldt   100t 2  100t  50 dt  33.3
0 0

Q59. Two spaceships A and B , each of the same rest length L , are moving in the same
4c 3c
direction with speeds and , respectively, where c is the speed of light. As
5 5
measured by B , the time taken by A to completely overtake B [see figure below] in
units of L / c (to the nearest integer) is _____________
(i) (ii)

A 4c / 5 A 4c / 5

B 3c / 5 B 3c / 5

Ans. : 5

H.No. 40-D, Ground Floor, Jia Sarai, Near IIT, Hauz Khas, New Delhi-110016
Phone: 011-26865455/+91-9871145498
Website: www.physicsbyfiziks.com | Email: fiziks.physics@gmail.com
25
fiziks
Institute for NET/JRF, GATE, IIT‐JAM, M.Sc. Entrance, JEST, TIFR and GRE in Physics

4 3 c
c c
5
Solution: u A, B  5 5  5  c
4 3 1 13 13
1  c. c. 2
5 5 c 25
Kinematic equation is given by

5 25 5L
c  t  L 1 Lt    5
13 169 c
Q60. Two events, one on the earth and the other one on the Sun, occur simultaneously in the
earth’s frame. The time difference between the two events as seen by an observer in a
spaceship moving with velocity 0.5 c in the earth’s frame along the line joining the earth
to the Sun is  t , where c is the speed of light. Given that light travels from the Sun to

the earth in 8.3 minutes in the earth’s frame, the value of  t in minutes (rounded off to

two decimal places) is____________


(Take the earth’s frame to be inertial and neglect the relative motion between the earth
and the sun)
Ans. : 4.77
Solution: t2'  t1'  0 x2'  x1'  8.3  3 108  60 v  0.5c

 vx '   vx '   
 t2'  22
t  t2  t1   c
  t1'  21
 c
  ' '
   t2  t1

 
x'  x'
  v 2 1  4.77 min
 v2   v2   v2  c2 v2
 1  2   1  2   1  2  1 2
 c   c   c  c

H.No. 40-D, Ground Floor, Jia Sarai, Near IIT, Hauz Khas, New Delhi-110016
Phone: 011-26865455/+91-9871145498
Website: www.physicsbyfiziks.com | Email: fiziks.physics@gmail.com
26
fiziks
Institute for NET/JRF, GATE, IIT‐JAM, M.Sc. Entrance, JEST, TIFR and GRE in Physics

ELECTROMAGNETIC THEORY SOLUTIONS


GATE- 2010
Q1. An insulating sphere of radius a carries a charge density


 r   0 a 2  r 2 cos  ; r  a .
The leading order term for the electric field at a distance d, far away from the charge
distribution, is proportional to
(a) d 1 (b) d 2 (c) d 3 (d) d 4
Ans: (c)
1 1 
Solution: V r     d  2   cos d   ,
r V r 
a  2

  d       a  r 2  cos   r 2 sin  drd d  0


st 2
I term, 0
0 0 0

a  2

  cos  d       a  r 2  cos 2   r 2 sin  drd d  0 .


nd 2
II term, 0
0 0 0

1 1
 V 2
 E 3
r r
Q2. Two magnetic dipoles of magnitude m each are placed in a plane as shown in figure.
m
The energy of interaction is given by
45 o 2
 m2
(a) Zero (b) 0 3
4d d
o
3 0 m 2 3 0 m 2 45
(c) (d) 
2d 3 8d 3 m 1
Ans: (d)
0 
Solution: U  m1  m2  3m 1  rˆ m 2  rˆ ,
4r 3


   
Since m1  m2  m1  m2  0  U  0 3  3  m cos 45 0  m cos 45 0
4d
 
3 0 m 2
U   .
8 d 3

H.No. 40-D, Ground Floor, Jia Sarai, Near IIT, Hauz Khas, New Delhi-110016
Phone: 011-26865455/+91-9871145498
Website: www.physicsbyfiziks.com | Email: fiziks.physics@gmail.com
1
fiziks
Institute for NET/JRF, GATE, IIT‐JAM, M.Sc. Entrance, JEST, TIFR and GRE in Physics

Statement for Linked Answer Questions 3 and 4:


Consider the propagation of electromagnetic waves in a linear, homogeneous and
isotropic material medium with electric permittivity ε and magnetic permeability μ.

Q3. For a plane wave of angular frequency ω and propagation vector k propagating in the
medium Maxwell’s equations reduce to

(a) k  E  0; k  H  0; k  E   H ; k  H   E

(b) k  E  0; k  H  0; k  E   H ; k  H   E

(c) k  E  0; k  H  0; k  E   H ; k  H   E

(d) k  E  0; k  H  0; k  E   H ; k  H   E
Ans: (d)
Q4. If  and  assume negative values in a certain frequency range, then the directions of
the propagation vector k and the Poynting vector S in that frequency range are related
as
(a) k and S are parallel

(b) k and S are anti-parallel

(c) k and S are perpendicular to each other

(d) k and S makes an angle that depends on the magnitude of |ε| and |μ|
Ans: (a)
Q5. Consider a conducting loop of radius a and total loop resistance R placed in a region with
a magnetic field B thereby enclosing a flux 0. The loop is connected to an electronic
circuit as shown, the capacitor being initially uncharged
 C


 

   
 Vout

 



If the loop is pulled out of the region of the magnetic field at a constant speed u, the final
output voltage Vout is independent of
(a) 0 (b) u (c) R (d) C
Ans: (a)

H.No. 40-D, Ground Floor, Jia Sarai, Near IIT, Hauz Khas, New Delhi-110016
Phone: 011-26865455/+91-9871145498
Website: www.physicsbyfiziks.com | Email: fiziks.physics@gmail.com
2
fiziks
Institute for NET/JRF, GATE, IIT‐JAM, M.Sc. Entrance, JEST, TIFR and GRE in Physics

GATE-2011

Q6. If a force F is derivable from a potential function V(r), where r is the distance from the
origin of the coordinate system, it follows that

(a)   F  0 (b)   F  0 (c)  V  0 (d)  2 V  0


Ans: (a)
Q7. Two charges q and 2q are placed along the x -axis in front of a grounded, infinite
conducting plane, as shown in the figure. They
are located respectively at a distance of 0.5 m and
1.5 m from the plane. The force acting on the
0 .5 m q 2q
charge q is   x
1 .5 m
7q 2
1 1 2
(a) (b) 2q
4 0 2 4 0

1 1 q2
(c) q2 (d)
4 0 4 0 2
Ans: (a)
Solution: Using method of Images we can draw equivalent figure as shown below:

 2q q 0 .5 m 0 .5 m q 2q
    x
1 .5 m 1 .5 m

q  2q q 2q  q 7q 1 7q 2
F      
4 0  12 12  2 2  4 0 2 4 0 2

Q8. A uniform surface current is flowing in the positive y-direction over an infinite sheet
lying in x-y plane. The direction of the magnetic field is
(a) along iˆ for z > 0 and along  iˆ for z < 0
(b) along k̂ for z > 0 and along  k̂ for z < 0
(c) along  iˆ for z > 0 and along iˆ for z < 0
(d) along  k̂ for z > 0 and along k̂ for z < 0
Ans: (a)

H.No. 40-D, Ground Floor, Jia Sarai, Near IIT, Hauz Khas, New Delhi-110016
Phone: 011-26865455/+91-9871145498
Website: www.physicsbyfiziks.com | Email: fiziks.physics@gmail.com
3
fiziks
Institute for NET/JRF, GATE, IIT‐JAM, M.Sc. Entrance, JEST, TIFR and GRE in Physics

Q9. A magnetic dipole of dipole moment m is placed in a non-uniform magnetic field B . If

the position vector of the dipole is r , the torque acting on the dipole about the origin is

(a) r  m  B   
(b) r   m  B

(c) m  B  
(d) m  B  r   m  B
Ans: (c)

Q10. A spherical conductor of radius a is placed in a uniform electric field E  E 0 kˆ . The


potential at a point P(r, θ) for r > a, is given by
E0 a 3
Φ(r, θ) = constant – E 0 r cos   2 cos 
r
where r is the distance of P from the centre O of the sphere and θ is the angle OP makes
with the z-axis P
The charge density on the sphere at θ = 30o is r
(a) 3 3 0 E 0 / 2 (b) 3 0 E 0 / 2 
O k̂
(c) 3 0 E 0 / 2 (d))  0 E 0 / 2
Ans: (a)

V  2E a 3 
Solution:    0   0  E 0 cos   03 cos   .
r r a  r  r a

3 3
   0  E 0 cos   2 E 0 cos      3E 0  0 cos   3E 0  0 cos 30 0   0 E0
2
Q11. Which of the following expressions for a vector potential A DOES NOT represent a
uniform magnetic field of magnitude B0 along the z-direction?
(a) A  0, B0 x,0 (b) A   B0 y,0,0 

 B0 x B0 y   B0 y B0 x 
(c) A   , ,0  (d) A    , ,0 
 2 2   2 2 
Ans: (c)
 
Solution: B    A .

H.No. 40-D, Ground Floor, Jia Sarai, Near IIT, Hauz Khas, New Delhi-110016
Phone: 011-26865455/+91-9871145498
Website: www.physicsbyfiziks.com | Email: fiziks.physics@gmail.com
4
fiziks
Institute for NET/JRF, GATE, IIT‐JAM, M.Sc. Entrance, JEST, TIFR and GRE in Physics

Statement for Linked Questions 12 and 13:


A plane electromagnetic wave has the magnetic field given by
 k 
B x, y, z , t   B0 sin x  y    t  kˆ
 2 

where k is the wave number and iˆ, ˆj and kˆ are the Cartesian unit vectors in x, y and z
directions respectively.

Q12. The electric field E  x, y, z , t  corresponding to the above wave is given by


(a) cB0 sin  x  y 
k  iˆ  ˆj
  t
  
(b) cB0 sin  x  y 
k
  t

 iˆ  ˆj 
 2  2  2  2
 k   k 
(c) cB0 sin  x  y    t  iˆ (d) cB0 sin  x  y    t  ˆj
 2   2 
Ans: (a)

 c   c  k iˆ  ˆj  
 x  y k  ˆ

Solution: E   k  B   
k
   B0 sin   t  k 
k

2  2  

 
E  cB0 sin  x  y 
k  iˆ  ˆj
 t 
 
 2  2
Q13. The average Poynting vector is given by

(a)

cB02 iˆ  ˆj  (b) 

cB02 iˆ  ˆj  (c)

cB02 iˆ  ˆj  (d) 

cB02 iˆ  ˆj 
2 0 2 2 0 2 2 0 2 2 0 2
Ans: (d)
 cB02 cB02  iˆ  ˆj  cB02  iˆ  ˆj 
Solution: S  ˆ
k    
2 0 2 0  2  2 0  2 

H.No. 40-D, Ground Floor, Jia Sarai, Near IIT, Hauz Khas, New Delhi-110016
Phone: 011-26865455/+91-9871145498
Website: www.physicsbyfiziks.com | Email: fiziks.physics@gmail.com
5
fiziks
Institute for NET/JRF, GATE, IIT‐JAM, M.Sc. Entrance, JEST, TIFR and GRE in Physics

GATE-2012
Q14. The space-time dependence of the electric field of a linearly polarized light in free space
ˆ 0 cos t  kz  where E0, ω and k are the amplitude, the angular frequency
is given by xE

and the wavevector, respectively. The time average energy density associated with the
electric field is
1 1
(a)  0 E 02 (b)  0 E 02 (c)  0 E 02 (d) 2 0 E 02
4 2
Ans: (a)
1 1 1
Solution: u E   0 E 2   0 E 2 cos 2 wt  kz   u E   0 E 02
2 2 4
Q15. A plane electromagnetic wave traveling in free space is incident normally on a glass plate
of refractive index 3/2. If there is no absorption by the glass, its reflectivity is
(a) 4% (b) 16% (c) 20% (d) 50%
Ans: (a)
2 2
 n  n2  1 3/ 2  1 4
Solution: R   1        .04 or 4%
 n1  n2  1 3/ 2  4 25

Q16. The electric and the magnetic field E  z , t  and B z , t  , respectively corresponding to the

scalar potential   z , t   0 and vector potential A z , t   iˆtz are
   
(a) E  iˆz and B  -ĵt (b) E  iˆz and B  ĵt
   
(c) E  iˆz and B  -ĵt (d) E  iˆz and B  -ĵt
Ans: (d)  
  A A   
Solution: E      iˆz , B    A   ˆjt .
t t
Q17. A plane polarized electromagnetic wave in free space at time t=0 is given
 
by E  x, z   10 ˆj exp i  6 x  8 z   . The magnetic field B x, z , t  is given by
 1
 
(a) B x, z , t   6kˆ  8iˆ expi 6 x  8 z  10ct 
c
 1
 
(b) B x, z , t   6kˆ  8iˆ expi 6 x  8 z  10ct 
c


1 ˆ ˆ

(c) B x, z , t   6k  8i expi 6 x  8 z  ct 
c
 1
 
(d) B x, z , t   6kˆ  8iˆ expi 6 x  8 z  ct 
c

H.No. 40-D, Ground Floor, Jia Sarai, Near IIT, Hauz Khas, New Delhi-110016
Phone: 011-26865455/+91-9871145498
Website: www.physicsbyfiziks.com | Email: fiziks.physics@gmail.com
6
fiziks
Institute for NET/JRF, GATE, IIT‐JAM, M.Sc. Entrance, JEST, TIFR and GRE in Physics

Ans: (a)
 1  k   1  6iˆ  8kˆ 
 1
c
ˆ  
Solution: B  k  E    E   
c k


  10 ˆjexp i k .r  t
 
 c  10 
 
 1
 
B  6kˆ  8iˆ expi 6 x  8 z  10ct ,   10c.
c
Q18. Two infinitely extended homogeneous isotopic dielectric media (medium-1and medium-2
1 
with dielectric constant  2 and 2  5 , respectively)
0 0
medium - 1
meet at the z = 0 plane as shown in the figure. A uniform
electric field exists everywhere. For z ≥ 0, the electric field
 medium - 2 z=0
is given by E1  2iˆ  3 ˆj  5kˆ . The interface separating the
two media is charge free. The electric displacement vector
in the medium-2 is given by


(a) D 2   0 10iˆ  15 ˆj  10kˆ  
(b) D 2   0 10iˆ  15 ˆj  10kˆ 
(c) D 2   4iˆ  6 ˆj  10kˆ 
0 (d) D 2   4iˆ  6 ˆj  10kˆ 
0

Ans: (b)
Solution:  E1  E 2  E 2  2iˆ  3 ˆj
1  2  5 ˆ 
and  f  0  D1  D2  E 2  E1  k  2kˆ  E 2  2iˆ  3 ˆj  2kˆ
2 5
 
 
 D2   2 E 2   0 10iˆ  15 ˆj  10kˆ .

GATE-2013
Q19. At a surface current, which one of the magnetostatic boundary condition is NOT
CORRECT?
(a) Normal component of the magnetic field is continuous.
(b) Normal component of the magnetic vector potential is continuous.
(c) Tangential component of the magnetic vector potential is continuous.
(d) Tangential component of the magnetic vector potential is not continuous.
Ans: (d)

H.No. 40-D, Ground Floor, Jia Sarai, Near IIT, Hauz Khas, New Delhi-110016
Phone: 011-26865455/+91-9871145498
Website: www.physicsbyfiziks.com | Email: fiziks.physics@gmail.com
7
fiziks
Institute for NET/JRF, GATE, IIT‐JAM, M.Sc. Entrance, JEST, TIFR and GRE in Physics

Q20. Interference fringes are seen at an observation plane z  0 , by the superposition of two
 
   
 
plane waves A1 exp i k1  r  t and A2 exp i k 2  r  t , where A1 and A2 are real
amplitudes. The condition for interference maximum is
     
 
(a) k1  k 2  r  2m  1 
(b) k1  k 2  r  2m 
     
 
(c) k1  k 2  r  2m  1 
(d) k1  k 2  r  2m 
Ans: (b)
Q21. For a scalar function  satisfying the Laplace equation,  has
(a) zero curl and non-zero divergence
(b) non-zero curl and zero divergence
(c) zero curl and zero divergence
(d) non-zero curl and non-zero divergence
Ans: (c)

 
Solution:  2  0  .   0 and      0 .  
Q22. A circularly polarized monochromatic plane wave is incident on a dielectric interface at
Brewaster angle. Which one of the following statements is correct?
(a) The reflected light is plane polarized in the plane of incidence and the transmitted
light is circularly polarized.
(b) The reflected light is plane polarized perpendicular to the plane of incidence and the
transmitted light is plane polarized in the plane of incidence.
(c) The reflected light is plane polarized perpendicular to the plane of incidence and the
transmitted light is elliptically polarized.
(d) There will be no reflected light and the transmitted light is circularly polarized.
Ans: (c)
Q23. A charge distribution has the charge density given by   Q x  x 0     x  x0  . For

this charge distribution the electric field at 2 x0 ,0,0

2Qxˆ Qxˆ Qxˆ Qxˆ


(a) (b) (c) (d)
9 0 x02 4 0 x03 4 0 x02 16 0 x02
Ans:

H.No. 40-D, Ground Floor, Jia Sarai, Near IIT, Hauz Khas, New Delhi-110016
Phone: 011-26865455/+91-9871145498
Website: www.physicsbyfiziks.com | Email: fiziks.physics@gmail.com
8
fiziks
Institute for NET/JRF, GATE, IIT‐JAM, M.Sc. Entrance, JEST, TIFR and GRE in Physics

Solution: Potential V  r  
1   x

a '

dx  
 
a  x'
 
a  x'  
       
2
x dx x dx ....
4 0   a x x 2
x 3

 a a

First term, total charge


x0 x0

QT     x dx  Q    x   x 0 dx   Q    x   x 0 dx   Q  Q  0


 x0  x0

Second term, dipole moment


x0 x0

p   x   x dx  Q  x   x   x 0 dx   Q  x  x   x 0 dx   Qx 0  Q   x 0  2Qx 0


 x0  x0

2Qx 0 V 4Qx 0 4Qx 0 Q


V E xˆ  xˆ  xˆ  xˆ
4 0 x x 4 0 x 4 0 2 x 0  8 0 x 20
2 3 3

Q24. A monochromatic plane wave at oblique incidence undergoes reflection at a dielectric


interface. If kˆi , kˆr and n̂ are the unit vectors in the directions of incident wave, reflected
wave and the normal to the surface respectively, which one of the following expressions
is correct?
 
(a) kˆi  kˆr  nˆ  0  
(b) kˆi  kˆr  nˆ  0  
(c) kˆi  nˆ  kˆr  0  
(d) kˆi  nˆ  kˆr  0
Ans: (c)
Q25. In a constant magnetic field of 0.6 Tesla along the z direction, find the value of the path

integral  A  dl in the units of (Tesla m 2 ) on a square loop of side length 1 / 2 meters.  
The normal to the loop makes an angle of 60 0 to the z-axis, as shown in the figure. The
answer should be up to two decimal places. ___________

60 o

Ans: 0.15

H.No. 40-D, Ground Floor, Jia Sarai, Near IIT, Hauz Khas, New Delhi-110016
Phone: 011-26865455/+91-9871145498
Website: www.physicsbyfiziks.com | Email: fiziks.physics@gmail.com
9
fiziks
Institute for NET/JRF, GATE, IIT‐JAM, M.Sc. Entrance, JEST, TIFR and GRE in Physics

 
2
  1  1
Solution:  A  dl     A .d a   B.d a  BA cos 60  0.6  
0
   0.15T .m 2
S S  2 2

GATE-2014
Q26. Which one of the following quantities is invariant under Lorentz transformation?
(a) Charge density (b) Charge (c) Current (d) Electric field
Ans: (b)
Q27. An unpolarized light wave is incident from air on a glass surface at the Brewster angle.
The angle between the reflected and the refracted wave is
(a) 0 o (b) 45 o (c) 90 o (d) 120 o
Ans: (c)
Q28. The electric field of a uniform plane wave propagating in a dielectric non-conducting

medium is given by E  xˆ 10 cos  6 107 t  0.4 z  V / m . The phase velocity of the

wave is _________ 10 8 m / s
Ans: 1.5
 6 107
Solution: v    1.5 108 m / sec
k 0.4

Q29. If the vector potential A  xxˆ  2 yyˆ  3zzˆ , satisfies the Coulomb gauge, the value of the
constant  is _______
Ans: 1
 
Solution: Coulomb gauge condition . A  0    2  3  0    1
Q30. A ray of light inside Region 1 in the xy -plane is incident y
at the semicircular boundary that carries no free charges.
  P r0 ,  / 4 
The electric field at the point P  r0 ,  in plane polar
 4
 O
coordinates is E1  7eˆr  3eˆ where êr and ê are the unit 1 2 x
Region 1 Region 2
vectors. The emerging ray in Region 2 has the electric

field E 2 parallel to x -axis. If  1 and  2 are the dielectric
2
constants of Region-1 and Region-2 respectively, then is ________
1

H.No. 40-D, Ground Floor, Jia Sarai, Near IIT, Hauz Khas, New Delhi-110016
Phone: 011-26865455/+91-9871145498
Website: www.physicsbyfiziks.com | Email: fiziks.physics@gmail.com
10
fiziks
Institute for NET/JRF, GATE, IIT‐JAM, M.Sc. Entrance, JEST, TIFR and GRE in Physics

Ans: 2.32
 y
Solution:  E1  7eˆr  3eˆ P r0 ,  / 4 

10 2 E 2
 Ex   7eˆr  3eˆ  .xˆ  7 cos 45  3sin 45  1 
2 E1

O
4 1 x
 E y   7eˆr  3eˆ  . yˆ  7 sin 45  3sin 45  2
2 Region 1 Region 2

 E   4
Thus E1 makes an angle   tan 1  y   tan 1    21.80
 Ex   10 
tan  2  2  tan 45
   2   2.32 . where 1    450 and  2  450
tan 1 1 1 tan 23.2
Q31. The value of the magnetic field required to maintain non-relativistic protons of energy
1MeV in a circular orbit of radius 100 mm is _______Tesla
(Given: m p  1.67  1027 kg , e  1.6  1019 C )

Ans: 1.44

1.6 1019  B 2  0.1 1.6 1013  2 1.67 1027 


2 2
q2 B2 R2 13
Solution: E   1.6 10  B 
2

2 1.67 1027  1.6 1019   0.12


2
2m p

1013  2 1.67 1027  3.34 1040


B  2
  2.08  B  2.08 Tesla  1.44Tesla
1.6 10   0.01
38
1.6 1040

Q32. In an interference pattern formed by two coherent sources, the maximum and minimum
intensities are 9I 0 and I 0 respectively. The intensities of the individual wave are

(a) 3I 0 and I 0 (b) 4 I 0 and I 0 (c) 5I 0 and 4 I 0 (d) 9 I 0 and I 0


Ans: (b)

   
2 2
Solution: I max  I1  I 2 and I min  I1  I 2

   
2 2
9I 0  I1  I 2 and I 0  I1  I 2  I1  4 I 0 and I 2  I 0

Q33. The intensity of a laser in free space is 150mW / m 2 . The corresponding amplitude of the

electric field of the laser is _________


V
m
 0  8.854  10 12 C 2 / N .m 2 

H.No. 40-D, Ground Floor, Jia Sarai, Near IIT, Hauz Khas, New Delhi-110016
Phone: 011-26865455/+91-9871145498
Website: www.physicsbyfiziks.com | Email: fiziks.physics@gmail.com
11
fiziks
Institute for NET/JRF, GATE, IIT‐JAM, M.Sc. Entrance, JEST, TIFR and GRE in Physics

Ans: 10.6

1 2I 2 150 103
Solution: I  c 0 E02  E0    10.6 V / m
2 c 0 3 108  8.854 1012

GATE-2015
Q34. A point charge is placed between two semi-infinite conducting plates which are inclined
at an angle of 30 o with respect to each other. The number of image charges
is___________.
Ans.: 11
360 360
Solution: n  1   1  11
 30
R  
Q35. Given that the magnetic flux through the closed loop PQRSP is  . If  A  dl   along
P
1

 
R
PQR , the value of   dl along PSR is
A
P Q
R

S
(a)   1 (b) 1   (c)  1 (d) 1
Ans.: (b)
    R   P  R   R  
Solution:    s B.d a   A.dl   A  dl   A  dl    1   A  dl   A  dl  1  
P R P P

Q36. The space between two plates of a capacitor carrying charges  Q and  Q is filled with
two different dielectric materials, as shown in the figure. Across the interface of the two
dielectric materials, which one of the following statements is correct?
 
(a) E and D are continuous
 
(b) E is continuous and D is discontinuous
 
(c) D is continuous and E is discontinuous Q Q
 
(d) E and D are discontinuous
Ans.: (d)

H.No. 40-D, Ground Floor, Jia Sarai, Near IIT, Hauz Khas, New Delhi-110016
Phone: 011-26865455/+91-9871145498
Website: www.physicsbyfiziks.com | Email: fiziks.physics@gmail.com
12
fiziks
Institute for NET/JRF, GATE, IIT‐JAM, M.Sc. Entrance, JEST, TIFR and GRE in Physics

Q37. Four forces are given below in Cartesian and spherical polar coordinates
   r2  
(i) F1  K exp 2 rˆ 
(ii) F2  K x 3 yˆ  y 3 zˆ 
 R 
   ˆ 
(iii) F3  K x 3 xˆ  y 3 yˆ  (iv) F4  K  
r
where K is a constant Identify the correct option
(a) (iii) and (iv) are conservative but (i) and (ii)are not
(b) (i) and (ii) are conservative but (iii) and (iv) are not
(c) (ii) and (iii) are conservative but (i) and (iv) are not
(d) (i) and (iii) are conservative but (ii) and (iv) are not
Ans.: (d)

r rˆ r sin 


  1   
Solution:   F 1  0
r sin 
2
r  
 r2 
k exp   2  0 0
 R 

x y z
    
 F2   x  3ky 2  0   zˆ  3kx 2  0   3ky 2 x  3kx 2 zˆ
x y z
0 kx 3  ky 3

x y z
    
 F3  0
x y z
kx3 ky 3 0

r r r sin 


  1    1
 F4  2  r  k cos    2
r sin  r   r sin 
k
0 0 r sin  
r

H.No. 40-D, Ground Floor, Jia Sarai, Near IIT, Hauz Khas, New Delhi-110016
Phone: 011-26865455/+91-9871145498
Website: www.physicsbyfiziks.com | Email: fiziks.physics@gmail.com
13
fiziks
Institute for NET/JRF, GATE, IIT‐JAM, M.Sc. Entrance, JEST, TIFR and GRE in Physics

Q38. A monochromatic plane wave (wavelength  600 nm ) E 0 expi kz   t  is incident


 

normally on a diffraction grating giving rise to a plane wave E1 exp i k1  r   t in the 
  1 3 
first order of diffraction. Here E1  E 0 and k1  k1  xˆ  zˆ  . The period (in m ) of
2 2 
the diffraction grating is ______________ (upto one decimal place)
Ans.: 1.2
grating

Solution: d sin   n  d  n 1
sin  
  1 3  k1
and k1  k1  xˆ  zˆ 
 2 2   ẑ
k  k1 zˆ
1 3 
  zˆ   xˆ  zˆ 
k  k1  2 2  1
 sin          300
k1 k 1 3 1 3 2
  
4 4 4 4
600
d  nm  1200 nm  1.2  m
sin 30
Q39. A long solenoid is embedded in a conducting medium and is insulated from the medium.
If the current through the solenoid is increased at a constant rate, the induced current in
the medium as a function of the radial distance r from the axis of the solenoid is
proportional to
1 1
(a) r 2 inside the solenoid and outside (b) r inside the solenoid and outside
r r2
1 1
(c) r 2 inside the solenoid and 2 outside (d) r inside the solenoid and outside
r r
Ans.: (d)

  B 
Solution:  E  dl     da ;
t
 dI
r
dI 2 r 2  1 dI
For r  R, E 2 r   0 n
dt r0 2 r dr     0 n
dt 2
 E   0 n r
2 dt

 dI
R
dI 2 R 2  1 dI
For r  R, E 2 r   0 n
dt 
r 0
2 r dr    0 n
dt 2
 E   0 n R 2
2r dt

H.No. 40-D, Ground Floor, Jia Sarai, Near IIT, Hauz Khas, New Delhi-110016
Phone: 011-26865455/+91-9871145498
Website: www.physicsbyfiziks.com | Email: fiziks.physics@gmail.com
14
fiziks
Institute for NET/JRF, GATE, IIT‐JAM, M.Sc. Entrance, JEST, TIFR and GRE in Physics

Q40. A plane wave xˆ  iyˆ E 0 expikz  t  after passing through an optical element emerges

as xˆ  iyˆ E 0 expi kz  t  , where k and  are the wavevector and the angular
frequency, respectively. The optical element is a
(a) quarter wave plate (b) half wave plate
(c) polarizer (d) Faraday rotator
Ans.: (b)

 
Solution: Incident wave: x  i y E0 ei   E0 cos  x  E0 sin  y 
 
Left circular polarization with phase angle 1     ei

 
Emergent wave: x  i y E0 ei   E0 cos  xˆ  E0 sin  y 
 
Right circular polarization with phase angle 1     ei 0


Thus there is phase change of  and hence path difference is .
2
Q41. A charge  q is distributed uniformly over a sphere, with a positive charge q at its center
in (i). Also in (ii), a charge  q is distributed uniformly over an ellipsoid with a positive
charge q at its center. With respect to the origin of the coordinate system, which one of
the following statements is correct?

X X

Z Z

Y Y
(i ) (ii )
(a) The dipole moment is zero in both (i) and (ii)
(b) The dipole moment is non-zero in (i) but zero in (ii)
(c) The dipole moment is zero in (i) but non-zero in (ii)
(d) The dipole moment is non-zero in both (i) and (ii)
Ans.: (a)
 
Solution: p   qi ri  0 in both cases.

H.No. 40-D, Ground Floor, Jia Sarai, Near IIT, Hauz Khas, New Delhi-110016
Phone: 011-26865455/+91-9871145498
Website: www.physicsbyfiziks.com | Email: fiziks.physics@gmail.com
15
fiziks
Institute for NET/JRF, GATE, IIT‐JAM, M.Sc. Entrance, JEST, TIFR and GRE in Physics

GATE-2016
Q42. Which of the following magnetic vector potentials gives rise to a uniform magnetic field
B0 kˆ ?

(a) B0 z kˆ (b)  B0 x ˆj (c)


B0
2

 yiˆ  xˆj  (d)
2

B0 ˆ ˆ
yi  xj 
Ans.: (c)
 
Solution: (a)   A  0
 
(b)   A   B0 kˆ
 
(c)   A  B0 kˆ
 
(d)   A  0
Q43. The magnitude of the magnetic dipole moment associated with a square shaped loop
carrying a steady current I is m . If this loop is changed to a circular shape with the same
pm
current I passing through it, the magnetic dipole moment becomes . The value of p

is ______.
Ans.: 4
Solution: Magnetic dipole moment associated with a square shaped loop (let side is a) carrying a
steady current I is m  Ia 2 .
Magnetic dipole moment associated with a circular shaped loop (let radius is r) carrying a
steady current I is m  I  r 2 .
2
2a  2a  4 Ia 2 4m
Here 4a  2 r  r   m  I  r  I    
2

    
Q44. In a Young’s double slit experiment using light, the apparatus has two slits of unequal
widths. When only slit- 1 is open, the maximum observed intensity on the screen is 4I 0 .

When only slit- 2 is open, the maximum observed intensity is I 0 . When both the slits are
open, an interference pattern appears on the screen. The ratio of the intensity of the
principal maximum to that of the nearest minimum is ________.
Ans.: 9

H.No. 40-D, Ground Floor, Jia Sarai, Near IIT, Hauz Khas, New Delhi-110016
Phone: 011-26865455/+91-9871145498
Website: www.physicsbyfiziks.com | Email: fiziks.physics@gmail.com
16
fiziks
Institute for NET/JRF, GATE, IIT‐JAM, M.Sc. Entrance, JEST, TIFR and GRE in Physics

     2 
2 2 2
I max I1  I 2 4I0  I0 I0  I0 9I0
Solution:   9
    2 
2 2 2
I min I1  I 2 4I0  I0 I0  I0 I0

Q45. An infinite, conducting slab kept in a horizontal plane carries a uniform charge density  .
Another infinite slab of thickness t, made of a linear dielectric material of dielectric
constant k , is kept above the conducting slab. The bound charge density on the upper
surface of the dielectric slab is
   k  2  k  1
(a) (b) (c) (d)
2k k 2k k
Ans.: (d)  1
k  1 z
Solution: 
  
Electric field due to infinite, conducting slab inside the dielectric is E  zˆ  zˆ
  0k
     k  1    k  1
Polarisation P   0  e E   0  k  1 zˆ  zˆ   1  P.zˆ 
 0k k k
Q46. The electric field component of a plane electromagnetic wave travelling in vacuum is

given by E  z , t   E 0 coskz  t iˆ . The Poynting vector for the wave is

 c   c 
(a)  0  E 02 cos 2 kz  t  ˆj (b)  0  E 02 cos 2 kz  t kˆ
 2   2 

(c) c 0 E 02 cos 2 kz  t  ˆj (d) c 0 E 02 cos 2 kz  t kˆ


Ans.: (d)
  1  E
Solution: E  z , t   E 0 coskz  t iˆ  B  zˆ  E  z , t   0 cos  kz  t  ˆj
c c
The Poynting vector for the wave is
 1   E2
S
0
 0 c

E  B  0 cos 2  kz  t  kˆ  c 0 E02 cos 2  kz  t  kˆ

Q47. The x  y plane is the boundary between free space and a magnetic material with relative

permeability  r . The magnetic field in the free space is Bx iˆ  Bz kˆ . The magnetic field in
the magnetic material is
1
(a) B x iˆ  B z kˆ (b) B x iˆ   r B z kˆ (c) B x iˆ  B z kˆ (d)  r B x iˆ  B z kˆ
r

H.No. 40-D, Ground Floor, Jia Sarai, Near IIT, Hauz Khas, New Delhi-110016
Phone: 011-26865455/+91-9871145498
Website: www.physicsbyfiziks.com | Email: fiziks.physics@gmail.com
17
fiziks
Institute for NET/JRF, GATE, IIT‐JAM, M.Sc. Entrance, JEST, TIFR and GRE in Physics

Ans.: (d)
B B
Solution: B1  Bz kˆ  B2 and H1  H 2  1  2  B2  r B1  r Bx iˆ
0 0 r

The magnetic field in the magnetic material is  r B x iˆ  B z kˆ

GATE- 2017
Q48. Identical charges q are placed at five vertices of a regular hexagon of side a . The
magnitude of the electric field and the electrostatic potential at the centre of the hexagon
are respectively
q q
(a) 0, 0 (b) ,
4 0 a 2
4 0 a
q 5q 5q 5q
(c) , (d) ,
4 0 a 2
4 0 a 4 0 a 4 0 a
2

Ans. : (c) q
q
Solution: The resultant field at P is E  q q
4 0 a 2
a
5q P
The electrostatic potential at P is V  q q
4 0 a
Q49. A parallel plate capacitor with square plates of side 1 m separated by 1 micro meter is
filled with a medium of dielectric constant of 10 . If the charges on the two plates are 1C
and 1C , the voltage across the capacitor is………….. kV . (up to two decimal places).

(  0  8.854 1012 F / m )
Ans. : 11.29
 0 r A 11 106
qd
Solution: q  CV  V V    11.29kV
d  0 r A 8.854 1012 10 1
Q50. Light is incident from a medium of refractive index n  1.5 onto vacuum. The smallest
angle of incidence for which the light is not transmitted into vacuum is…………...
degrees. (up to two decimal places)
Ans. : 41.8
n2 1  1 
Solution: sin C    C  sin 1    C  41.8
n1 1.5  1.5 

H.No. 40-D, Ground Floor, Jia Sarai, Near IIT, Hauz Khas, New Delhi-110016
Phone: 011-26865455/+91-9871145498
Website: www.physicsbyfiziks.com | Email: fiziks.physics@gmail.com
18
fiziks
Institute for NET/JRF, GATE, IIT‐JAM, M.Sc. Entrance, JEST, TIFR and GRE in Physics

Q51. A monochromatic plane wave in free space with electric field amplitude of 1 V / m is
normally incident on a fully reflecting mirror. The pressure exerted on the mirror
is……………… 1012 Pa . (up to two decimal places) (  0  8.854 1012 F / m )
Ans. : 8.85
2I 2 1
  c 0 E02   0 E02  8.854 1012  1  8.85 1012 Pa
2
Solution: P 
c c 2
Q52. Three charges  2C , 1C , 1C  are placed at the vertices of an equilateral triangle of side

1m as shown in the figure. The component of the electric dipole moment about the
marked origin along the ŷ direction is……… C m .
y
2C

1m

0 1C 1C x
1.5m
Ans. : 1.73


Solution: p  11xˆ   1 2 xˆ   2 1.5 xˆ  1  0.25 yˆ 
Along the ŷ direction  2  1  0.25  1.73

Q53. An infinite solenoid carries a time varying current I  t   At 2 , with A  0 . The axis of

the solenoid is along the ẑ direction. r̂ and ˆ are the usual radial and polar directions in

cylindrical polar coordinates. B  Br rˆ  B ˆ  Bz zˆ is the magnetic field at a point outside
the solenoid. Which one of the following statements is true?
(a) Br  0, B  0, Bz  0 (b) Br  0, B  0, Bz  0

(c) Br  0, B  0, Bz  0 (d) Br  0, B  0, Bz  0
Ans. : (d)
Q54. A uniform volume charge density is placed inside a conductor (with resistivity102 m ).
1
The charge density becomes of its original value after time…….Fermi seconds
 2.718
(up to two decimal places) (  0  8.854 1012 F / m )
Ans. : 88.54

H.No. 40-D, Ground Floor, Jia Sarai, Near IIT, Hauz Khas, New Delhi-110016
Phone: 011-26865455/+91-9871145498
Website: www.physicsbyfiziks.com | Email: fiziks.physics@gmail.com
19
fiziks
Institute for NET/JRF, GATE, IIT‐JAM, M.Sc. Entrance, JEST, TIFR and GRE in Physics

 t  1
Solution:   t     0  e  t /  0   t /  0  ln  ln 1
 0 2.718

0
t   8.854 1012 102  88.54 1015 sec  88.54 fs

Q55. Consider a metal with free electron density of 6 1022 cm 3 . The lowest frequency of

electromagnetic radiation to which this metal is transparent, is 1.38 1016 Hz . If this

metal had a free electron density of 1.8  1023 cm 3 instead, the lowest frequency

electromagnetic radiation to which it would be transparent is…………… 1016 Hz (up to


two decimal places).
Ans. : 2.39
Solution: Cut-off frequency is f  n .

f2 n n 1.8  1023
Thus  2  f 2  f1 2  f 2  1.38 1016  2.39 1016 Hz
f1 n1 n1 6  1022

GATE- 2018

  
Q56. Among electric field ( E ), magnetic field ( B ), angular momentum ( L ) and vector

potential ( A ), which is/are odd under parity (space inversion) operation?
  
(a) E only (b) E and A only
   
(c) E and B only (d) B and L only
Ans. : (b)
Solution: Under parity operation r  r
V
E ; E : P  E
r
 
B  I r ; B : P  B
 
Lrp ; L : P  L
A
E ; A: P  A
t

H.No. 40-D, Ground Floor, Jia Sarai, Near IIT, Hauz Khas, New Delhi-110016
Phone: 011-26865455/+91-9871145498
Website: www.physicsbyfiziks.com | Email: fiziks.physics@gmail.com
20
fiziks
Institute for NET/JRF, GATE, IIT‐JAM, M.Sc. Entrance, JEST, TIFR and GRE in Physics

Q57. An infinitely long straight wire is carrying a steady current I . The ratio of magnetic
energy density at distance r1 to that at r2   2r1  from the wire is ___________.

Ans. : 4
B2 1 u r 2  2r 
Solution: uB   2  B1  22  21  4
2 0 r uB 2 r1 r1

Q58. A light beam of intensity I 0 is falling normally on a surface. The surface absorbs 20 %
of the intensity and the rest is reflected. The radiation pressure on the surface is given by
X I 0 / c , where X is __________ (up to one decimal place). Here c is the speed of light.

Ans. : 1.8
I0  I  I
Solution: Radiation pressure    0.8 0   1.8 0
c  c c
Q59. The number of independent components of a general electromagnetic field tensor
is__________
Ans. : 6
Solution: In Cartesian co-ordinate, three Independent coordinate for electric field,  Ex , E y , Ez 

and three Independent co-ordinate for magnetic field  Bx , By , Bz  .

Q60. Consider an infinitely long solenoid with N turns per unit length, radius R and carrying
a current I  t    cos t , where  is a constant and  is the angular frequency. The

magnitude of electric field at the surface of the solenoid is


1 1
(a) 0 NR sin t (b) 0 NR cos t
2 2
(c) 0 NR sin t (d) 0 NR cos t
Ans. : (a)
   NI  t  zˆ, inside
Solution: B   0
0 , outside

  B 
Since,  E  dl     da
line
t

 E  2 R   0 N   sin t    R 2

H.No. 40-D, Ground Floor, Jia Sarai, Near IIT, Hauz Khas, New Delhi-110016
Phone: 011-26865455/+91-9871145498
Website: www.physicsbyfiziks.com | Email: fiziks.physics@gmail.com
21
fiziks
Institute for NET/JRF, GATE, IIT‐JAM, M.Sc. Entrance, JEST, TIFR and GRE in Physics

 1
 E  0 NR sin t
2

Q61. A constant and uniform magnetic field B  B0 kˆ pervades all space. Which one of the
following is the correct choice for the vector potential in Coulomb gauge?

(a)  B0  x  y  iˆ (b) B0  x  y  ˆj (c) B0 xjˆ


1

(d)  B0 xiˆ  yjˆ
2

Ans. : (c)
Solution: Check option (c),
    
  A  0, B    A  B0 kˆ

Q62. A long straight wire, having radius a and resistance per unit length r , carries a current
I . The magnitude and direction of the Poynting vector on the surface of the wire is
(a) I 2 r / 2 a , perpendicular to axis of the wire and pointing inwards
(b) I 2 r / 2 a , perpendicular to axis of the wire and pointing outwards
(c) I 2 r /  a , perpendicular to axis of the wire and pointing inwards
(d) I 2 r /  a , perpendicular to axis of the wire and pointing outwards
Ans. : (a)
  
Solution: S 
1
0
 E  B   1 Vl  2 Ia  IRl  2I a
0

R  I 2r
V  IR, r   S 
l 2 a
Q63. A quarter wave plate introduces a path difference of  / 4 between the two components
of polarization parallel and perpendicular to the optic axis. An electromagnetic wave with

E   xˆ  yˆ  E0 ei kz t  is incident normally on a quarter wave plate which has its optic axis
y
making an angle 1350 with the x - axis as shown.
The emergent electromagnetic wave would be
Optic axis 1350
(a) elliptically polarized x
(b) circularly polarized
(c) linearly polarized with polarization as that of incident wave
(d) linearly polarized but with polarization at 900 to that of the incident wave
Ans. : (c)
H.No. 40-D, Ground Floor, Jia Sarai, Near IIT, Hauz Khas, New Delhi-110016
Phone: 011-26865455/+91-9871145498
Website: www.physicsbyfiziks.com | Email: fiziks.physics@gmail.com
22
fiziks
Institute for NET/JRF, GATE, IIT‐JAM, M.Sc. Entrance, JEST, TIFR and GRE in Physics

Q64. An electromagnetic plane wave is propagating with an intensity I  1.0 105 Wm 2 in a


medium with  3 0 and   0 . The amplitude of the electric field inside the medium

is _________ 103 Vm 1 (up to one decimal place).

( 0  8.85 1012 C 2 N 1m 2 , 0  4 107 NA2 , c  3 108 ms 1 )

Ans. : 6.6

1 2I 2I 
Solution: I  v  E2  E2    2I
2 v 1 



0 4  107
 E 2  2  105  2  105  4363.4  104
3 0 3  8.8  1012

 E  66 102  6.6 103 V / m

GATE-2019

Q65. The electric field of an electromagnetic wave is given by E  3sin  kz   t  xˆ 

4 cos  kz  t  yˆ . The wave is

4
(a) linearly polarized at an angle tan 1   from the x - axis
3
3
(b) linearly polarized at an angle tan 1   from the x - axis
4
(c) elliptically polarized in clockwise direction when seen travelling towards the observer
(d) elliptically polarized in counter-clockwise direction when seen travelling towards the
observer
Ans. : (d)
y
Solution: At z  0, Ex  3sin t , E y  4 cos t

At t  0, Ex  0, E y  4
x

At  t  , Ex  3, E y  0
2

H.No. 40-D, Ground Floor, Jia Sarai, Near IIT, Hauz Khas, New Delhi-110016
Phone: 011-26865455/+91-9871145498
Website: www.physicsbyfiziks.com | Email: fiziks.physics@gmail.com
23
fiziks
Institute for NET/JRF, GATE, IIT‐JAM, M.Sc. Entrance, JEST, TIFR and GRE in Physics

Q66. An infinitely long thin cylindrical shell has its axis coinciding with the z -axis. It carries a
surface charge density  0 cos  , where  is the polar and  0 is a constant. The
magnitude of the electric field inside the cylinder is
0 0 0
(a) 0 (b) (c) (d)
2 0 3 0 4 0
Ans. : (b)

Solution: dE 
d  cos   Rd    0 cos 
 0
2 0 R 2 0 R 2 0
2
0 0
Along axis of cylinder dEx  dE cos   Ex   cos  d 
2

2 0 0
2 0

Q67. A circular loop made of a thin wire has radius 2 cm and resistance 2  . It is placed

perpendicular to a uniform magnetic field of magnitude B0  0.01 Tesla. At time t  0
 
the field starts decaying as B  B0 e  t / t0 , where t0  1s . The total charge that passes

through a cross section of the wire during the decay is Q . The value of Q in  C
(rounded off to two decimal places) is____________
Ans. : 6.28
d AdB  d 1
Solution:     , I  
dt dt R dt R
d d

dt
  r 2
dt
 
B0 e  t / t0   r 2 B0 e t  t0  1


 
 r2  r 2 B0 e t
Q   I  t  dt   t
B0 e dt 
0 0
R R 1 0

 3.14   2  102   0.01  6.28C


2

Q68. The electric field of an electromagnetic wave in vacuum is given by




E  E0 cos 3 y  4 z  1.5 109 t xˆ 
The wave is reflected from the z  0 surface. If the pressure exerted on the surface is
  E02 , the value of  (rounded off to one decimal place) is___________
Ans. : 0.8

H.No. 40-D, Ground Floor, Jia Sarai, Near IIT, Hauz Khas, New Delhi-110016
Phone: 011-26865455/+91-9871145498
Website: www.physicsbyfiziks.com | Email: fiziks.physics@gmail.com
24
fiziks
Institute for NET/JRF, GATE, IIT‐JAM, M.Sc. Entrance, JEST, TIFR and GRE in Physics

Solution: K  3 yˆ  4 zˆ
Ky 3
tan  R  
Kz 4
I 2 1 4
P  2 cos  R   0 cE02 
c c 2 5
P  0.80 E02

Q69. A solid cylinder of radius R has total charge Q distributed uniformly over its volume. It
is rotating about its axis with angular speed  . The magnitude of the total magnetic
moment of the cylinder is
1 1 1
(a) QR 2 (b) QR 2 (c) QR 2 (d) QR 2
2 4 8
Ans. : (c)
 R 4
Solution: Magnetic moment due to disc  
4
 R 4
Due to cylinder d     dz     dz 
4
 R 4 L
Q Q R 4

4 0  R 2 L dz 
4

Q70. An infinitely long wire parallel to the x -axis is kept at z  d and carries a current I in
the positive x direction above a superconductor filling the region z  0 (see figure). The

magnetic field B inside the superconductor is zero so that the field just outside the
superconductor is parallel to its surface. The magnetic field due to this configuration at a
point  x, y, z  0  is ẑ

  I    z  d  ˆj  ykˆ
(a)  0 
 2   y 2   z  d  
2

  l
  I     z  d  ˆj  ykˆ  z  d  ˆj  ykˆ  d
(b)  0   2  2 
 2   y   z  d  y   z  d  
2 2

superconductor
 I     z  d  ˆj  ykˆ  z  d  ˆj  ykˆ 
(c)  0  2  2 
 2   y   z  d  y   z  d  
2 2

H.No. 40-D, Ground Floor, Jia Sarai, Near IIT, Hauz Khas, New Delhi-110016
Phone: 011-26865455/+91-9871145498
Website: www.physicsbyfiziks.com | Email: fiziks.physics@gmail.com
25
fiziks
Institute for NET/JRF, GATE, IIT‐JAM, M.Sc. Entrance, JEST, TIFR and GRE in Physics

 I   yjˆ   z  d  kˆ yjˆ   z  d  kˆ 
(d)  0  2  2 
 2   y   z  d  y   z  d  
2 2

Ans. : (b)

Solution: Verify that B  0 , when d  0
Q71. The vector potential inside a long solenoid with n turns per unit length and carrying
  nI
current I , written in cylindrical coordinates is A  s,  , z   0 sˆ . If the term
2
0 nI 
2
 
s  cos  ˆ   sin  sˆ , where   0,   0 is added to A  S ,  , z  , the magnetic

field remains the same if



(a)    (b)     (c)   2  (d)  
2
  t ˆ 1 t ˆ t 
 Useful formulae:  t S   zˆ; 
S S  z
 
  v      sv  v  
  v   1 vz    sˆ   vs  vz ˆ  1   s  zˆ  
  s  z   z s s  s   
    
Ans. : (d)

rˆ rˆ zˆ
   1   
Solution: B    A   0 nIzˆ
r r  z
Ar rA 0

rˆ rˆ zˆ
   1      cos  
B    A   0 nI  cos   1  zˆ
r r  z  2 
Ar rA 0

    cos  
Equate B  B   cos   1   0 nI
 2 
 
  cos   cos    
2 2

H.No. 40-D, Ground Floor, Jia Sarai, Near IIT, Hauz Khas, New Delhi-110016
Phone: 011-26865455/+91-9871145498
Website: www.physicsbyfiziks.com | Email: fiziks.physics@gmail.com
26
fiziks
Institute for NET/JRF, GATE, IIT‐JAM, M.Sc. Entrance, JEST, TIFR and GRE in Physics

 P 
Q72. For a given load resistance RL  4.7 ohm, the power transfer efficiencies   load  of
 Ptotal 

a dc voltage source and a dc current source with internal resistances R1 and R2 ,

respectively, are equal. The product R1 R2 in units of ohm2 (rounded off to one decimal
place) is___________
Ans. : 22.09
Solution: For dc voltage source
R1
2
V 2
 V 
Ptotal  and PRL    RL
R1  RL R
 1  RL 
V RL
PRL RL
 dc vol  
Ptotal R1  RL
For dc current source
2
 RR   R2 I 
Ptotal  I  2 L  and PRL  I L2 RL  
2
 RL
 R2  RL   R2  RL 
PRL R2
 dc curr   I R2 RL
Ptotal R2  RL

Since  dc vol  dc curr

RL R2
   RL  R2  RL   R2  R1  RL   R1 R2  RL2
R1  RL R2  RL

 R1 R2   4.7   22.09  2
2

Q73. Consider a system of three charges as shown in the figure below:


z
 r , 
q

q d  q
 
2 2
y
d d

H.No. 40-D, Ground Floor, Jia Sarai, Near IIT, Hauz Khas, New Delhi-110016
Phone: 011-26865455/+91-9871145498
Website: www.physicsbyfiziks.com | Email: fiziks.physics@gmail.com
27
fiziks
Institute for NET/JRF, GATE, IIT‐JAM, M.Sc. Entrance, JEST, TIFR and GRE in Physics

For r  10 m;  60 degrees; q  106 Coulomb, and d  103 m , the electric dipole

potential in volts (rounded off to three decimal places) at a point  r ,   is _________

1 Nm 2
[Use:  9 109 2 ]
4 0 C
Ans. : 0.045
q q
Solution: Monopole moment     q  0
2 2
 q q
p     dyˆ    dyˆ   q  dzˆ 
2 2

p  qdzˆ

1 pr 1 qd cos 
V  r ,   
4 0 r 2
4 0 r2

106  103  cos 600


V  r ,   9  109 
10 
2

109
 9 10  9
 0.045
2  100

Q74. The electric field of an electromagnetic wave is given by E  3sin  kz   t  xˆ 

4 cos  kz  t  yˆ . The wave is

4
(a) linearly polarized at an angle tan 1   from the x - axis
3
3
(b) linearly polarized at an angle tan 1   from the x - axis
4
(c) elliptically polarized in clockwise direction when seen travelling towards the observer
(d) elliptically polarized in counter-clockwise direction when seen travelling towards the
observer
Ans. : (d)
Solution: At z  0, Ex  3sin t , E y  4 cos t y

At t  0, Ex  0, E y  4
x

H.No. 40-D, Ground Floor, Jia Sarai, Near IIT, Hauz Khas, New Delhi-110016
Phone: 011-26865455/+91-9871145498
Website: www.physicsbyfiziks.com | Email: fiziks.physics@gmail.com
28
fiziks
Institute for NET/JRF, GATE, IIT‐JAM, M.Sc. Entrance, JEST, TIFR and GRE in Physics


At  t  , Ex  3, E y  0
2
 m 
Q75. In a set of N successive polarizers, the m th polarizer makes an angle   with the
 2N 
vertical. A vertically polarized light beam of intensity I 0 is incident on two such sets with

N  N1 and N  N 2 , where N 2  N1 . Let the intensity of light beams coming out be

I  N1  and I  N 2  , respectively. Which of the following statements is correct about the

two outgoing beams?


(a) I  N 2   I  N1  ; the polarization in each case is vertical

(b) I  N 2   I  N1  ; the polarization in each case is vertical

(c) I  N 2   I  N1  ; the polarization in each case is horizontal

(d) I  N 2   I  N1  ; the polarization in each case is horizontal

Ans. : (c)
2 N1 2 N2
  n / 2    n / 2 
Solution: I  N1   I 0 cos   , I  N 2   I 0 cos  
  N1     N2 
I  N 2   I  N1 

For last polarization, pass axis will be horizontal.


Ex: N1  5

 
10
I  5   I 0 cos 18   0.605 I 0
 
N 2  10

 
20
I 10   I 0  cos 9   0.780 I 0
 
I 10   I  5 

H.No. 40-D, Ground Floor, Jia Sarai, Near IIT, Hauz Khas, New Delhi-110016
Phone: 011-26865455/+91-9871145498
Website: www.physicsbyfiziks.com | Email: fiziks.physics@gmail.com
29
fiziks
Institute for NET/JRF, GATE, IIT‐JAM, M.Sc. Entrance, JEST, TIFR and GRE in Physics

QUANTUM MECHANICS SOLUTIONS


GATE- 2010
Q1. Which of the following is an allowed wavefunction for a particle in a bound state? N is
a constant and  ,   0 .

e r
(a)   N (b)   N 1  e r 
r3
(c)   Ne x e   x  (d)   0non - zero constant if r  R
2
 y2 z2
 if r  R
Ans: (c)
Q2. A particle of mass m is confined in the potential
1
 m x ,
2 2
for x  0
V  x   2
 , for x  0
Let the wavefunction of the particle be given by
1 2
 x     0  1 ,
5 5
where  0 and  1 are the eigenfunctions of the ground state and the first excited state

respectively. The expectation value of the energy is


31 25 13 11
(a)  (b)  (c)  (d) 
10 10 10 10
Ans: (a)
Solution: For half parabolic potential
3 7 1 3 4 7 31
E0   , E1    E          .
2 2 5 2 5 2 10
2
Q3. For a spin-s particle, in the eigen basis of S , S x the expectation value sm S x2 sm is

(a)

 2 s s  1  m 2  
(b)  2 s s  1  2m 2 
2

(c)  s s  1  m 2
2
 (d)  2 m 2
Ans: (a)
1 1
sm  S  S   sm  sm S2  S 2  S  S   S  S  sm
2
Solution: sm S x2 sm 
4 4
 
2
1 
 sm S S   S  S  sm 
4 2
s s  1  m 2  S S  S S  2 S 2  S z2 
   

H.No. 40-D, Ground Floor, Jia Sarai, Near IIT, Hauz Khas, New Delhi-110016
Phone: 011-26865455/+91-9871145498
Website: www.physicsbyfiziks.com | Email: fiziks.physics@gmail.com
1
fiziks
Institute for NET/JRF, GATE, IIT‐JAM, M.Sc. Entrance, JEST, TIFR and GRE in Physics

Q4. A particle of mass m is confined in an infinite potential well:


0, if 0  x  L,
V  x  
 , otherwise.
 2x 
It is subjected to a perturbing potential V p  x   Vo sin   V x 
 L 
within the well. Let E 1 and E  2 be corrections to the ground
V p x 
state energy in the first and second order in V0 , respectively. L
0
Which of the following are true?
(a) E 1  0; E  2  0 (b) E 1  0; E  2  0

(c) E 1  0; E  2 depends on the sign of V0 (d) E 1  0; E  2  0


Ans: (a)
2
2
L
2x  m VP  1
Solution: E   V0 sin
1
1 dx  0 ; E12    E1  E m so E12  ve .
L0 L m 1 E1  Em

GATE- 2011
Q5. The quantum mechanical operator for the momentum of a particle moving in one
dimension is given by
d d  2 d 2
(a) i (b)  i (c) i (d) 
dx dx t 2m dx 2
Ans: (b)
Q6. An electron with energy E is incident from left on a potential V x 
barrier, given by
V0
0, for x  0
V  x   E
V0 , for x  0

as shown in the figure. For E < V0, the space part of the x
0
wavefunction for x > 0 is of the form
(a) e ax (b) e ax (c) eiax (d) e iax
Ans: (b)
Solution:  E  V0 , so there is decaying wave function.

H.No. 40-D, Ground Floor, Jia Sarai, Near IIT, Hauz Khas, New Delhi-110016
Phone: 011-26865455/+91-9871145498
Website: www.physicsbyfiziks.com | Email: fiziks.physics@gmail.com
2
fiziks
Institute for NET/JRF, GATE, IIT‐JAM, M.Sc. Entrance, JEST, TIFR and GRE in Physics

Q7. If Lx, Ly and Lz are respectively the x, y and z components of angular momentum operator
L. The commutator [Lx Ly, Lz] is equal to
(a) i L2x  L2y  (b) 2iLz (c) i L2x  L2y  (d) 0
Ans: (c)
 
Solution: Lx L y , Lz = Lx Ly Lz   Lx , Lz Ly = iL2x  L2y 

Q8. The normalized ground state wavefunciton of a hydrogen atom is given by


1 2 r / a
 r   e , where a is the Bohr radius and r is the distance of the electron
4 a
3/ 2

1
from the nucleus, located at the origin. The expectation value 2 is
r
8 4 4 2
(a) 2 (b) 2 (c) 2 (d) 2
a a a a
Ans: (d)
1 4  1 2  2ar  2 2
Solution: 2  3  2
r e dr   sin  d d  2
r 4 a 0 r 0 0 a
Q9. The normalized eigenstates of a particle in a one-dimensional potential well
0 if 0  x  a
V  x  
 otherwise
2  nx 
are given by  n  x   sin   , where n = 1, 2, 3,….
a  a 
The particle is subjected to a perturbation
 x  a
V0 cos   , for 0  x 
V ' x    a  2
0 , otherwise

The shift in the ground state energy due to the perturbation, in the first order perturbation
theory,
2Vo Vo Vo 2Vo
(a) (b) (c)  (d) 
3 3 3 3
Ans: (a)
a/2
x
sin 3
 x   x 
a/2 a/2
2 2 2V0
Solution: E11   V x  dx   sin  V0 cos dx  V0 a 
* 2
1 1
a  a  a a  3
0 0 3
a 0

Common data questions Q10 and Q11


H.No. 40-D, Ground Floor, Jia Sarai, Near IIT, Hauz Khas, New Delhi-110016
Phone: 011-26865455/+91-9871145498
Website: www.physicsbyfiziks.com | Email: fiziks.physics@gmail.com
3
fiziks
Institute for NET/JRF, GATE, IIT‐JAM, M.Sc. Entrance, JEST, TIFR and GRE in Physics

In a one-dimensional harmonic oscillator, 0 , 1 and 2 are respectively the ground, first


and the second excited states. These three states are normalized and are orthogonal to one
another  1 and  2 are two states defined by

 1  0  21  3 2 ,  2  0  1  2 ,  2  0  1   2
where  is a constant
Q10. The value of  which  2 is orthogonal to  1 is
(a) 2 (b) 1 (c) – 1 (d) – 2
Ans: (c)
Solution: For orthogonal condition scalar product  2 , 1   0 , so 1  2  3  0    1

Q11. For the value of α determined in Q10, the expectation value of energy of the oscillator in
the state  2 is

(a)  (b) 3 / 2 (c) 3 (d) 9 / 2


Ans: (b)
 3 5
2 H 2  
Solution:  2  0  1  2 put   1 , H   2 2 2  3 
2 2 3 2

GATE- 2012
Q12. A particle of mass m is confined in a two dimensional square well potential of
dimension a. This potential V(x, y) is given by
V(x, y) = 0 for –a < x < a and –a < y < a
= ∞ elsewhere
The energy of the first excited state for this particle is given by,
 2 2 2 2  2 5 2  2 4 2  2
(a) (b) (c) (d)
ma 2 ma 2 8ma 2 ma 2
Ans: (c)
 2 2  2 2 5 2  2
Solution: E   nx2  n y2   nx2  n y2    n x  1, n y  2 .
2m  2a 
2
8ma 2 8ma 2

H.No. 40-D, Ground Floor, Jia Sarai, Near IIT, Hauz Khas, New Delhi-110016
Phone: 011-26865455/+91-9871145498
Website: www.physicsbyfiziks.com | Email: fiziks.physics@gmail.com
4
fiziks
Institute for NET/JRF, GATE, IIT‐JAM, M.Sc. Entrance, JEST, TIFR and GRE in Physics
 
Q13. Consider the wavefunction    r1 , r2  s for a fermionic system consisting of two spin-
half particles. The spatial part of the wavefunction is given by
 
 r1 , r2  
1
1 r1  2 r2    2 r1 1 r2 
2
where 1and 2 are single particle states. The spin part χs of the wavefunction with spin
states   1 / 2 and - 1/2  should be
1 1
(a)     (b)     (c) αα (d) ββ
2 2
Ans: (b)
Solution: Since  r1 , r2  is symmetric the total wavefunction must be antisymmetric for fermions
so spin part must be antisymmetric.
Q14. A particle is constrained to move in a truncated harmonic potential well (x > 0) as shown
in the figure. Which one of the following statements is CORRECT?
(a) The parity of the first excited state is even
Vx
(b) The parity of the ground state is even
1
(c) the ground state energy is 
2
7 x
(d) The first excited state energy is 
2
Ans: (d)
3 7
Solution: There is only odd parity. Ground state is  and first excited  
2 2

Q15. Consider a system in the unperturbed state described by the Hamiltonian, H0  1 0 .
0 1 

  
The system is subjected to a perturbation of the form H '    , where   1 . The
  
energy eigenvalues of the perturbed system using the first order perturbation
approximation are
(a) 1 and (1 + 2δ) (b) (1 + δ) and (1 - δ)
(c) (1+ 2δ) and (1 - 2δ) (d) (1+ δ) and (1 - 2δ)
Ans: (a)

H.No. 40-D, Ground Floor, Jia Sarai, Near IIT, Hauz Khas, New Delhi-110016
Phone: 011-26865455/+91-9871145498
Website: www.physicsbyfiziks.com | Email: fiziks.physics@gmail.com
5
fiziks
Institute for NET/JRF, GATE, IIT‐JAM, M.Sc. Entrance, JEST, TIFR and GRE in Physics

Solution: H 0  H  , H0 is degenerate so after using degenerate perturbation through diagonalized


 2 0 1 0  2 0
H  one will get H     , H      .
0 0 0 1  0 0
So E  1  2 and 1 0 .

Q16. The ground state wavefunction for the hydrogen atom is given by
3/ 2
1  1 
 100    e  r / a0 , where a0 is the Bohr radius. The plot of the radial probability
4  a 0 
density, P(r) for the hydrogen atom in the ground state is

(a) P(r) (b) P(r)

r/a 0 r/a 0

(c) P(r) (d) P(r)

r/a 0 r/a 0
Ans: (d)
3/ 2
1 1
Solution: The ground state is given by  100    e r / a0
4  a0 
Radial probability function
P(r)
1 1 2 2 r / a0
Pr    r 2 =
2
r e
4 a03
r/a 0
Common Data for Questions 17–18
The wavefunction of particle moving in free space is given by,   eikx  2e  ikx  
Q17. The energy of the particle is
5 2 k 2 3 2 k 2 2k 2 2k 2
(a) (b) (c) (d)
2m 4m 2m m
Ans: (c)

H.No. 40-D, Ground Floor, Jia Sarai, Near IIT, Hauz Khas, New Delhi-110016
Phone: 011-26865455/+91-9871145498
Website: www.physicsbyfiziks.com | Email: fiziks.physics@gmail.com
6
fiziks
Institute for NET/JRF, GATE, IIT‐JAM, M.Sc. Entrance, JEST, TIFR and GRE in Physics

  2  2   2
Solution: H  E , H    ik  ik  eikx  2  ik  ik  eikx 
2m x 2
2m
 2 k 2 ikx 2 k 2
 H  (e  2e 2ikx )  
2m 2m

Q18. The probability current density for the real part of the wavefunction is
k k
(a) 1 (b) (c) (d) 0
m 2m
Ans: (d)
Solution: The real part of the wave function real  cos kx  2 cos kx
Current density for real part of wave function = 0

GATE- 2013
Q19. Which one of the following commutation relations is NOT CORRECT? Here, symbols
have their usual meanings.
 
(a) L2 , L z  0  
(b) L x , L y  iL z

(c) L z , L   L (d) L z , L   L


Ans: (d)
Q20. A proton is confined to a cubic box, whose sides have length 10 12 m . What is the
minimum kinetic energy of the proton? The mass of proton is 1.67  10 27 kg and

Planck’s constant is 6.63  10 34 Js .


(a) 1.1  10 17 J (b) 3.3  10 17 J (c) 9.9  10 17 J (d) 6.6  10 17 J
Ans: (c)
3 2  2
Solution: 2
 9.9 1017
2ma
Q21. A spin-half particle is in a linear superposition 0.8   0.6  of its spin-up and spin-

down states. If  and  are the eigenstates of  z , then what is the expectation value

up to one decimal place, of the operator 10 z  5 x ? Here, symbols have their usual
meanings. _______________
Ans: 7.6

H.No. 40-D, Ground Floor, Jia Sarai, Near IIT, Hauz Khas, New Delhi-110016
Phone: 011-26865455/+91-9871145498
Website: www.physicsbyfiziks.com | Email: fiziks.physics@gmail.com
7
fiziks
Institute for NET/JRF, GATE, IIT‐JAM, M.Sc. Entrance, JEST, TIFR and GRE in Physics

1  0   0.8 
Solution:   .8   .6   0.8    0.6     
0  1   0.6 
1 0  0 1 10 5 
Operator A  10  z  5 x  10    5   A 
 0 1   1 0   5 10 
 10 5   0.8 
A   A    0.8 0.6      =  8.8  1.2   7.6
 5 10   0.6 
Q22. Consider the wave function Ae i k r r0 / r  , where A is the normalization constant.

For r  2r0 , the magnitude of probability current density up to two decimal places, in

 
units of A 2 k / m is _____________
Ans: 0.25
2 2
 k 2 r k 2 r k 2 k 2 k
  0.25  A
2
Solution: J    A 0 J A 0 J A
m r m 2r0 m 4m m

Common data questions 23 and 24


5 2 0 
To the given unperturbed Hamiltonian 2 5 0
0 0 2

1 1 1
we add a small perturbation given by  1 1  1 where  is small quantity.

1  1 1 

Q23. The ground state eigenvector of the unperturbed Hamiltonian is



(a) 1 / 2 ,1 2 ,0  
(b) 1 / 2 ,1 / 2 ,0 
(c) 0,0,1 (d) 1,0,0 
Ans: (c)
5 2 0 1 1 1
   
H 0   2 5 0  , H P    1 1  1
0 0 2 1  1 1 
 

H.No. 40-D, Ground Floor, Jia Sarai, Near IIT, Hauz Khas, New Delhi-110016
Phone: 011-26865455/+91-9871145498
Website: www.physicsbyfiziks.com | Email: fiziks.physics@gmail.com
8
fiziks
Institute for NET/JRF, GATE, IIT‐JAM, M.Sc. Entrance, JEST, TIFR and GRE in Physics

Eigen value of H 0 is E1  2, E2  3, E3  7 and the Eigen vector corresponds

0 1 1


1   1  
to 1   0  , 2  1 , 3  1 .
1 2   2  
  0 0
Q24. A pair of eigenvalues of the perturbed Hamiltonian, using first order perturbation theory,
is
(a) 3  2 ,7  2 (b) 3  2 ,2   (c) 3, 7  2 (d) 3, 2  2
Ans: (c)
Solution: E1  1 H P 1  1  E1  2  1

1 1 1 1 1
  1  
E2  2 H P 2 
1
1  1 0. 1 1  1.   1   0 0 1  1  0
2 1  1 1  2  0  0
     

1 1 1  1 1
1   1   1  
E3  3 H P 3  1 1 0  . 1 1 1 .  1  =  . 2 2 0. 1 
2 1 1 1  2  0  2  0
     
1
 E 3  4  2
2
E1  2  1 , E 2  3  0 , E3  7  2 .

GATE- 2014
Q25. The recoil momentum of an atom is p A when it emits an infrared photon of wavelength

1500 nm , and it is p B when it emits a photon of visible wavelength 500 nm . The ratio

pA
is
pB

(a) 1 : 1 (b) 1 : 3 (c) 1 : 3 (d) 3 : 2


Ans: (c)
h p A B  B 500
Solution: p  ,  ,  =1 : 3
 pB A  A 1500
Q26. The ground state and first excited state wave function of a one dimensional infinite
potential well are  1 and  2 respectively. When two spin-up electrons are placed in this
H.No. 40-D, Ground Floor, Jia Sarai, Near IIT, Hauz Khas, New Delhi-110016
Phone: 011-26865455/+91-9871145498
Website: www.physicsbyfiziks.com | Email: fiziks.physics@gmail.com
9
fiziks
Institute for NET/JRF, GATE, IIT‐JAM, M.Sc. Entrance, JEST, TIFR and GRE in Physics

potential which one of the following with x1 and x2 denoting the position of the two
electrons correctly represents the space part of the ground state wave function of the
system?
1 1
(a)  1 x1  2 x1    1 x2  2 x2  (b)  1 x1  2 x2    1 x2  2 x1 
2 2
1 1
(c)  1 x1  2 x1    1 x2  2 x2  (d  1 x1  2 x 2    1 x 2  2 x1  )
2 2
Ans: (d)
Solution: From the given information only possible spin configuration is symmetric in nature so
space part will anti symmetric
1
 1 x1  2 x 2    1 x 2  2 x1 
2
 
Q27. If L is the orbital angular momentum and S is the spin angular momentum, then L.S
does not commute with
 
(a) S z (b) L2 (c) S 2 
(d) L  S  2

Ans: (d)
Q28. An electron in the ground state of the hydrogen atom has the wave function
 r 
  
 1
 r  

 a0 
e , where a0 is constant. The expectation value of the operator
a 03

Qˆ  z 2  r 2 , where z  r cos  is
 n  n  1! )
(Hint: 0
e  ar r n dr 
a n 1

a n 1
 a 02  3a 02
(a) (b)  a 02 (c) (d)  2a02
2 2
Ans: (d)

Solution: Qˆ  z 2  r 2  a02  3a02  2a02

Q29. A particle of mass m is subjected to a potential


1
V  x, y  
2
 
m 2 x 2  y 2 ,    x  ,  y  

H.No. 40-D, Ground Floor, Jia Sarai, Near IIT, Hauz Khas, New Delhi-110016
Phone: 011-26865455/+91-9871145498
Website: www.physicsbyfiziks.com | Email: fiziks.physics@gmail.com
10
fiziks
Institute for NET/JRF, GATE, IIT‐JAM, M.Sc. Entrance, JEST, TIFR and GRE in Physics

The state with energy 4  is g  fold degenerate. The value of g is ______


Ans: 4
Solution: This is two isotropic dimensional harmonic oscillator the energy eigen value for nth
state is E n  (n  1) with degeneracy g n  (n  1) so degeneracy for 4  is 4 .
Q30. A hydrogen atom is in the state

8 3 4
  200   310   321 ,
21 7 21
where n, l , m in  nl m denote the principal, orbital and magnetic quantum numbers,

respectively. If L is the angular momentum operator, then the average value of L2
is_______  2
Ans: 2
Solution: If L2 will measure on state  the measurement is 0 2 , 2 2 and 6 2 with probability
8 3 4 3 4
, , so, L2  2 2   6 2  = 2 2
21 7 21 7 21
1
Q31.  1and 2 are two orthogonal states of a spin system. It is given that
2
1 1 2 0 1 0
1      , where   and   represent the spin-up and spin-down states,
3 0 3 1  0 1
respectively. When the system is in the state  2 its probability to be in the spin-up state
is_______
2
Ans:
3
1 1 2 0 2 1 1  0
Solution: If is  1      , then  2      ,
3 0 3  1  3  0 3  1 

2
so probability that  2 is in up state is
3

Q32. A particle is confined to a one dimensional potential box, with the potential
0, 0 xa
V  x  
 , otherwise

H.No. 40-D, Ground Floor, Jia Sarai, Near IIT, Hauz Khas, New Delhi-110016
Phone: 011-26865455/+91-9871145498
Website: www.physicsbyfiziks.com | Email: fiziks.physics@gmail.com
11
fiziks
Institute for NET/JRF, GATE, IIT‐JAM, M.Sc. Entrance, JEST, TIFR and GRE in Physics

If particle is subjected to a perturbation within the box. W   x . Where  is small


constant, the first order correction to the ground state energy is
(a) 0 (b) a / 4 (c) a / 2 (d) a
Ans: (c)
Solution: First order energy correction is W   x . The average value of position in ground

a
state is x  so answer is a / 2
2
Q33. A one dimensional harmonic oscillator is in the superposition of number state n given

1 3
by   2  3 .
2 2
The average energy of the oscillator in the given state is______  .
Ans: 3.25
1 5 3 7 
  
Solution: Average energy will 4 2 4 2  3.25
1 3

4 4
Q34. If L and L are the angular momentum ladder operators then the expectation value of

L L  L L  in the state l  1, m  1 of an atom is _____  2

Ans: 2
Solution: L L  L L   2( L2  L2z )  2(l.(l  1)  m 2 ) 2 = 2 2

H.No. 40-D, Ground Floor, Jia Sarai, Near IIT, Hauz Khas, New Delhi-110016
Phone: 011-26865455/+91-9871145498
Website: www.physicsbyfiziks.com | Email: fiziks.physics@gmail.com
12
fiziks
Institute for NET/JRF, GATE, IIT‐JAM, M.Sc. Entrance, JEST, TIFR and GRE in Physics

GATE- 2015
1  
Q35. An operator for a spin particle is given by     B , where
2
 B 
B  xˆ  yˆ  ,  denotes Pauli matrices and  is a constant. The eigenvalues of  are
2
B
(a)  (b)  B (c) 0, B (d) 0,   B
2
Ans.: (b)
   B
Solution: Aˆ     , B  xˆ  yˆ 
2

Aˆ    x Bx   y By   z Bz   Aˆ    x Bx   y By 

 0 1  B  0 i  B  B  0 1 i
Aˆ         Aˆ   
 1 0  2  i 0  2  2 1  i 0 

 B   1  i 
A   I  0     0      B
2 1  i  
1   
Q36. The Pauli matrices for three spin  particles are  1 ,  2 and  3 , respectively. The
2
  
dimension of the Hilbert space required to define an operator Oˆ   1   2   3 is_______
Ans.: 8
Solution:  2   3 has dimension of 4 and  1.  2   3 has dimension of 2  4  8

 
Q37. Let L and p be the angular and linear momentum operators, respectively, for a a particle.

The commutator L x , p y gives 
(a) i pz (b) 0 (c) i px (d) i pz
Ans.: (d)
Solution:  Lx , p y    ypz  zp y , p y    ypz , p y    zp y , p y    y, p y  pz

  p y , p y   0 and  z , p y   0   Lx , p y   ipz   y, p y   i

H.No. 40-D, Ground Floor, Jia Sarai, Near IIT, Hauz Khas, New Delhi-110016
Phone: 011-26865455/+91-9871145498
Website: www.physicsbyfiziks.com | Email: fiziks.physics@gmail.com
13
fiziks
Institute for NET/JRF, GATE, IIT‐JAM, M.Sc. Entrance, JEST, TIFR and GRE in Physics

3
Q38. Consider a system of eight non-interacting, identical quantum particles of spin  in a
2
one dimensional box of length L . The minimum excitation energy of the system, in units
 2 2
of is ________
2mL2
Ans.: 5
3  3 
Solution: spin  degeneracy   2S  1   2   1  4
2  2 
 22 4 2  2 20 2  2
Eground  4   4 
2mL2 2mL2 2mL2
st  22  22  22  2 2
I
Eexcited  4  3 4  1 9  25
2mL2 2mL2 2mL2 2mL2
st  22  22  2 2
Now minimum excitation energy E  Eexcited
I
 Eground  25  20 5
2mL2 2mL2 2mL2
Q39. A particle is confined in a box of length L as shown in the figure. If the
potential V0 is treated as a perturbation, including the first order
correction, the ground state energy is
 2 2  2  2 V0
(a) E   V0 (b) E   V0
2mL2 2mL2 2 L/2
 
2 2
V  
2
V 2
(c) E  2
 0 (d) E  2
 0
2mL 4 2mL 2
Ans.: (d)
 L2 
2 x 2 x
L
2  
Solution: E0    V0 sin
1
dx   0  sin dx 
L 0 L L
 L
2

L L
2 V0  2 x 
2
V0   2 x  L  2
E   1  cos dx  
1
 x sin   
 L  2  0
0
L 2 0 L  L 

V0  2 2 V0
 E01  E 
2 2mL2 2

H.No. 40-D, Ground Floor, Jia Sarai, Near IIT, Hauz Khas, New Delhi-110016
Phone: 011-26865455/+91-9871145498
Website: www.physicsbyfiziks.com | Email: fiziks.physics@gmail.com
14
fiziks
Institute for NET/JRF, GATE, IIT‐JAM, M.Sc. Entrance, JEST, TIFR and GRE in Physics
 
Q40. Let the Hamiltonian for two spin-½ particles of equal masses m , momenta p1 and p2
 
and positions r1 and r2 be H 
1 2
2m
p1 
1 2 1
2m
  
 
p 2  m 2 r12  r22  k 1   2 , where  1
2

and  2 denote the corresponding Pauli matrices,   0.1eV and k  0.2 eV . If the
ground state has net spin zero, then the energy (in eV ) is ___________
Ans.: 0.3
1 2 1 2 1  
Solution: H  p1  p2  m 2  r12  r22   k 1. 2
2m 2m 2
    2      2
   1   2     12   22  2 1. 2  2 1. 2     12   22
   
 2 1. 2  0  3I  3I  6 I   1. 2  3
3
Now energy E  2    k  3  3   0.1   0.2  3  0.3 eV
2
Q41. Suppose a linear harmonic oscillator of frequency  and mass m is in the state

1  i 
  
 0  e 2
 1  at t  0 where  0 and  1 are the ground and the first
2 


excited states, respectively. The value of  x  in the units of at t  0 is _____
m
Ans. : 0
i
1  
Solution:     0  e 1 
2

2 

 
 x  
2m
 a  a†     x  
2m
  a    a†  
1 i2 1  i

a   e 0 and a †   
 1  2 e 2
2 
2 2 

  1 1 i2 1 1  i2 
  x   e    e 1 1 
2m  2 2
0 0
2 2 


  x  0  0
2m

H.No. 40-D, Ground Floor, Jia Sarai, Near IIT, Hauz Khas, New Delhi-110016
Phone: 011-26865455/+91-9871145498
Website: www.physicsbyfiziks.com | Email: fiziks.physics@gmail.com
15
fiziks
Institute for NET/JRF, GATE, IIT‐JAM, M.Sc. Entrance, JEST, TIFR and GRE in Physics

GATE-2016
Q42. Which of the following operators is Hermitian?
d d2 d2 d3
(a) (b) 2 (c) i 2 (d) 3
dx dx dx dx
Ans. : (b)
Q43. The scattering of particles by a potential can be analyzed by Born approximation. In
particular, if the scattered wave is replaced by an appropriate plane wave, the
corresponding Born approximation is known as the first Born approximation. Such an
approximation is valid for
(a) large incident energies and weak scattering potentials.
(b) large incident energies and strong scattering potentials.
(c) small incident energies and weak scattering potentials.
(d) small incident energies and strong scattering potentials.
Ans.: (a)
Q44. Consider an elastic scattering of particles in l  0 states. If the corresponding phase shift
 0 is 900 and the magnitude of the incident wave vector is equal to 2 fm 1 then the

total scattering cross section in units of fm 2 is _______.


Ans.: 2
4
2 
Solution:   2l  1 sin  0 for l  0 , it is given  0  90 0 and k  2 fm 1
k l 0
4
 sin 90  2
2
Q45. A hydrogen atom is in its ground state. In the presence of a uniform electric field

E  E0 zˆ , the leading order change in its energy is proportional to  E0  . The value of
n

the exponent n is _______.


Ans.: 2
Solution: First order energy correction is zero  1,0,0 E0 r cos   1,0,0  0
2
 n,l ,m E0 r cos   1,0,0
So one need to find correction of second   E02
n 1 E10  Em0

So value of n  2
H.No. 40-D, Ground Floor, Jia Sarai, Near IIT, Hauz Khas, New Delhi-110016
Phone: 011-26865455/+91-9871145498
Website: www.physicsbyfiziks.com | Email: fiziks.physics@gmail.com
16
fiziks
Institute for NET/JRF, GATE, IIT‐JAM, M.Sc. Entrance, JEST, TIFR and GRE in Physics
 
Q46. If s1 and s2 are the spin operators of the two electrons of a He atom, the value of
 
s1 .s 2 for the ground state is

3 3 1 2
(a)   2 (b)   2 (c) 0 (d) 
2 4 4
Ans.: (b)
   1 1   s s  1 2  s1 s1  1 2  s 2 s 2  1 2
Solution: s  s1  s2 , s1  , s1  , s  0,1 , s1 .s 2 
2 2 2
3 3
2 2   2   2
  4 4  3 2
For s  1, s1  s2 
2 4
3 3
0 2   2   2
  4 4   3 2
s  0, s1  s2 
2 4
Q47. A two-dimensional square rigid box of side L contains six non-interacting electrons at
T  0 K . The mass of the electron is m . The ground state energy of the system of
 2 2
electrons, in units of is _________.
2mL2
Ans.: 24

Solution: 2 
1 2

 12  2  2
 4
2 2

 12  2  2

24 2  2
2mL2 2mL2 2mL2
Q48.  x ,  y and  z are the Pauli matrices. The expression 2 x y   y x is equal to

(a)  3i z (b)  i z (c) i z (d) 3i z


Ans.: (c)
Solution: 2 x y   y x   x y   x y   y x   x y  i z

Q49. If x and p are the x components of the position and the momentum operators of a

particle respectively, the commutator x 2 , p 2 is  


(a) i  xp  px  (b) 2i  xp  px  (c) i  xp  px  (d) 2i  xp  px 
Ans.: (d)
     
Solution: x 2 , p 2  p x 2 , p  x 2 p p  2ipx  2ixp  2i  xp  px 

H.No. 40-D, Ground Floor, Jia Sarai, Near IIT, Hauz Khas, New Delhi-110016
Phone: 011-26865455/+91-9871145498
Website: www.physicsbyfiziks.com | Email: fiziks.physics@gmail.com
17
fiziks
Institute for NET/JRF, GATE, IIT‐JAM, M.Sc. Entrance, JEST, TIFR and GRE in Physics

Q50. Let l, m be the simultaneous eigenstates of L2 and L z . Here L is the angular

momentum operator with Cartesian components L x , L y , L z , l is the angular momentum

quantum number and m is the azimuthal quantum number. The value of

1, 0 ( Lx  iLy ) 1, 1 is

(a) 0 (b)  (c) 2 (d) 3


Ans.: (c)

Solution: 1, 0 ( Lx  iLy 1, 1  1, 0 L 1, 1  2 1, 0 1, 0  2

Q51. For the parity operator P , which of the following statements is NOT true?
(a) P†  P (b) P 2   P (c) P 2  I (d) P †  P 1
Ans.: (b)
Q52. The state of a system is given by   1  2  2  3 3 , where 1 , 2 and 3 form

an orthonormal set. The probability of finding the system in the state  2 is ________.

(Give your answer upto two decimal places)


Ans. : 0.28

22 4 4 2
Solution: Probability that  in state 2      0.28
2 2
1 2 32
1  4  9 14 7
Q53. A particle of mass m and energy E , moving in the positive x
V0
direction, is incident on a step potential at x  0 , as indicated in the
E
figure. The height of the potential is V0 , where V0  E . At x  x0 ,

1
where x0  0 , the probability of finding the electron is times the
e x0 x  x0
2mV0  E 
probability of finding it at x  0 . If   , the value of x0 is
2
2 1 1 1
(a) (b) (c) (d)
  2 4
Ans.: (c)
1 1
Solution:  e 2 x0  e 1  e 2 x0  x0 
e 2

H.No. 40-D, Ground Floor, Jia Sarai, Near IIT, Hauz Khas, New Delhi-110016
Phone: 011-26865455/+91-9871145498
Website: www.physicsbyfiziks.com | Email: fiziks.physics@gmail.com
18
fiziks
Institute for NET/JRF, GATE, IIT‐JAM, M.Sc. Entrance, JEST, TIFR and GRE in Physics

GATE- 2017
Q54. The Compton wavelength of a proton is…………….. fm. (up to two decimal places).
Ans. : 3  108
Solution:  m p  1.67 1027 kg , h  6.626  1034 Js, e  1.602  1019 C , c  3 108 ms 1 

p2 1 2
Q55. A one dimensional simple harmonic oscillator with Hamiltonian H 0   kx is
2m 2
subjected to a small perturbation, H1   x   x 3   x 4 . The first order correction to the
ground state energy is dependent on
(a) only  (b)  and  (c)  and  (d) only 
Ans. : (d)

Solution: H1   x   x 3   x 4 , E1g   x   x3   x 4 , x  0, x3  0, x 4  0
  
Q56. For the Hamiltonian H  a0 I  b . where a0  R, b is a real vector, I is the 2  2

identity matrix, and  are the Pauli matrices, the ground state energy is
(a) b (b) 2a0  b (c) a0  b (d) a0

Ans. : (c)
  1 0 0 1  0 i   1 0   a0  bz bx  iby 
Solution: a0 I  b .  a0  
 xb 
 yb 
 zb  
0 1 1 0 i 0   0 1  bx  iby a0  bz 

   a0  bz bx  iby 
H  a0 I  b .   
 bx  iby a0  bz 

 a0  bz   bx  iby 
For eigen value  0
 bx  iby a0  bz   

 a0  bz    a0  bz      bx2  by2   0
1  a0  b , 1  a0  b
Q57. The degeneracy of the third energy level of a 3-dimensional isotropic quantum harmonic
oscillator is
(a) 6 (b) 12 (c) 8 (d) 10
Ans. : (a)

H.No. 40-D, Ground Floor, Jia Sarai, Near IIT, Hauz Khas, New Delhi-110016
Phone: 011-26865455/+91-9871145498
Website: www.physicsbyfiziks.com | Email: fiziks.physics@gmail.com
19
fiziks
Institute for NET/JRF, GATE, IIT‐JAM, M.Sc. Entrance, JEST, TIFR and GRE in Physics

Solution: First energy level is n  0


Second energy level is n  1
Third energy level is n  2

Degeneracy of third level


 n  1 n  2   3  4  6
2 2
Q58. A free electron of energy 1eV is incident upon a one-dimensional finite potential step of
height 0.75eV . The probability of its reflection from the barrier is…………. (up to two
decimal places).
Ans. : 0.11
2 2
 E  E  V0   1  0.25   1  0.5 2
Solution: R           0.11
 
 E  E  V0   1  0.25   1  0.5 
Q59. Consider a one-dimensional potential well of width 3nm . Using the uncertainty principle

 
 x  p   , an estimate of the minimum depth of the well such that it has at least one
 2

bound state for an electron is ( me  9.311031 kg , h  6.626 1034 Js, e  1.602  1019 C )

(a) 1  eV (b) 1meV (c) 1eV (d) 1MeV


Ans. : (b)

p2  
Solution: E  , p   p 
2m 2x 2a

 
2
2 h2 6.6  1034
So, E    31 18
 .0011019 J  1 meV
8ma 2
32 ma
2 2
32 10  9.3110  9 10

x e
2  x2
Q60. The integral dx is equal to……….. (up to two decimal places).
0

Ans. : 0.44

x
2
Solution: The given integral is 2
e  x dx
0

dt
Let x 2  t then 2 xdx  dt  dx 
2 t

H.No. 40-D, Ground Floor, Jia Sarai, Near IIT, Hauz Khas, New Delhi-110016
Phone: 011-26865455/+91-9871145498
Website: www.physicsbyfiziks.com | Email: fiziks.physics@gmail.com
20
fiziks
Institute for NET/JRF, GATE, IIT‐JAM, M.Sc. Entrance, JEST, TIFR and GRE in Physics

Thus, the given integral can be written as


  
dt 1  t 1/ 2 1  t 32 1 1 3 1 1 1 
0 t e 2 t  2 0 e t dt  2 0 e t dt  2   2   2  2   2   4
t

Hence the value of the integral up to two decimal places is 0.44 .


Q61. Which one of the following operators is Hermitian?

(a) i
p x x
2
 x 2 px 
(b) i
p x x
2
 x 2 px 
2 2
(c) ei px a (d) e  i px a
Ans. : (a)

Solution: A  i
p x x
2
 x 2 px 
, A  i

 p x    x p    i  p x  x p 
x
2 † 2
x

x
2 2
x

2 2 2

GATE-2018

Q62. The ground state energy of a particle of mass m in an infinite potential well is E0 . It

 22
 
changes to E0 1   103 , when there is a small potential pump of height V0 
50 mL2
and width a  L /100 , as shown in the figure. The value of  is ________ (up to two
decimal places). V  x

x a
V0
L
Ans. : 0.81
L a L a L
Solution: 1     ,  2     , a 
 2 2  2 2 100
2 2
 2 2 x 
E1  V0    sin   dx
1  L  L 
2 
V  2 x  V  L 2 x  2
 0
L  1  cos L  dx  L0  x 
2
sin
L 1
1

H.No. 40-D, Ground Floor, Jia Sarai, Near IIT, Hauz Khas, New Delhi-110016
Phone: 011-26865455/+91-9871145498
Website: www.physicsbyfiziks.com | Email: fiziks.physics@gmail.com
21
fiziks
Institute for NET/JRF, GATE, IIT‐JAM, M.Sc. Entrance, JEST, TIFR and GRE in Physics

V0  L  2  L  a  2  L  a   
 a   sin  sin 
L  2  2L 2L  

V0  L L   a    a  
    sin      sin     
L 100 2   L   L  

 1
 0.0314  0.0314 
1
 V0  
100 2 
 20 
 V0 103 10  10   E0 103      E0 103  0.81 E0 103
 25 
Hence,   0.81
Q63. A two-state quantum system has energy eigenvalues   corresponding to the normalized
1
states   . At time t  0 , the system is in quantum state        . The
2
probability that the system will be in the same state at t  h /  6  is _________ (up to

two decimal places).


Ans. : 0.25
1
Solution:   0         
2

1  
it it

And   t   
  e 
   e 

2 

At t  ,
6
ih 2 ih 2  i i
1    1  
 t   
  e 6h
  e 6h
  
  e 3
   e 3

2  2 
Now, probability in same state
2
 t    0 1 2 1 
2
1 1
2

P  e  i / 3  ei / 3  2 cos   2  0.25


 4 4 3 4 2

H.No. 40-D, Ground Floor, Jia Sarai, Near IIT, Hauz Khas, New Delhi-110016
Phone: 011-26865455/+91-9871145498
Website: www.physicsbyfiziks.com | Email: fiziks.physics@gmail.com
22
fiziks
Institute for NET/JRF, GATE, IIT‐JAM, M.Sc. Entrance, JEST, TIFR and GRE in Physics

GATE-2019

Q64. An electric field E  E0 zˆ is applied to a Hydrogen atom in n  2 excited state. Ignoring

spin the n  2 state is fourfold degenerate, which in the l , m basis are given by

0, 0 , 1,1 , 1, 0 and 1, 1 . If H  is the interaction Hamiltonian corresponding to the

applied electric field, which of the following matrix elements is nonzero?


(a) 0, 0 H  0, 0 (b) 0, 0 H  1,1

(c) 0, 0 H  1, 0 (d) 0, 0 H  1, 1

Ans. : (c)
1
Q65. For a spin particle, let  and  denote its spin up and spin down states
2
respectively. If a 
1
2

     and b   1
2

     are composite
states of two such particles, which of the following statements is true for their total spin
S?
(a) S  1 for a and b is not an eigenstate of the operator Ŝ 2

(b) a is not an eigenstate of the operator Ŝ 2 and S  0 for b

(c) S  0 for a , and S  1 for b

(d) S  1 for a , and S  0 for b

Ans. : (d)
Solution: S  1 is triplet a , and S  0 for singlet for b

Q66. The Hamiltonian for a quantum harmonic oscillator of mass m in three dimensions is
p2 1
H  m 2 r 2
2m 2
where  is the angular frequency. The expectation value of r 2 in the first excited state of

the oscillator in units of (rounded off to one decimal place) is___________
m
Ans. : 2.5

Solution: r 2  x 2  y 2  z 2

H.No. 40-D, Ground Floor, Jia Sarai, Near IIT, Hauz Khas, New Delhi-110016
Phone: 011-26865455/+91-9871145498
Website: www.physicsbyfiziks.com | Email: fiziks.physics@gmail.com
23
fiziks
Institute for NET/JRF, GATE, IIT‐JAM, M.Sc. Entrance, JEST, TIFR and GRE in Physics

 

2m 
 
 2nx  1  2n y  1   2nz  1
For first excited state nx  1, n y  0, nz  0

Hence it is triply degenerate one can take


nx  0, n y  1, nz  0 or nx  0, n y  0, nz  1

5  
putting any one combination, expectation value of r 2   2.5
2 m m
1  0
Q67. Let  2    ,  2    represent two possible states of a two-level quantum system.
0 1 
The state obtained by the incoherent superposition of  1 and  2 is given by a density

matrix that is defined as   c1  1  1  c2  2  2 . If c1  0.4 and c2  0.6 , the matrix

element  22 (rounded off to one decimal place) is __________

Ans. : 0.6
Solution:  2,2   2   2    c1  2  1  1  2  c2  2  2  2  2

 c2  0.6

Q68. The wave function   x  of a particle is as shown below


  x

d d

K
x
a/2 a/2

Here K is a constant, and a  d . The position uncertainty  x  of the particle is

a 2  3d 2 3a 2  d 2 d2 d2
(a) (b) (c) (d)
12 12 6 24
Ans. : (b)

H.No. 40-D, Ground Floor, Jia Sarai, Near IIT, Hauz Khas, New Delhi-110016
Phone: 011-26865455/+91-9871145498
Website: www.physicsbyfiziks.com | Email: fiziks.physics@gmail.com
24
fiziks
Institute for NET/JRF, GATE, IIT‐JAM, M.Sc. Entrance, JEST, TIFR and GRE in Physics

 a d a d
k ,  2  2  x   2  2

 0,  a  d  x  a  d
 2 2 2 2
Solution:   x   
 k, a  d  x  a  d
 2 2 2 2
 a d
 0,  0
 2 2
  1
a d a d
  
2 2 2 2
k2  dx  k a d dx  1
2

a d
  
2 2 2 2

 a d   a d    a d   a d  
k 2            k 2           1
 2 2   2 2    2 2   2 2  
d d d d  1
k2      1 k 
2 2 2 2 2d
Hence wavefunction is symmetric about x  0 , so x  0
a d a d
  
2 2 2 2
x2  k 2  x dx  k a d x dx
2 2 2

a d
  
2 2 2 2

k2  3 ad   
a d

  x  a d   x  a d 
3 2 2
 2 2
3   
2 2 2 2 
 

k2 
 a  d    a  d    a  d    a  d  
3 3 3 3

3 8  


k2
24
    
 a3  d 3  3a 2 d  3ad 2  a3  d 3  3a 2 d  3ad 2  a 3  d 3  3a 2 d  3ad 2 
 
 a 3  d 3  3ad  a  d  

x 2

k2
 4a  12a d  
3 2
4d d 2  3a 2 


3a 2  d 2
24   24  2d 12

H.No. 40-D, Ground Floor, Jia Sarai, Near IIT, Hauz Khas, New Delhi-110016
Phone: 011-26865455/+91-9871145498
Website: www.physicsbyfiziks.com | Email: fiziks.physics@gmail.com
25
fiziks
Institute for NET/JRF, GATE, IIT‐JAM, M.Sc. Entrance, JEST, TIFR and GRE in Physics

2 3a 2  d 2
x  x2  x 
12
Q69. Consider the motion of a particle along the x - axis in a potential V  x   F x . Its ground

state energy E0 is estimated using the uncertainty principle. Then E0 is proportional to

(a) F 1/ 3 (b) F 1/ 2 (c) F 2 / 5 (d) F 2 / 3


Ans. : (d)

p2 p2 p2
Solution: E  F x E  Fx for x  0 E   Fx  0 from uncertainty theory
2m 2m 2m

x.p    p 
x

 p 2  F 2
E  x   E   F x
2m  x 
2
2m

For minimum energy,


1/ 3 2/3 1/ 3
dE 2  2   2  mF   2 
  F  0  x      F   E  F 2/3
d x m  x 
3
 mF  2m   2   mF 

E 0
Q70. The Hamiltonian operator for a two-level quantum system is H   1  . If the state
0 E2 

1 1 2
of the system at t  0 is given by   0     then   0    t  at a later time t
2 1
is

(a)
1
2

1  e  E1  E2 t /   (b)
1
2

1  e  E1  E2 t /  
1 1
(c)
2
1  cos  E1  E2  t /   (d)
2
1  cos  E1  E2  t /  
Ans. : (c)
 iE t 
 exp 1 
1 1 1 
Solution:   0      t    
2 1 2 iE2t 
 exp 
  

H.No. 40-D, Ground Floor, Jia Sarai, Near IIT, Hauz Khas, New Delhi-110016
Phone: 011-26865455/+91-9871145498
Website: www.physicsbyfiziks.com | Email: fiziks.physics@gmail.com
26
fiziks
Institute for NET/JRF, GATE, IIT‐JAM, M.Sc. Entrance, JEST, TIFR and GRE in Physics

2
1 iE t iE t 1
 exp 1  exp  2  1  cos  E1  E2  t /  
2
  0  t 
4   2

Q71. Consider a potential barrier V  x  of the form:


V  x
V  x  
V0

x0 xa xb x

where V0 is a constant. For particles of energy E  V0 incident on this barrier from the
left which of the following schematic diagrams best represents the probability density

  x  as a function of x ?
2

  x   x
2 2

(a) (b)

x0 xa xb x x0 xa xb x

  x   x
2 2

(c) (d)

x0 xa xb x x0 xa xb x


Ans. : (a)
1  
Q72. The Hamiltonian of a system is H    with   1 . The fourth order contribution
  1 
to the ground state energy of H is  4 . The value of  (rounded off to three decimal
places) is_________.
Ans. : 0.125
1  
 the eigen value of the hamiltonion is Eg   1   , E f   1  
2 2
Solution: H  
  1 

H.No. 40-D, Ground Floor, Jia Sarai, Near IIT, Hauz Khas, New Delhi-110016
Phone: 011-26865455/+91-9871145498
Website: www.physicsbyfiziks.com | Email: fiziks.physics@gmail.com
27
fiziks
Institute for NET/JRF, GATE, IIT‐JAM, M.Sc. Entrance, JEST, TIFR and GRE in Physics

The ground state is Eg   1   2

 2 4  2 4
Taylor expansion of  1   2   1   .....   1   .....
 2 8  2 8

1
   0.125
8
Q73. Electrons with spin in the z - direction  ẑ  are passed through a Stern-Gerlach (SG) set

up with the magnetic field at   600 from ẑ . The fraction of electrons that will emerge
with their spin parallel to the magnetic field in the SG set up (rounded off to two decimal
places) is___________
 0 1  0 i   1 0 
 x   ,  y   ,  z   
 1 0 i 0   0 1  
Ans. : 0.25
 cos 600   1/ 2  1 1 1
Solution:       state related to up state is ,     
 sin 600  2 2 0
   3 / 2
The fraction of electrons that will emerge with their spin parallel to the magnetic field
2 1
   0.25
4

H.No. 40-D, Ground Floor, Jia Sarai, Near IIT, Hauz Khas, New Delhi-110016
Phone: 011-26865455/+91-9871145498
Website: www.physicsbyfiziks.com | Email: fiziks.physics@gmail.com
28
fiziks
Institute for NET/JRF, GATE, IIT‐JAM, M.Sc. Entrance, JEST, TIFR and GRE in Physics

THERMODYNAMICS AND STATISTICAL PHYSICS SOLUTIONS


GATE 2010
Q1. A system of N non-interacting classical point particles is constrained to move on the two-
dimensional surface of a sphere. The internal energy of the system is
3 1 5
(a) Nk BT (b) Nk BT (c) Nk BT (d) Nk BT
2 2 2
Ans: (c)
Solution: There are 2 N degree of freedom.
Nk BT Nk BT
The internal energy of the system is   Nk BT
2 2
Q2. Which of the following atoms cannot exhibit Bose-Einstein condensation, even in
principle?
(a) 1H1 (b) 4H2 (c) 23Na11 (d) 30K19
Ans: (d)
Solution: For Bose-Einstein condensation:
Number of electron + number of proton + number of neutron = Even
For 30 K19
Number of proton = 19, Number of electron = 19, Number of neutron = 11.
19 + 19 + 11 = 49 this is odd. So it will not exhibit Bose-Einstein condensation.
Q3. For a two-dimensional free electron gas, the electronic density n, and the Fermi energy EF,
are related by

 2mEF 
3/ 2
mE F
(a) n (b) n 
3 
2 3
 2

 2mEF 
3/ 2
mE F
(c) n  (d) n
2 2 
Ans: (c)
EF
2m
Solution: n   g ( E ) f ( E )dE ,
0
g ( E )dE 
h2
dE

1, if E  EF 2mE F mE
At T=0 , f  E    n  2 F2
0, if E  EF h 2
2 

H.No. 40-D, Ground Floor, Jia Sarai, Near IIT, Hauz Khas, New Delhi-110016
Phone: 011-26865455/+91-9871145498
Website: www.physicsbyfiziks.com | Email: fiziks.physics@gmail.com
1
fiziks
Institute for NET/JRF, GATE, IIT‐JAM, M.Sc. Entrance, JEST, TIFR and GRE in Physics

Q4. Which among the following sets of Maxwell relations is correct? (U-internal energy,
H-enthalpy, A-Helmholtz free energy and G-Gibbs free energy)
 U   U   H   H 
(a) T    and P    (b) V    and T   
 V  S  S V  P  S  S  P

 G   G   A   A 
(c) P    and V    (d) P    and S   
 V T  P  S  S T  P V
Ans: (b)
 H   H 
Solution: dH  TdS  VdP     T,  V
 S  P  P  S

 2V  k BT 
3

Q5. Partition function for a gas of photons is given as, ln Z  . The specific heat
453C 3
of the photon gas varies with temperature as
(a) (b)
CV CV

T T
(c) (d)

CV CV

T T
Ans: (a)
 ln z  U 
Solution: U  K B T 2 , CV     CV  T .
3

T  T  v
Q6. From Q. no. 5, the pressure of the photon gas is
 2 k B T 3  2 k B T 4  2 k B T 4  2 k B T 3 / 2
(a) (b) (c) (d)
15 3 C 3 8 3 C 3 45 3 C 3 45 3 C 3
Ans: (c)

  ln z    k0T 
4
F
2

Solution: Since, P    P  KT   
V  V T 453C 3
H.No. 40-D, Ground Floor, Jia Sarai, Near IIT, Hauz Khas, New Delhi-110016
Phone: 011-26865455/+91-9871145498
Website: www.physicsbyfiziks.com | Email: fiziks.physics@gmail.com
2
fiziks
Institute for NET/JRF, GATE, IIT‐JAM, M.Sc. Entrance, JEST, TIFR and GRE in Physics

GATE 2011

Q7. A Carnot cycle operates on a working substance between two reservoir at temperatures T1
and T2 with T1 > T2. During each cycle, an amount of heat Q1 is extracted from the
reservoir at T1 and an amount Q2 is delivered in the reservoir at T2. Which of the
following statements is INCORRECT?
(a) Work done in one cycle is Q1 – Q2
Q1 Q2
(b) 
T1 T2

(c) Entropy of the hotter reservoir decreases


(d) Entropy of the universe (consisting of the working substance and the two reservoirs)
increases
Ans: (c)
Solution: Entropy of hotter reservoirs decreases.
Q8. In a first order phase transition, at the transition temperature, specific heat of the system
(a) diverges and its entropy remains the same
(b) diverges and its entropy has finite discontinuity
(c) remains unchanged and its entropy has finite discontinuity
(d) has finite discontinuity and its entropy diverges
Ans: (b)
Q9. A system of N non-interacting and distinguishable particle of spin 1 is in thermodynamic
equilibrium. The entropy of the system is
(a) 2kB ln N (b) 3kB ln N (c) NkB ln 2 (d) NkB ln 3
Ans: (d)
Solution: S  k B  ln  ,  =3 is number of microstate. S  1; Sz  1, 0, 1
i

The entropy of the system is NkB ln 3.

H.No. 40-D, Ground Floor, Jia Sarai, Near IIT, Hauz Khas, New Delhi-110016
Phone: 011-26865455/+91-9871145498
Website: www.physicsbyfiziks.com | Email: fiziks.physics@gmail.com
3
fiziks
Institute for NET/JRF, GATE, IIT‐JAM, M.Sc. Entrance, JEST, TIFR and GRE in Physics

Q10. A system has two energy levels with energies ε and 2ε. The lower level is 4-fold
degenerate while the upper level is doubly degenerate. If there are N non-interacting
classical particles in the system, which is in thermodynamic equilibrium at a temperature
T, the fraction of particles in the upper level is
1 1
(a) (b)
1  e  / k BT 1  2e  / k BT
1 1
(c)  / k BT
(d)
2e  4e 2  / k B T 2e  / k BT
 4e 2  / k B T
Ans: (b)
2e2/ kT 1
Solution: Partition function Z  4e / kT  2e / kT  P  2   / kT 2/ kT

4e  2e 1  2e/ kT
GATE 2012

Q11. The isothermal compressibility,  of an ideal gas at temperatures T0 and V0 is given by

1 V 1 V P P
(a)  (b) (c)  V0 (d) V0
V0 P T0 V0 P T0 V T0 V T0

Ans: (c)
 P 
Solution: Isothermal compressibility   V  
 V T
Q12. For an ideal Fermi gas in three dimensions, the electron velocity VF at the Fermi surface
is related to electron concentration n as,
(a) VF  n 2 / 3 (b) VF  n (c) VF  n1 / 2 (d) V F  n1 / 3
Ans: (d)
1
Solution: E F  mV F2  E F  n 2 / 3  VF2  n 2 / 3  VF  n1 / 3 .
2
Q13. A classical gas of molecules, each of mass m, is in thermal equilibrium at the absolute
temperature T. The velocity components of the molecules along the Cartesian axes are
v x , v y and v z . The mean value of v x  v y  is
2

k BT 3 k BT 1 k BT 2k B T
(a) (b) (c) (d)
m 2 m 2 m m
Ans: (d)

H.No. 40-D, Ground Floor, Jia Sarai, Near IIT, Hauz Khas, New Delhi-110016
Phone: 011-26865455/+91-9871145498
Website: www.physicsbyfiziks.com | Email: fiziks.physics@gmail.com
4
fiziks
Institute for NET/JRF, GATE, IIT‐JAM, M.Sc. Entrance, JEST, TIFR and GRE in Physics

Solution: Vx  Vy 
2k B T
 vx2  v y2  2 vx  v y  vx2  v y2  2 vx    v y 
2

m
2 k BT
 vx    v y  0 and Vx2  Vy2  .
m
Q14. The total energy, E of an ideal non-relativistic Fermi gas in three dimensions is given by
N 5/3
E , where N is the number of particles and V is the volume of the gas. Identify the
V 2/3
CORRECT equation of state (P being the pressure),
1 2 5
(a) PV  E (b) PV  E (c) PV  E (d) PV  E
3 3 3
Ans: (b)
5 5
 E  2  N 3 2N 3 2
Solution: P         PV   E.
 V  N 3  V  3 V23 3
Q15. Consider a system whose three energy levels are given by 0, ε and 2ε. The energy level ε
is two-fold degenerate and the other two are non-degenerate. The partition function of the
1
system with   is given by
k BT

(a) 1  2e   (b) 2e    e 2  (c) (1  e   ) 2 (d) 1  e    e 2 


Ans: (c)
Solution: E1  0, E 2   , E 3  2 ; g1  1, g 2  2, g 3  1 where g1 , g 2 and g 3 are degeneracy.


The partition function Z  g1e   E1  g 2 e   E2  g 3 e   E3  1  2e    e   2  1  e   
2

GATE 2013

Q16. If Planck’s constant were zero, then the total energy contained in a box filled with
radiation of all frequencies at temperature T would be ( k is the Boltzmann constant and
T is nonzero)
3
(a) zero (b) Infinite (c) kT (d) kT
2
Ans: (d)
Solution: If Planck’s constant were zero, then the system behaved as a classical system and the
energy is kT .
H.No. 40-D, Ground Floor, Jia Sarai, Near IIT, Hauz Khas, New Delhi-110016
Phone: 011-26865455/+91-9871145498
Website: www.physicsbyfiziks.com | Email: fiziks.physics@gmail.com
5
fiziks
Institute for NET/JRF, GATE, IIT‐JAM, M.Sc. Entrance, JEST, TIFR and GRE in Physics

Q17. Across a first order phase transition, the free energy is


(a) proportional to the temperature
(b) a discontinuous function of the temperature
(c) a continuous function of the temperature but its first derivative is discontinuous
(d) such that the first derivative with respect to temperature is continuous
Ans: (c)
Q18. Two gases separated by an impermeable but movable partition are allowed to freely
exchange energy. At equilibrium, the two sides will have the same
(a) pressure and temperature (b) volume and temperature
(c) pressure and volume (d) volume and energy
Ans: (a)
Q19. The entropy function of a system is given by S E   aE E0  E  where a and E0 are
positive constants. The temperature of the system is
(a) negative for some energies (b) increases monotonically with energy
(c) decreases monotonically with energy (d) Zero
Ans: (a)
Solution: From first and second law of thermodynamics
S
 dU  PdV       E  U
1 1
TdS  dU  PdV  dS 
T  E V T

 S 
S E   aE E0  E   
1
  a  E0  E   aE  a  E0  2 E   T  .
 E V a  E0  2 E 

Q20. Consider a linear collection of N independent spin ½ particles, each at a fixed location.
The entropy of this system is ( k is the Boltzmann constant)

(d) Nk ln 2 
1
(a) zero (b) Nk (c) Nk
2
Ans: (d)
particle, so entropy is given by Nk ln 2  .
1
Solution: There are two microstates possible for spin
2

H.No. 40-D, Ground Floor, Jia Sarai, Near IIT, Hauz Khas, New Delhi-110016
Phone: 011-26865455/+91-9871145498
Website: www.physicsbyfiziks.com | Email: fiziks.physics@gmail.com
6
fiziks
Institute for NET/JRF, GATE, IIT‐JAM, M.Sc. Entrance, JEST, TIFR and GRE in Physics

Q21. Consider a gas of atoms obeying Maxwell-Boltzmann statistics. The average value of
   
e a p over all the moments p of each of the particles (where a is a constant vector and a
is the magnitude, m is the mass of each atom, T is temperature and k is Boltzmann’s
constant) is,
1 3
 a 2 mkT  a 2 mkT
(a) one (b) zero (c) e 2
(d) e 2

Ans: (c)
  
f  px , p y , pz e p.a dpx dp y dpz where f  px , p y , pz  is Maxwell probability
   
Solution:  e   
p.a

  
distribution at temperature T.
 p x2  p 2y  p z2
    
 Ae  Ae  Ae
py ay
e  
p.a
x
2 mkT
e px ax
dpx y
2 mkT
e dp y z
2 mkT
e pz az dpz
  

 ( a x2  a 2y  a z2 ) mkT  ( px  mkTax )2  ( p y  mkTa y )2  ( pz  mkTa z ) 2


    
e   e  Ae  Ae  Ae
p.a 2
x
2 mkT
dpx y
2 mkT
dp y z
2 mkT
dpz
  

 ( ax2  a 2y  a z2 ) mkT 1
   a 2 mkT
e p .a
e 2
.1.1.1 = e 2

Common Data for Questions 22 and 23: There are four energy levels E , 2E , 3 E and
4 E (where E  0 ). The canonical partition function of two particles is, if these particles
are
Q22. Two identical fermions
(a) e 2 E  e 4 E  e 6 E  e 8 E (b) e 3 E  e 4 E  e 5 E  e 6 E  e 7 E

(c) e  E  e  2 E  e 3 E  e  4 E  (d) e 2 E  e 4 E  e 6 E  e 8 E
2

Ans: (b)
Solution: The possible value of Energy for two Fermions
E1  3E, E2  4 E , E3  5E, E4  6 E , E5  7 E

The partition function is Z  e 3E  e 4 E  2e 5 E  e 6 E  e 7 E , then the answer may


be option (b).
Q23. Two distinguishable particles
(a) e 2 E  e 4 E  e 6 E  e 8 E (b) e 3 E  e 4 E  e 5 E  e 6 E  e 7 E

(c) e  E  e  2 E  e 3 E  e  4 E  (d) e 2 E  e 4 E  e 6 E  e 8 E
2

H.No. 40-D, Ground Floor, Jia Sarai, Near IIT, Hauz Khas, New Delhi-110016
Phone: 011-26865455/+91-9871145498
Website: www.physicsbyfiziks.com | Email: fiziks.physics@gmail.com
7
fiziks
Institute for NET/JRF, GATE, IIT‐JAM, M.Sc. Entrance, JEST, TIFR and GRE in Physics

Ans: (c)
Solution: When two particles are distinguishable then minimum value of Energy is 2 E and
maximum value is 8 E .

So from checking all four options Z  e  E  e 2 E  e 3 E  e 4 E 


2

GATE 2014

Q24. For a gas under isothermal condition its pressure P varies with volume V as P  V 5 / 3 .
The bulk modules B is proportional to
(a) V 1 / 2 (b) V 2 / 3 (c) V 3 / 5 (d) V 5 / 3
Ans: (d)
dP
Solution: P  KV 5 / 3 , B  V B  V 5 / 3
dV
Q25. At a given temperature T , the average energy per particle of a non-interacting gas of
two-dimensional classical harmonic oscillators is _________ k B T

( k B is the Boltzmann constant)


Ans: 2
Q26. Which one of the following is a fermion?
(a)  particle (b) 4 Be 2 nucleus (c) Hydrogen atom (d) deuteron
Ans (d)
Solution: If total number of particles i.e., electron, proton and neutron is odd, then it is a
fermions: P  N  E  3
Q27. For a free electron gas in two dimensions the variations of the density of states. N E  as a
function of energy E , is best represented by

(a) N E  (b) N E 

E E

N E 
(c) (d) N E 

E E

H.No. 40-D, Ground Floor, Jia Sarai, Near IIT, Hauz Khas, New Delhi-110016
Phone: 011-26865455/+91-9871145498
Website: www.physicsbyfiziks.com | Email: fiziks.physics@gmail.com
8
fiziks
Institute for NET/JRF, GATE, IIT‐JAM, M.Sc. Entrance, JEST, TIFR and GRE in Physics

Ans. : (c)
Solution: N E   E 0
Q28. For a system of two bosons each of which can occupy any of the two energy levels 0 and
1
 . The mean energy of the system at temperature T with   is given by
k T

 e     2 e 2   1   e   2 e     e 2    e     2 e 2  
(a) (b) (c) (d)
1  2e     e  2   2e     e  2   2  e    e 2   2  e    e  2  
Ans. : None of the options are matched.
Solution: If both particle will in ground state the energy will 0 , which is non-degenerate. If one
particle is in ground state and other is in first excited state then energy is  and non
degenerate. If both particles will in first excited state, then energy will 2 , which is
non-degenerate.
Then partition function is Z  1  exp      exp  2  

exp    2 exp  2 
Average value of energy 
1  exp    exp  2 
No one answer is correct, but answer may be (a).
Q29. Consider a system of 3 fermions which can occupy any of the 4 available energy states
with equal probability. The entropy of the system is
(a) k B ln 2 (b) 2k B ln 2 (c) 2k B ln 4 (d) 3k B ln 4
Ans: (b)
Solution: Number of ways that 3 fermions will adjust in 4 available energy is 4 C3  4 so

entropy is k B ln 4 = 2k B ln 2
GATE 2015
Q30. In Boss-Einstein condensation, the particles
(a) have strong interparticle attraction
(b) condense in real space
(c) have overlapping wavefunctions
(d) have large and positive chemical potential
Ans.: (c)
Solution: In Bose- Einstein condensates, the particles have overlapping wave function.

H.No. 40-D, Ground Floor, Jia Sarai, Near IIT, Hauz Khas, New Delhi-110016
Phone: 011-26865455/+91-9871145498
Website: www.physicsbyfiziks.com | Email: fiziks.physics@gmail.com
9
fiziks
Institute for NET/JRF, GATE, IIT‐JAM, M.Sc. Entrance, JEST, TIFR and GRE in Physics

Q31. For a black body radiation in a cavity, photons are created and annihilated freely as a
result of emission and absorption by the walls of the cavity. This is because
(a) the chemical potential of the photons is zero
(b) photons obey Pauli exclusion principle
(c) photons are spin-1 particles
(d) the entropy of the photons is very large
Ans.: (a)
Solution: The chemical potential of photon is zero
1
Q32. Consider a system of N non-interacting spin  particles, each having a magnetic
2

moment  , is in a magnetic field B  Bzˆ . If E is the total energy of the system, then
number of accessible microstates  is given by
 E 
 N  !
N! B 
(a)   (b)   
1 E  1 E   E 
 N   !  N  !  N  !
2 B  2  B   B 

1 E  1 E  N!
(c)    N  !  N  ! (d)  
2 B  2  B   E 
 N  !
 B 
Ans.: (a)
1
Solution: Number of microstate is N
Cn1 , where n1 is number of particle in  state and
2
1
n2   N  n1  is number of state in  state.
2
1 E  1 E 
where n1   N   , n2   N 
2 B  2  B 
N
So, number of microstate 
1 E  1 E 
 N   N
2 B  2   B 

H.No. 40-D, Ground Floor, Jia Sarai, Near IIT, Hauz Khas, New Delhi-110016
Phone: 011-26865455/+91-9871145498
Website: www.physicsbyfiziks.com | Email: fiziks.physics@gmail.com
10
fiziks
Institute for NET/JRF, GATE, IIT‐JAM, M.Sc. Entrance, JEST, TIFR and GRE in Physics

Q33. The average energy U of a one dimensional quantum oscillator of frequency  and in
contact with a heat bath at temperature T is given by
1 1  1 1 
(a) U   coth    (b) U   sinh    
2 2  2 2 
1 1  1 1 
(c) U   tanh    (d) U   cosh   
2 2  2 2 
Ans.: (a)
  1
 Ei    n   
 1 1
Solution:  Z   e  e  2
where E   n     Z 
i 0  2    
2sinh  
 2 
 
   1      
U  ln Z   ln   coth  
        2  2 
2sinh  
  2  
Q34. The entropy of a gas containing N particles enclosed in a volume V is given by
 aVE 3 / 2 
S  Nk B ln  , where E is the total energy, a is a constant and k B is the
 N
5/ 2

Boltzmann constant. The chemical potential  of the system at a temperature T is given
by
  aVE 3 / 2  5   aVE 3 / 2  3
(a)    k B T ln    (b)    k B T ln   
  N   N
5/ 2 5/ 2
 2  2

  aVE 3/ 2  5    aVE 3 / 2  3
(c)   k BT ln  3/ 2 
  (d)   k B T ln   
  N   N
3/ 2
 2  2
Ans.: (a)

 G   aVE 3/ 2   aVE 3/ 2 
Solution:     S   Nk B ln  5/ 2 
 S  Nk B ln  5/ 2 
 T  P  N   N 

 aVE 3/ 2 
 G   Nk BT ln  5/ 2 
 ln A
 N 
  
 G    aVE 3/ 2  N 5 / 2  5 / 2  
  aVE 3/ 2  5 
      k BT ln  5/ 2 
 Nk BT . aVE    k BT ln
3/ 2

 N    N  aVE 3 / 2 N 7 / 2    5
 2
  N 2
 
H.No. 40-D, Ground Floor, Jia Sarai, Near IIT, Hauz Khas, New Delhi-110016
Phone: 011-26865455/+91-9871145498
Website: www.physicsbyfiziks.com | Email: fiziks.physics@gmail.com
11
fiziks
Institute for NET/JRF, GATE, IIT‐JAM, M.Sc. Entrance, JEST, TIFR and GRE in Physics

GATE-2016
Q35. The total power emitted by a spherical black body of radius R at a temperature T is P1 .
R
Let P2 be the total power emitted by another spherical black body of radius kept at
2
P1
temperature 2T . The ratio, is _______. (Give your answer upto two decimal places)
P2
Ans.: 0.25

P1 R12T14 R 2T 4 4 1
Solution: P  AT   2 4 
4
   0.25
P2 R2 T2  R  2
16 4
   2T 
4

2
Q36. The entropy S of a system of N spins, which may align either in the upward or in the
downward direction, is given by S   k B N  p ln p  1  p  In(1  p)  . Here k B is the

Boltzmann constant. The probability of alignment in the upward direction is p. The value
of p, at which the entropy is maximum, is _______. (Give your answer upto one decimal
place)
Ans.: 0.5
Solution: S   k B N  p ln p  1  p  In(1  p) 

dS 1 1
For maximum entropy,  0  ln p  p   ln 1  p   1  p    1  0
dp p 1 p

 p 
ln p  1  ln 1  p   1  0  ln    0  p  1  p  p  0.5
 1 p 
Q37. For a system at constant temperature and volume, which of the following statements is
correct at equilibrium?
(a) The Helmholtz free energy attains a local minimum.
(b) The Helmholtz free energy attains a local maximum.
(c) The Gibbs free energy attains a local minimum.
(d) The Gibbs free energy attains a local maximum.
Ans.: (a)
Solution: dF   SdT  PdV

H.No. 40-D, Ground Floor, Jia Sarai, Near IIT, Hauz Khas, New Delhi-110016
Phone: 011-26865455/+91-9871145498
Website: www.physicsbyfiziks.com | Email: fiziks.physics@gmail.com
12
fiziks
Institute for NET/JRF, GATE, IIT‐JAM, M.Sc. Entrance, JEST, TIFR and GRE in Physics

Q38. N atoms of an ideal gas are enclosed in a container of volume V . The volume of the
container is changed to 4V , while keeping the total energy constant. The change in the
entropy of the gas, in units of Nk B ln 2 , is _______, where k B is the Boltzmann constant.
Ans.: 2
1
Solution: S1   Nk B ln1, S2   Nk B ln  S  S2  S1  Nk B ln 4  2 Nk B ln 2
4
Q39. Consider a system having three energy levels with energies 0, 2 and 3 ,with
respective degeneracies of 2,2 and 3 . Four bosons of spin zero have to be
accommodated in these levels such that the total energy of the system is 10  . The
number of ways in which it can be done is ______.
Ans.: 18
Solution: The system have energy 10  , if out of four boson two boson are in energy level
2 and two boson are in energy level 3 and
ni  gi  1
W  , n1  2, g1  2 and n2  2, g 2  3
i ni gi  1

2  2 1 2  3 1
W   3  6  18
2 2 1 2 3 1
Q40. A two-level system has energies zero and E . The level with zero energy is non-
degenerate, while the level with energy E is triply degenerate. The mean energy of a
classical particle in this system at a temperature T is
E E E E
k BT k BT k BT k BT
Ee Ee 3Ee 3Ee
(a) E
(b) E
(c) E
(d) E

1  3e k BT
1 e k BT
1 e k BT
1  3e k BT

Ans.: (d)
Ei
 0 E E
 gi Ei e kT  
k BT
0e kT  3 E  e kT 3Ee
Solution: E  i
E
 0 E
 E
 i  
 gi e kT e kT  3 e kT
1  3e k BT
i

H.No. 40-D, Ground Floor, Jia Sarai, Near IIT, Hauz Khas, New Delhi-110016
Phone: 011-26865455/+91-9871145498
Website: www.physicsbyfiziks.com | Email: fiziks.physics@gmail.com
13
fiziks
Institute for NET/JRF, GATE, IIT‐JAM, M.Sc. Entrance, JEST, TIFR and GRE in Physics

GATE 2017
Q41. Consider a triatomic molecule of the shape shown in the figure in three
dimensions. The heat capacity of this molecule at high temperature
(temperature much higher than the vibrational and rotational energy scales
of the molecule but lower than its bond dissociation energies) is:
3 9
(a) kB (b) 3k B (c) kB (d) 6k B
2 2
Ans. : (d)
Solution: If given molecules are at lower temperature i.e. atoms are attached to rigid rod then
6 k BT
degree of freedom is 6 , so internal energy is , but at high temperature, vibration
2
mode will active, so there are three extra vibration mode will active, so total energy
U  3k BT  3k BT  6k BT

 U 
CV     6k B
 T V

Q42. A reversible Carnot engine is operated between temperatures T1 and T2 T2  T1  with a

photon gas as the working substance. The efficiency of the engine is


3/ 4 4/3
3T T T  T 
(a) 1  1 (b) 1  1 (c) 1   1  (d) 1   1 
4T2 T2  T2   T2 
Ans. : (b)
Solution: Efficiency of Carnot engine does not depends on nature of working substance rather
depends on temperature of source and sink
T1
  1
T2

Q43. Water freezes at 00 C at atmospheric pressure 1.01105 Pa  . The densities of water and

ice at this temperature and pressure are 1000 kg / m3 and 934 kg / m3 respectively. The

latent heat of fusion is 3.34 105 J / kg . The pressure required for increasing the melting

temperature of ice by 100 C is…………… GPa . (up to two decimal places)


Ans. : 0.01102

H.No. 40-D, Ground Floor, Jia Sarai, Near IIT, Hauz Khas, New Delhi-110016
Phone: 011-26865455/+91-9871145498
Website: www.physicsbyfiziks.com | Email: fiziks.physics@gmail.com
14
fiziks
Institute for NET/JRF, GATE, IIT‐JAM, M.Sc. Entrance, JEST, TIFR and GRE in Physics

p2
 dP  L L T2 dT L T
Solution:      dP    P2  P1  ln 2
 dT V T  v2  v1  P1  v2  v1  T1 T  v2  v1  T1
L T
 P2  P1  ln 2  1 105 Pa  0.01 102 GPa
 v2  v1  T1
Q44. Consider N non- interacting, distinguishable particles in a two-level system at
temperature T . The energies of the levels are 0 and  , where   0 . In the high
temperature limit  k BT    , what is the population of particles in the level with energy

?
N N 3N
(a) (b) N (c) (d)
2 4 4
Ans. : (a)

exp 
Solution: P     kT , population of particle in the level with energy  is

1  exp
kT
 
exp exp 
kT kT 1 N
NP     N , for  k BT    , NP     N N 
  11 2
1  exp 1  exp
kT kT
u
Q45. The energy density and pressure of a photon gas are given by u  aT 4 and P  . Where
3
T is the temperature and a is the radiation constant. The entropy per unit volume is given
by  aT 3 . The value of  is…………… (up to two decimal places)
Ans. : 1.33
 S   U 
Solution: TdS  dU  PdV  T     P
 V T  V T

 S  1  U  P aT 4 aT 4 4 3
         aT  1.33
 V T T  V T T T 3T 3

H.No. 40-D, Ground Floor, Jia Sarai, Near IIT, Hauz Khas, New Delhi-110016
Phone: 011-26865455/+91-9871145498
Website: www.physicsbyfiziks.com | Email: fiziks.physics@gmail.com
15
fiziks
Institute for NET/JRF, GATE, IIT‐JAM, M.Sc. Entrance, JEST, TIFR and GRE in Physics

Q46. Consider two particles and two non-degenerate quantum levels 1 and 2. Level 1 always
contains a particle. Hence, what is the probability that level 2 also contains a particle for
each of the two cases:
(i) when the two particles are distinguishable and (ii) when the two particles are bosons?
1 1 1 1
(a) (i) and (ii) (b) (i) and (ii)
2 3 2 2
2 1
(c) (i) and (ii) (d) (i) 1 and (ii) 0
3 2
Ans. : (c)
B A
2
Solution: (I): For distinguishable particle: A B AB , P  2 
3

A 1
(II): For indistinguishable particle (Bosons): , P  2 
A AA 2

GATE-2018

Q47. A microcanonical ensemble consists of 12 atoms with each taking either energy 0 state,
or energy  state. Both states are non-degenerate. If the total energy of this ensemble is
4  , its entropy will be _________ k B (up to one decimal place), where k B is the
Boltzmann constant.
Ans. : 6.204
Solution: The number of ways having total energy 4  , out of 12 atom is
12 12 1110  9
 12C4    495
48 4  3 2

Hence, entropy, S  k B ln w  k B ln  495   k B  6.204   6.204 k B

Q48. An air-conditioner maintains the room temperature at 270 C while the outside temperature
is 470 C . The heat conducted through the walls of the room from outside to inside due to
temperature difference is 7000 W . The minimum work done by the compressor of the
air-conditioner per unit time is__________ W .

H.No. 40-D, Ground Floor, Jia Sarai, Near IIT, Hauz Khas, New Delhi-110016
Phone: 011-26865455/+91-9871145498
Website: www.physicsbyfiziks.com | Email: fiziks.physics@gmail.com
16
fiziks
Institute for NET/JRF, GATE, IIT‐JAM, M.Sc. Entrance, JEST, TIFR and GRE in Physics

Ans. : 466.67
Solution: Q2  W  Q1 T1  47  273  323 k
Q2
Coefficient of performance of refrigerator  AC   Q1
W
T2
Also, coefficient of performance of refrigerator,  W
T1  T2
Q2  7000 J / S
300 7000
  Q2  Heat coming in room 
47  27 W
T2  27  273  300 k
7000  20 1400
W  J /s   466.67 W
300 3
Q49. Two solid spheres A and B have same emissivity. The radius of A is four times the
radius of B and temperature of A is twice the temperature of B . The ratio of the rate of
heat radiated from A to that from B is __________.
Ans. : 256
Rate of heat radiation from solid sphere  A  4 RA2TA4
Solution: 
Rate of heat radiation from solid sphere  B  4 RB2TB4

 RA  4 RB and TA  2TB

4 RA2TA4  4 RB    2TB 
2 4

   16  16  256
4 RB2TB4  RB   TB 
2 4

Q50. The partition function of an ensemble at a temperature T is


N
  
Z   2 cosh 
 k BT 


where k B is the Boltzmann constant. The heat capacity of this ensemble at T  is
kB

X Nk B , where the value of X is __________ (up to two decimal places).

Ans. : 0.42
N
   
Solution: The partition function, z   2 cosh  
  k BT  
  ln z 
The average energy, E  k BT 2
T

H.No. 40-D, Ground Floor, Jia Sarai, Near IIT, Hauz Khas, New Delhi-110016
Phone: 011-26865455/+91-9871145498
Website: www.physicsbyfiziks.com | Email: fiziks.physics@gmail.com
17
fiziks
Institute for NET/JRF, GATE, IIT‐JAM, M.Sc. Entrance, JEST, TIFR and GRE in Physics

       
Nk BT 2  2sinh    2 
  k B T    k BT    
   N  tanh  
    k BT 
2 cosh  
 k BT 
d E      
C   N  sec h 2   . 2 
dT  k BT   k B T 
 N 2
At T  , C  sec h 2 1  Nk sec h 2 1  0.42 Nk B
k k   / k 
2 2

GATE-2019
Q51. Consider a one-dimensional gas of N non-interacting particles of mass m with the
Hamiltonian for a single particle given by
p2 1
H  m 2  x 2  2 x 
2m 2
The high temperature specific heat in units of R  Nk B ( k B is the Boltzmann constant) is

(a) 1 (b) 1.5 (c) 2 (d) 2.5


Ans. : (c)
p2 1 1 NkT NkT
Solution: H   m 2 x 2  m 2 2 x    U0
2m 2 2 2 2

H  NkT

H
CV   NkT
T
Q52. A large number N of ideal bosons, each of mass m , are trapped in a three-dimensional
m 2 r 2
potential V  r   . The bosonic system is kept at temperature T which is much
2
lower than the Bose-Einstein condensation temperature TC . The chemical potential   

satisfies
3 3
(a)    (b) 2    
2 2
(c) 3    2 (d)   3

H.No. 40-D, Ground Floor, Jia Sarai, Near IIT, Hauz Khas, New Delhi-110016
Phone: 011-26865455/+91-9871145498
Website: www.physicsbyfiziks.com | Email: fiziks.physics@gmail.com
18
fiziks
Institute for NET/JRF, GATE, IIT‐JAM, M.Sc. Entrance, JEST, TIFR and GRE in Physics

Ans. : (a)
Q53. At temperature T Kelvin  K  , the value of the Fermi function at an energy 0.5 eV

above the Fermi energy is 0.01 . Then T , to the nearest integer, is __________
( k B  8.62 105 eV / K )

Ans.: 1262
1 1
Solution: F  E    e
E  EF  / k BT
1 
e  E  EF  / k B T
1 F E

  E  EF  / k B T 1 F E  EF  1 F  E  EF
e    ln   T 
F k BT  F   1 F 
k B ln  
 F 
0.5 0.5
 T 
 0.99  8.62  ln  99 
8.62 105 ln  
 0.01 
0.5 105
  1262.3 K
8.62  4.595
Q54. In a thermally insulated container, 0.01 kg of ice at 273 K is mixed with 0.1 kg of
water at 300 K . Neglecting the specific heat of the container, the change in the entropy
of the system in J / K on attaining thermal equilibrium (rounded off to two decimal
places) is____________
Ans. : 1.03
Solution: Teq  290.29 K (Heat gain  Heat lost)

mice L  miceC T  273  m C  300  T 

T  290.29 K
s   sice  swater

mice L T
 s ice   miceC ln i  14.85 J / K
Tice Tice
290.29
 S water  m C ln  13.82 J / K
300
S  1.03 J / K

H.No. 40-D, Ground Floor, Jia Sarai, Near IIT, Hauz Khas, New Delhi-110016
Phone: 011-26865455/+91-9871145498
Website: www.physicsbyfiziks.com | Email: fiziks.physics@gmail.com
19
fiziks
Institute for NET/JRF, GATE, IIT‐JAM, M.Sc. Entrance, JEST, TIFR and GRE in Physics

Q55. Consider two system A and B each having two distinguishable particles. In both the
systems, each particle can exist in states with energies 0,1, 2 and 3 units with equal
probability. The total energy of the combined system is 5 units. Assuming that the
system A has energy 3 units and the system B has energy 2 units, the entropy of the
system is k B ln  . The value of  is__________
Ans. : 12
Solution: EA  3 EB  2
A B 3
A B A B
  4  3  12 2
B A AB
1
S  ln   k B ln12 B A B A
0
  12 .
A  4 B  3

H.No. 40-D, Ground Floor, Jia Sarai, Near IIT, Hauz Khas, New Delhi-110016
Phone: 011-26865455/+91-9871145498
Website: www.physicsbyfiziks.com | Email: fiziks.physics@gmail.com
20
fiziks
Institute for NET/JRF, GATE, IIT‐JAM, M.Sc. Entrance, JEST, TIFR and GRE in Physics

ELECTRONICS SOLUTIONS
GATE-2010
Q1. The voltage resolution of a 12-bit digital to analog converter (DAC), whose output varies
from  10 V to  10 V is, approximately
(a) 1 mV (b) 5 mV (c) 20 mV (d) 100 mV
Ans: (b)
20V
Solution: Voltage resolution=  4.8 mV
212  1
Q2. The figure shows a constant current source charging a capacitor that is initially uncharged.

Vout

If the switch is closed at t = 0, which of the following plots depicts correctly the output
voltage of the circuit as a function of time?
(a) (b)

Vout Vout

t t
(c) (d)

Vout Vout

t t

Ans: (d)
CdV0 I
Solution: I 0   V0  0 t
dt C

H.No. 40-D, Ground Floor, Jia Sarai, Near IIT, Hauz Khas, New Delhi-110016
Phone: 011-26865455/+91-9871145498
Website: www.physicsbyfiziks.com | Email: fiziks.physics@gmail.com
1
fiziks
Institute for NET/JRF, GATE, IIT‐JAM, M.Sc. Entrance, JEST, TIFR and GRE in Physics

Q3. In one of the following circuits, negative feedback does not operate for a negative input.
Which one is it? The opamps are running from ± 15 V supplies.
(a) (b)
 

 

(c) (d)
 

 

Ans: (c)
Q4. For any set of inputs, A and B, the following circuits give the same output, Q, except one.
Which one is it?
(a)  (b) 
Q
 Q 

(c)  (d) 
Q
 Q 

Ans. : (d)

H.No. 40-D, Ground Floor, Jia Sarai, Near IIT, Hauz Khas, New Delhi-110016
Phone: 011-26865455/+91-9871145498
Website: www.physicsbyfiziks.com | Email: fiziks.physics@gmail.com
2
fiziks
Institute for NET/JRF, GATE, IIT‐JAM, M.Sc. Entrance, JEST, TIFR and GRE in Physics

GATE-2011
Q5. Which of the following statements is CORRECT for a common emitter amplifier circuit?
(a) The output is taken from the emitter
(b) There is 180o phase shift between input and output voltages
(c) There is no phase shift between input and output voltages
(d) Both p-n junctions are forward biased
Ans: (b)
Q6. For an intrinsic semiconductor, me* and mh* are respectively the effective masses of
electrons and holes near the corresponding band edges. At a finite temperature the
position of the Fermi level
(a) depends on me* but not on mh* (b) depends on mh* but not on me*
(c) depends on both me* and mh* (d) depends neither on me* nor on mh*
Ans: (c)
Q7. In the following circuit, the voltage across and the current through the 2 kΩ resistance are

500 1k

20V 10V
2k
30V

(a) 20 V, 10 mA (b) 20 V, 5 mA (c) 10 V, 10 mA (d) 10 V, 5 mA


Ans: (d)
Q8. In the following circuit, Tr1 and Tr2 are identical transistors having VBE = 0.7 V. The
current passing through the transistor Tr2 is

100
Tr2
5V

Tr1

(a) 57 mA (b) 50 mA (c) 48 mA (d) 43 mA

H.No. 40-D, Ground Floor, Jia Sarai, Near IIT, Hauz Khas, New Delhi-110016
Phone: 011-26865455/+91-9871145498
Website: www.physicsbyfiziks.com | Email: fiziks.physics@gmail.com
3
fiziks
Institute for NET/JRF, GATE, IIT‐JAM, M.Sc. Entrance, JEST, TIFR and GRE in Physics

Ans: (d)
5  0.7
Solution: Current through 100 , I   43 mA
100
I  I C  2 I B  I C  43 mA .
Q9. Consider the following circuit 1k 4k
 10V

 Vout
Vin
 10V
Which of the following correctly represents the output Vout corresponding to the input Vin?
 5V  5V
(a) (b)
 2V  2V
Vin Vin
 2V time  2V time
 5V  5V

 10V
 10V Vout
Vout time
- 10V
time
- 10V

 5V  5V
(c) (d)
 2V  2V
Vin Vin
 2V time  2V time
 5V  5V

 10V
 10V Vout
Vout time
- 10V
time
- 10V

Ans: (a)
 1   1 
Solution: Vut     10  2V , Vlt     10  2V .
1 4  1 4 

H.No. 40-D, Ground Floor, Jia Sarai, Near IIT, Hauz Khas, New Delhi-110016
Phone: 011-26865455/+91-9871145498
Website: www.physicsbyfiziks.com | Email: fiziks.physics@gmail.com
4
fiziks
Institute for NET/JRF, GATE, IIT‐JAM, M.Sc. Entrance, JEST, TIFR and GRE in Physics

Q10. The following Boolean expression


Y  A  B  C  D  A  B  C  D  A  B  C  D  A  B  C  D  A  B  C  D  A  B  C  D can
be simplified to
(a) A  B  C  A  D (b) A  B  C  A  D
(c) A  B  C  A  D (d) A  B  C  A  D
Ans: (c)
CD CD CD CD
AD
AB 1 1
AB 1 1
AB
AB 1 1

AB C
GATE-2012
Q11. If the peak output voltage of a full wave rectifier is 10 V, its d.c. voltage is
(a) 10.0 V (b) 7.07 V (c) 6.36 V (d) 3.18 V
Ans: (c)
2Vm 2  10 14  10 70
Solution: Vdc      6.36V
 22 / 7 22 11
Q12. A Ge semiconductor is doped with acceptor impurity concentration of 1015 atoms/cm3.
For the given hole mobility of 1800 cm2/V-s, the resistivity of the material is
(a) 0.288 Ω cm (b) 0.694 Ω cm (c) 3.472 Ω cm (d) 6.944 Ω cm
Ans: (c)
1 1 1
Solution:     15  3.47 cm
 N A e u h 10  1.6  10 19  1800

H.No. 40-D, Ground Floor, Jia Sarai, Near IIT, Hauz Khas, New Delhi-110016
Phone: 011-26865455/+91-9871145498
Website: www.physicsbyfiziks.com | Email: fiziks.physics@gmail.com
5
fiziks
Institute for NET/JRF, GATE, IIT‐JAM, M.Sc. Entrance, JEST, TIFR and GRE in Physics

Q13. Identify the CORRECT energy band diagram for silcon doped with Arsenic. Here CB,
VB, ED and EF are conduction band, valence band, impurity level and Fermi level,
respectively.
(a) (b)
CB CB
ED
ED
EF
EF

VB VB

(c) (d)
CB CB

EF EF
ED ED
VB VB

Ans: (b)
Solution: N-type material ( Si doped with AS ).

Q14. In the following circuit, for the output voltage to be V0   V1  V2 / 2  the ratio R1/R2 is
(a) 1/2 R

(b) 1
 VCC
(c) 2 R
V1 
(d) 3
Vo
Ans: (d) 
V2
R1
Solution: When V2  0, v 01  V1 - VCC
R2

 R   R2 
when V1  0, v02  1     V2
 R   R1  R2 
V2 R2 1 R
Since V0  V1   2   1 3
2 R1  R2 2 R2

H.No. 40-D, Ground Floor, Jia Sarai, Near IIT, Hauz Khas, New Delhi-110016
Phone: 011-26865455/+91-9871145498
Website: www.physicsbyfiziks.com | Email: fiziks.physics@gmail.com
6
fiziks
Institute for NET/JRF, GATE, IIT‐JAM, M.Sc. Entrance, JEST, TIFR and GRE in Physics

Q15. Consider the following OP-AMP circuit.  10V


Which one of the following correctly represents the output Vout
Vin 
corresponding to the input Vin?
5V Vout

4k
- 10V
1k
(a)  5V (b)  5V
Vin Vin
 1V  1V
0V t 0V t

 10V  10V
Vout Vout

t t

 10V  10V

 5V  5V
(c) Vin (d) Vin

0V t 0V t

 10V  10V
Vout Vout

t t

 10V  10V
Ans: (a)
 1 
Solution: Voltage at inverting input V2     5  1V .
1 4 
When vin  1V , v 0  VCC and when vin  1V , v0  VCC

H.No. 40-D, Ground Floor, Jia Sarai, Near IIT, Hauz Khas, New Delhi-110016
Phone: 011-26865455/+91-9871145498
Website: www.physicsbyfiziks.com | Email: fiziks.physics@gmail.com
7
fiziks
Institute for NET/JRF, GATE, IIT‐JAM, M.Sc. Entrance, JEST, TIFR and GRE in Physics

Q16. Consider the following circuit in which the current gain βdc of the transistor is 100.
 15 V

100 k 900 

100 

Which one of the following correctly represents the load line (collector current IC with
respect to collector-emitter voltage VCE) and Q-point of this circuit?

15 mA Q - point 13 mA Q - point
(a) (2 V, 13 mA) (b) (2 V, 10 mA)
IC →

IC →

VCE  15 V VCE  15 V

15 mA 13 mA
(c) (d)
Q - point Q - point
(7.5 V, 7.5 mA) (7.5 V, 6.5 mA)
IC →

IC →

VCE  15 V VCE  15 V
Ans: (a)
VCC  V BE 15  0.7 14.3
Solution: I B    mA.
RB  RE 100  10  100 100
3

I C  I B  14.3mA  13mA , VCE  VCC  I C RC  RE   15  900  100  13  10 3  2V .

VCC 15
I C , Sat    15 mA.
R C  R E 1000

H.No. 40-D, Ground Floor, Jia Sarai, Near IIT, Hauz Khas, New Delhi-110016
Phone: 011-26865455/+91-9871145498
Website: www.physicsbyfiziks.com | Email: fiziks.physics@gmail.com
8
fiziks
Institute for NET/JRF, GATE, IIT‐JAM, M.Sc. Entrance, JEST, TIFR and GRE in Physics

Q17. In the following circuit, the voltage drop across the ideal diode in forward bias condition
is 0.7V. The current passing through the diode is
(a) 0.5 mA
12k
(b) 1.0 mA
 24 Volt
(c) 1.5 mA

(d) 2.0 mA
6k 3.3 k

Ans: (b)
Solution: Let current through 12k is I and through diode is I D
Then 0 .7  I D  3 .3   I  I D   6 (1)
and  24  I  12  I  I D   6  0 (2)
From (1) and (2) I D  1mA.
GATE-2013
Q18. What should be the clock frequency of a 6  bit A / D converter so that its maximum
conserved time is 32s ?
(a) 1 MHz (b) 2 MHz (c) 0.5 MHz (d) 4 MHz
Ans: (c)
Q19. A phosphorous doped silicon semiconductor (doping density: 1017/cm3) is heated from
100C to 200C. Which one of the following statements is CORRECT?
(a) Position of Fermi level moves towards conduction band
(b) Position of dopant level moves towards conduction band
(c) Position of Fermi level moves towards middle of energy gap
(d) Position of dopant level moves towards middle of energy gap
Ans: (c)

H.No. 40-D, Ground Floor, Jia Sarai, Near IIT, Hauz Khas, New Delhi-110016
Phone: 011-26865455/+91-9871145498
Website: www.physicsbyfiziks.com | Email: fiziks.physics@gmail.com
9
fiziks
Institute for NET/JRF, GATE, IIT‐JAM, M.Sc. Entrance, JEST, TIFR and GRE in Physics

Statement for Linked Answer Questions 20 and 21:


Consider the following circuit
Q20. For this circuit the frequency above which the gain will decrease by 20 dB per decade is
(a) 15.9 kHz (b) 1.2kHz 10 k 
Vin  
(c) 5.6 kHz (d) 22.5 kHz Vout 
1000pF 
Ans: (a)
1 1 k
Solution: f H   16kHz
2RC

2 k
Q21. At 1.2 kHz the closed loop gain is
(a) 1 (b) 1.5 (c) 3 (d) 0.5
1  R F 
v0  R1 

Ans: (b)   1 .5
v in 2
1   f 

 fH 
GATE-2014
Q22. The input given to an ideal OP-AMP integrator circuit is
V

V0

t
t0
The correct output of the integrator circuit is
(a) V (b) V

V0 V0

t0 t t0 t

(c) V (d) V

V0
V0
t0 t
t0 t
Ans: (a)
H.No. 40-D, Ground Floor, Jia Sarai, Near IIT, Hauz Khas, New Delhi-110016
Phone: 011-26865455/+91-9871145498
Website: www.physicsbyfiziks.com | Email: fiziks.physics@gmail.com
10
fiziks
Institute for NET/JRF, GATE, IIT‐JAM, M.Sc. Entrance, JEST, TIFR and GRE in Physics

Q23. The minimum number of flip-flops required to construct a mod-75 counter is


__________
Ans: 7
Q24. The donor concentration in a sample of n -type silicon is increased by a factor of 100.
The shift in the position of the Fermi level at 300K, assuming the sample to non
degenerate is ________ meV . k BT  25meV at 300 K 
Ans: 115.15
N   Nc   Nc 
Solution: EC  EF  kT ln  c  and EC  EF  kT ln    kT ln    kT ln 100 
 Nd   100 N d   Nd 
Thus shift is E  kT ln 100   25ln 100  meV  115.15 meV

Q25. The current gain of the transistor in the following circuit is  dc  100 . The value of

collector current I C is_________ mA 12V

3k  20  F
V0
150 k 
Vi
20  F

3k 

Ans: 1.6
VCC  VBE 12  0
Solution: I B    0.016 mA  I C   I B  1.6 mA
RB    RC  RE  150  100  3  3

Q26. In order to measure a maximum of 1V with a resolution of 1mV using a n  bit


A
converter working under the principle of ladder network the minimum value of n
D
is________
Ans: 10
1
Solution: 1103   n  10
2 1
n

H.No. 40-D, Ground Floor, Jia Sarai, Near IIT, Hauz Khas, New Delhi-110016
Phone: 011-26865455/+91-9871145498
Website: www.physicsbyfiziks.com | Email: fiziks.physics@gmail.com
11
fiziks
Institute for NET/JRF, GATE, IIT‐JAM, M.Sc. Entrance, JEST, TIFR and GRE in Physics

Q27. A low pass filter is formed by a resistance R and a capacitance C . At the cut-off angular
1
frequency C  the voltage gain and the phase of the output voltage relative to the
RC
input voltage respectively are
(a) 0.71and 45 o (b) 0.71and  45 o (c) 0.5 and  90 o (d) 0.5 and 90 o
Ans: (b)
v0 XC 1 1
Solution:   
 1 1  jCR
vin R  X C R
XC

1 v 1 1 1  j 450
At   C   0    e
RC vin 1 j 2e j 450
2

GATE-2015
Q28. The band gap of an intrinsic semiconductor is E g  0.72 eV and mh*  6mn* . At 300 K , the

Fermi level with respect to the edge of the valence band (in eV ) is at _______(upto three
decimal places) k B  1.38  10 23 JK 1
Ans.: 0.395
Ec  E 3  m* 
Solution: Ei   kT ln  h* 
2 4  mn 

  Ei  Ev  / kT  Ei  Ev  / kT N c  Eg / 2 kT N v Eg / 2 kT
 e i v  
 Eg / 2 kT E  E / kT
ni  NV e  Nc Nv e e  e e
Nv Nc
3
Ei  Ev  N v  Eg  mh*  4 Eg 3 E
 ln     ln  *    Ei  Ev  kT ln  6   g
kT  N c  2kT  me  2kT 4 2

3 0.72
 Ei  Ev   0.026 1.7917   0.3949eV  0.395 eV
4 2
Q29. Which one of the following DOES NOT represent an exclusive OR operation for inputs
A and B ?
(a)  A  B AB (b) AB  BA (c)  A  B A  B  (d)  A  B AB
Ans.: (d)

H.No. 40-D, Ground Floor, Jia Sarai, Near IIT, Hauz Khas, New Delhi-110016
Phone: 011-26865455/+91-9871145498
Website: www.physicsbyfiziks.com | Email: fiziks.physics@gmail.com
12
fiziks
Institute for NET/JRF, GATE, IIT‐JAM, M.Sc. Entrance, JEST, TIFR and GRE in Physics

Solution: (a) ( A  B) AB  ( A  B)( A  B)  AB  AB

(b) AB  AB
(c) AB  AB
(d)  A  B  AB  AB

Q30. Consider the circuit shown in the figure, where RC  1 . For an input signal Vi shown

below, choose the correct V0 from the options:


R
Vi
C
Vi 
V0 1

R

1 2 3 t

V0 V0
(a) (b)
1 1

1 2 3 t 1 2 3 t
1 1

(c) V0 (d) V
0

0 .1
1
1 2 3 t

 0 .1 3 t
1 2

Ans.: (b)
dvi 0  v0 dv dv
Solution: C   v0   RC in   in  vin  v0t
dt R dt dt
vin  t  v0  1V and vin  t  v0  1V

H.No. 40-D, Ground Floor, Jia Sarai, Near IIT, Hauz Khas, New Delhi-110016
Phone: 011-26865455/+91-9871145498
Website: www.physicsbyfiziks.com | Email: fiziks.physics@gmail.com
13
fiziks
Institute for NET/JRF, GATE, IIT‐JAM, M.Sc. Entrance, JEST, TIFR and GRE in Physics

Q31. In the simple current source shown in the figure, Q1 and Q2 are identical transistors with
current gain   100 and VBE  0.7 V Vice  30 V

5 k I0

Q1 Q2

The current I 0 in mA is __________ (upto two decimal places)


Ans.: 5.86
30  0.7 29.3
Solution: VCC  I C RC  VBE  0 , I C    5.86 mA
5 5
Q32. In the given circuit, if the open loop gain A  10 5 the feedback configurations and the
closed loop gain A f are Vi 
V0

9 k
1 k RL

(a) series-shunt, A f  9 (b) series-series, A f  10

(c) series-shunt, A f  10 (d) shunt-shunt, A f  10

Ans.: (c)
 R 
Solution: AF  1  F   1  9   10.
 R1 
Q33. In the given circuit, the voltage across the source resistor is1 V . The drain voltage (in V )
is ___________ 25V

5k 

2 M
500 
Ans.: 15
1 1
Solution: VS  I D RS  I D  A  VD  VDD  I D RD  25   5000  VD  15V
500 500

H.No. 40-D, Ground Floor, Jia Sarai, Near IIT, Hauz Khas, New Delhi-110016
Phone: 011-26865455/+91-9871145498
Website: www.physicsbyfiziks.com | Email: fiziks.physics@gmail.com
14
fiziks
Institute for NET/JRF, GATE, IIT‐JAM, M.Sc. Entrance, JEST, TIFR and GRE in Physics

GATE-2016
Q34. The number density of electrons in the conduction band of a semiconductor at a given
temperature is 2  1019 m 3 . Upon lightly doping this semiconductor with donor
impurities, the number density of conduction electrons at the same temperature
becomes 4  10 20 m 3 . The ratio of majority to minority charge carrier concentration
is________.
Ans : 400
Solution: Intrinsic carrier concentration is ni  2 1019 m 3
Majority carrier concentration is n  4 1020 m 3

ni2  2  10 
19 2

Minority carrier concentration is p    1018 m 3


n 4  10 20

n 4  1020
The ratio of majority to minority charge carrier concentration is   400
p 1018
Q35. For the digital circuit given below, the output X is

X
B
C

(a) A  B.C (b) A . B  C  (c) A . B  C  (d) A  B.C 


Ans.: (b)
Q36. For the transistor shown in the figure, assume V BE  0.7V and  dc  100 . If Vin  5V , Vout
(in Volts) is _________. (Give your answer upto one decimal place)
10V

3 k

Vin Vout

200 k 

1k 

Ans.: 5.7

H.No. 40-D, Ground Floor, Jia Sarai, Near IIT, Hauz Khas, New Delhi-110016
Phone: 011-26865455/+91-9871145498
Website: www.physicsbyfiziks.com | Email: fiziks.physics@gmail.com
15
fiziks
Institute for NET/JRF, GATE, IIT‐JAM, M.Sc. Entrance, JEST, TIFR and GRE in Physics

Vin  VBE 5  0.7 4.3


Solution: I B    mA , I C   I B  1.433 mA
RB   RE 200  100 300

Vout  VCC  I C RC  Vout  10  1.433  3  5.7 V

GATE-2017
Q37. The best resolution that a 7 bit A/D convertor with 5V full scale can achieve
is…………… mV . (up to two decimal places)
Ans. : 39.37
5
Solution: Resolution   39.37 mV
2 1
7

Q38. In the figure given below, the input to the primary of the transformer is a voltage varying
sinusoidally with time. The resistor R is connected to the centre tap of the secondary.

Which one of the following plots represents the voltage across the resistor R as a
function of time?

V V
(a) 0 (b) 0
t t

(c) V (d) V
0 0
t t

Ans. : (a)
Solution:
Full wave rectifier with RC filter.

H.No. 40-D, Ground Floor, Jia Sarai, Near IIT, Hauz Khas, New Delhi-110016
Phone: 011-26865455/+91-9871145498
Website: www.physicsbyfiziks.com | Email: fiziks.physics@gmail.com
16
fiziks
Institute for NET/JRF, GATE, IIT‐JAM, M.Sc. Entrance, JEST, TIFR and GRE in Physics

Q39. The minimum number of NAND gates required to construct an OR gate is:
(a) 2 (b) 4 (c) 5 (d) 3
Ans. : (d)
Q40. For the transistor amplifier circuit shown below with R1  10 k , R2  10 k , R3  1 k  ,

and   99 . Neglecting the emitter diode resistance, the input impedance of the amplifier
looking into the base for small ac signal is…………. k  . (up to two decimal places)
VCC

R1
C
Vin B

E Vout
R2 R3

Ans. : 4.75
Solution: Z i  Z b  R where Z b   R3  99k  and R  R1  R2  5k 

 Z i  Z b  R  4.75k 
Q41. Consider an ideal operational amplifier as shown in the figure below with
R1  5 k , R2  1 k , RL  100 k  . For an applied input voltage V  10 mV , the current

passing through R2 is……………..  A . (up to two decimal places)



R1
V
RL
R2

Ans. : 10.0
V 10
Solution: I 2    10 A
R2 1

H.No. 40-D, Ground Floor, Jia Sarai, Near IIT, Hauz Khas, New Delhi-110016
Phone: 011-26865455/+91-9871145498
Website: www.physicsbyfiziks.com | Email: fiziks.physics@gmail.com
17
fiziks
Institute for NET/JRF, GATE, IIT‐JAM, M.Sc. Entrance, JEST, TIFR and GRE in Physics

GATE – 2018
Q42. The logic expression ABC  ABC  ABC  ABC can be simplified to
(a) A XOR C (b) A AND C (c) 0 (d) 1
Ans. : (a)
Solution: Y  ABC  ABC  ABC  ABC  AC  B  B   AC  B  B 

 Y  AC  AC  A XOR C
Q43. In a 2-to-1 multiplexer as shown below, the output X  A0 if C  0 and X  A1 if C  1 .

Which one of the following is the correct implementation of this multiplexer?

A0 A0
(a) C (b) C
X X
A1 A1

A0 A0
(c) C X (d) C X
A1 A1
Ans. : (a)
Solution: Check option (a),
X  A0C  A1C

If C  0  X  A0

If C  1  X  A1
Q44. For an operational amplifier (ideal) circuit shown below,
4k 

2k  10V
V1 
H.No. 40-D, Ground Floor, Jia Sarai, Near IIT, Hauz Khas, New Delhi-110016 V0
V2
Phone: 011-26865455/+91-9871145498 
RL
Website: www.physicsbyfiziks.com5k| 
Email: fiziks.physics@gmail.com
18
fiziks
Institute for NET/JRF, GATE, IIT‐JAM, M.Sc. Entrance, JEST, TIFR and GRE in Physics

If V1  1V and V2  2V , the value of V0 is __________ V (up to one decimal place).

Ans. : 3.6
4 4
Solution: V0  V01  V02   1V   2 V
2 5
V0  2  1.6  3.6V

Q45. A p - doped semiconductor slab carries a current I  100 mA in a magnetic field


B  0.2 T as shown. One measures Vy  0.25 mV and Vx  2 mV . The mobility of holes

in the semiconductor is___________ m 2V 1s 1 (up to two decimal places)

B x
Vy y t  1mm
I z
w  4mm
l  10mm
Vx
Ans. : 1.55
Q46. An n - channel FET having Gate-Source switch-off voltage VGS  OFF  2 V is used to

invert a 0  5V square-wave signal as shown. The maximum allowed value of R would


be _________ k  (up to two decimal places). 5V
5k 
Vout
5V
5V Vin R
0V
0V
1k  100

12V
Ans. : 0.70

H.No. 40-D, Ground Floor, Jia Sarai, Near IIT, Hauz Khas, New Delhi-110016
Phone: 011-26865455/+91-9871145498
Website: www.physicsbyfiziks.com | Email: fiziks.physics@gmail.com
19
fiziks
Institute for NET/JRF, GATE, IIT‐JAM, M.Sc. Entrance, JEST, TIFR and GRE in Physics

GATE-2019
Q47. For the following circuit, what is the magnitude of Vout if Vin  1.5V ?
100 R

R 15V
Vin 
Vout

15V

(a) 0.015V (b) 0.15V (c) 15V (d) 150V


Ans. : (c)
100 R
Solution: Vout   1.5  150 V  V0  15V
R
Q48. Consider the following Boolean expression:

 A  B   A  B  C   A  B  C 
It can be represented by a single three-input logic gate. Identify the gate
(a) AND (b) OR (c) XOR (d) NAND
Ans. : (d)

Solution: Y   A  B   A  B  C    A  B  C 
 

Y   A  B   A   B  C    AB  AC
 

  A  B   A  BC   AB  AC

 A  ABC  AB  BC  AB  AC
 A  ABC  BC  AB  AB  AC
 A  BC  B  AC  A  B  AC

 
  A  AC   B  A  AC  B  A  C  B

Y  ABC
Q49. A 3 - bit analog-to-digital converter is designed to digitize analog signals ranging from
0V to 10V . For this converter, the binary output corresponding to an input of 6 V is
(a) 011 (b) 101 (c) 100 (d) 010

H.No. 40-D, Ground Floor, Jia Sarai, Near IIT, Hauz Khas, New Delhi-110016
Phone: 011-26865455/+91-9871145498
Website: www.physicsbyfiziks.com | Email: fiziks.physics@gmail.com
20
fiziks
Institute for NET/JRF, GATE, IIT‐JAM, M.Sc. Entrance, JEST, TIFR and GRE in Physics

Ans. : (c)
Solution: 0   0 0 0   0V

10
1   0 01   1.42 V
7
20
2   010    2.8V
7
30
3   011   4.28V
7
40
4  10 0    5.71V
7
50
5  101   7.14V
7
60
6  110    8.57 V
7
70
7  111   10V
7
Q50. For the following circuit, the correct logic values for the entries X 2 and Y2 in the truth
table are
C
G A B P C X Y
A X 1 0 1 0 0 0 1
0 0 0 1 0 X2 Y2
G
Y 1 0 0 0 1 0 1
B

(a) 1 and 0 (b) 0 and 0 (c) 0 and 1 (d) 1 and 1


Ans. : (a)

H.No. 40-D, Ground Floor, Jia Sarai, Near IIT, Hauz Khas, New Delhi-110016
Phone: 011-26865455/+91-9871145498
Website: www.physicsbyfiziks.com | Email: fiziks.physics@gmail.com
21
fiziks
Institute for NET/JRF, GATE, IIT‐JAM, M.Sc. Entrance, JEST, TIFR and GRE in Physics

ATOMIC AND MOLECULAR PHYSICS SOLUTIONS

GATE-2010

Q1. To detect trace amounts of gaseous species in a mixture of gases, the preferred probing
tool is
(a) Ionization spectroscopy with X-rays (b) NMR spectroscopy
(c) ESR spectroscopy (d) Laser spectroscopy
Ans: (a)
Q2. A collection of N atoms is exposed to a strong resonant electromagnetic radiation with Ng
atoms in the ground state and Ne atoms in the excited state, such that Ng+Ne=N. This
collection of two-level atoms will have the following population distribution:
(a) N g  N e (b) N g  N e

(c) N g  N e  N / 2 (d) N g  N e  N / 2

Ans: (c)
Solution: In two level lair population inversion is possible to achieve at any power level. The
N
maximum possible situation can be N g  N e 
2
Q3. Two states of an atom have definite parities. An electric dipole transition between these
states is
(a) Allowed if both the sates have even parity
(b) Allowed if both the states have odd parity
(c) Allowed if the two states have opposite parities
(d) Not allowed unless a static electric field is applied
Ans: (c)
Q4. The spectrum of radiation emitted by a black body at a temperature 1000 K peaks in the
(a) Visible range of frequencies (b) Infrared range of frequencies
(c) Ultraviolet range of frequencies (d) Microwave range of frequencies
Ans: (a)

H.No. 40-D, Ground Floor, Jia Sarai, Near IIT, Hauz Khas, New Delhi-110016
Phone: 011-26865455/+91-9871145498
Website: www.physicsbyfiziks.com | Email: fiziks.physics@gmail.com
1
fiziks
Institute for NET/JRF, GATE, IIT‐JAM, M.Sc. Entrance, JEST, TIFR and GRE in Physics

Q5. The three principal moments of inertia of a methanol (CH3OH) molecule have the
property Ix = Iy = I and Iz ≠ I. The rotation energy eigenvalues are
2  2 m12  1 1  2
(a) l l  1     (b) l l  1
2I 2 I  I z I  2I

 2 m12  1 1  2  2 m12  1 1 
(c)    (d) l l  1    
2 I  I z I  2I 2  I z I 
Ans: (a)
Solution: CH 3 OH is example of symmetric rotator, where I x  I y  I z , ( I x  I y  I and I z  I )

The classical expression for energy is E 


1 2
2I

J x  J y2  1 2
2I z
Jz .

This can be expressed in term of J 2  J x2  J y2  J z2 by adding and subtracting J z2

1 2  1 1 
E J     J z2 .
2I  2I z 2I 
2  2 m J2  1 1 
Quantum mechanically E  J J  1    
2I 2  I z I 
Q6. Match the typical spectra of stable molecules with the corresponding wave-number range
1. Electronic spectra (i) 106 cm-1 and above
2. Rotational spectra (ii) 105 – 106 cm-1
3. Molecule dissociation (iii) 108 – 102 cm-1
(a) 1 – ii, 2 – i, 3 – iii (b) 1 – ii, 2 – iii, 3 – i
(b) 1 – iii, 2 – ii, 3 – i (d) 1 – i, 2 – ii, 3 – iii
Ans: (b)

Q7. Consider the operations P : r   r (parity) and T: t → - t (time reversal). For the electric

and magnetic fields E and B , which of the following set of transformations is correct?

(a) P : E   E , B  B; (b) P : E  E , B  B;

T : E  E, B   B T : E  E, B  B

(c) P : E   E , B  B; (d) P : E  E , B   B;

T : E   E, B   B T : E  E, B  B

Ans: (b)

H.No. 40-D, Ground Floor, Jia Sarai, Near IIT, Hauz Khas, New Delhi-110016
Phone: 011-26865455/+91-9871145498
Website: www.physicsbyfiziks.com | Email: fiziks.physics@gmail.com
2
fiziks
Institute for NET/JRF, GATE, IIT‐JAM, M.Sc. Entrance, JEST, TIFR and GRE in Physics

Common Data Questions 8 and 9:


In the presence of a weak magnetic field, atomic hydrogen undergoes the transition:
2
P1/ 2  2 S1/ 2 , by emission of radiation

Q8. The number of distinct spectral lines that are observed in the resultant Zeeman spectrum
is
(a) 2 (b) 3 (c) 4 (d) 6
Ans: (c)
Solution: 2 p1/ 2  2S1/ 2 is sodium D1 lines and it has total 4 zeeman components.
Q9. The spectral line corresponding to the transition
 1  1
2
P1  m j     2 S1/ 2  m j   
2 2  2

is observed along the direction of the applied magnetic field. The emitted electromagnetic
field is
(a) Circularly polarized (b) Linearly polarized
(c) Unpolarized (d) Not emitted along the magnetic field direction
Ans: (a)
 1  1
Solution: For 2 P1 / 2  m j     2 S1 / 2  m j   
 2  2

Here m j  1 gives   component.

In longitudinal observation   is circularly polarized.

GATE-2011

Q10. The population inversion in a two layer material CANNOT be achieved by optical
pumping because
(a) the rate of upward transitions is equal to the rate of downward transitions
(b) the upward transitions are forbidden but downward transitions are allowed
(c) the upward transitions are allowed but downward transitions are forbidden
(d) the spontaneous decay rate of the higher level is very low
Ans: (a)

H.No. 40-D, Ground Floor, Jia Sarai, Near IIT, Hauz Khas, New Delhi-110016
Phone: 011-26865455/+91-9871145498
Website: www.physicsbyfiziks.com | Email: fiziks.physics@gmail.com
3
fiziks
Institute for NET/JRF, GATE, IIT‐JAM, M.Sc. Entrance, JEST, TIFR and GRE in Physics

Q11. A heavy symmetrical top is rotating about its own axis of symmetry (the z-axis). If I1, I2
and I3 are the principal moments of inertia along x, y and z axes respectively, then
(a) I2 = I3; I1 ≠ I2 (b) I1 = I3; I1 ≠ I2 (c) I1 = I2; I1 ≠ I3 (d) I1 ≠ I2 ≠ I3
Ans: (c)
Q12. A neutron passing through a detector is detected because of
(a) the ionization it produces (b) the scintillation light it produces
(c) the electron-hole pairs it produces
(d) the secondary particles produced in a nuclear reaction in the detector medium
Ans: (b)
Q13. An atom with one outer electron having orbital angular momentum l is placed in a weak
magnetic field. The number of energy levels into which the higher total angular
momentum state splits, is
(a) 2l + 2 (b) 2l + 1 (c) 2l (d) 2l – 1
Ans: (b)
Q14. For a multi-electron atom l, L and S specify the one-electron orbital angular momentum,
total orbital angular momentum and total spin angular momentum, respectively. The
selection rules for electric dipole transition between the two electronic energy levels,
specified by l, L and S are
(a) ∆L = 0, ±1; ∆S = 0; ∆l = 0, ±1 (b) ∆L = 0, ±1; ∆S = 0; ∆l = ±1
(c) ∆L = 0, ±1; ∆S = ±1; ∆l = 0, ±1 (d) ∆L = 0, ±1; ∆S = ±1; ∆l = ±1
Ans: (b)
Q15. The lifetime of an atomic state is 1 nanosecond. The natural line width of the spectral line
in the emission spectrum of this state is of the order of
(a) 10-10 eV (b) 10-9 eV (c) 10-6 eV (d) 10-4 eV
Ans: (c)
h 6.625  10 34 J  S 6.625  10 25
Solution: E  h   9
 19
eV  4.14  10 6 eV
t 10 1.6  10
Q16. The degeneracy of an excited state of nitrogen atom having electronic configuration
1s22s22p23d1 is
(a) 6 (b) 10 (c) 15 (d) 150
Ans: (b)

H.No. 40-D, Ground Floor, Jia Sarai, Near IIT, Hauz Khas, New Delhi-110016
Phone: 011-26865455/+91-9871145498
Website: www.physicsbyfiziks.com | Email: fiziks.physics@gmail.com
4
fiziks
Institute for NET/JRF, GATE, IIT‐JAM, M.Sc. Entrance, JEST, TIFR and GRE in Physics

Solution: Degeneracy = 2 (2l + 1)


Q17. The far infrared rotational absorption spectrum of a diatomic molecule shows equilibrium
lines with spacing 20 cm-1. The position of the first Stokes line in the rotational Raman
spectrum of this molecule is
(a) 20 cm-1 (b) 40 cm-1 (c) 60 cm-1 (d) 120 cm-1
Ans: (c)
Solution: Given 2B = 20 cm-1  B = 10 cm-1
The position of the first stokes line in the rotational Raman spectrum = 6B
   6 B  6  10  60 cm 1 .

GATE-2012
Q18. The ground state of sodium atom ( 11 Na ) is a 2 S1 / 2 state. The difference in energy levels
arising in the presence of a weak external magnetic field B, given in terms of Bohr
magnet on,  B , is
(a)  B B (b) 2  B B (c) 4  B B (d) 6  B B
Ans: (b)
Solution: The energy separation in the Zeeman level is E  gM J  B B 

1
For 2 S1 / 2 state; g  2 and M J   . Therefore E1   B B  and E 2   B B  .
2
Thus E  2  B B
Q19. The first Stokes line of a rotational Raman spectrum is observed at 12.96 cm 1 .
Considering the rigid rotor approximation, the rotational constant is given by
(a) 6.48 cm-1 (b) 3.24 cm-1 (c) 2.16 cm-1 (d) 1.62 cm-1
Ans: (c)
Solution: The first Stoke line of the Rotational Raman spectrum lies at = 6B
Thus 6 B  12.96 cm 1  B  2.16 cm 1 .

H.No. 40-D, Ground Floor, Jia Sarai, Near IIT, Hauz Khas, New Delhi-110016
Phone: 011-26865455/+91-9871145498
Website: www.physicsbyfiziks.com | Email: fiziks.physics@gmail.com
5
fiziks
Institute for NET/JRF, GATE, IIT‐JAM, M.Sc. Entrance, JEST, TIFR and GRE in Physics

Q20. Match the typical spectroscopic regions specified in Group I with the corresponding type
of transitions in Group II.
Group I Group II
(P) Infra-red region (i) electronic transitions involving valence electrons
(Q) Ultraviolet-visible region (ii) nuclear transitions
(R) X-ray region (iii) vibrational transitions of molecules
(S) γ-ray region (iv) transitions involving inner shell electrons
(a) (P, i); (Q, iii); (R, ii); (S, iv) (b) (P, ii); (Q, iv); (R, i); (S, iii)
(c) (P, iii); (Q, i); (R, iv); (S, ii) (d) (P, iv); (Q, i); (R, ii); (S, iii)
Ans: (c)
Q21. The term  j1 , j 2 J arising from 2s1 3d 1 electronic in j-j coupling scheme are

1 3  1 5  1 1  1 3
(a)  ,  and  ,  (b)  ,  and  , 
 2 2  2,1  2 2  3, 2  2 2 1, 0  2 2  2,1

1 1  1 5  3 1 1 5 
(c)  ,  and  ,  (d)  ,  and  , 
 2 2 1,0  2 2  3, 2  2 2  2,1  2 2  3, 2
Ans: (a)
Q22. The equilibrium vibration frequency for an oscillator is observed at 2990 cm-1. The ratio
of the frequencies corresponding to the first and the fundamental spectral lines is 1.96.
Considering the oscillator to be anharmonic, the anharmonicity constant is
(a) 0.005 (b) 0.02 (c) 0.05 (d) 0.1
Ans: (b)
2e 1  3xe  1  3xe   0.98  x  0.02 .
Solution:  e 1  2 xe   2990cm 1 and  1.96 
e 1  2 xe  1  2 xe  e

GATE-2013
Q23. The number of spectral lines allowed in the spectrum for the 3 2 D  3 2 P transition in
sodium is _____________.
Ans: 28
Solution: The numbers of Zeeman components for 2D5/2 → 2P3/2 transition = 12
The numbers of Zeeman components for 2D3/2 → 2P3/2 transition = 10
The numbers of Zeeman components for 2D3/2 → 2P1/2 transition = 6

H.No. 40-D, Ground Floor, Jia Sarai, Near IIT, Hauz Khas, New Delhi-110016
Phone: 011-26865455/+91-9871145498
Website: www.physicsbyfiziks.com | Email: fiziks.physics@gmail.com
6
fiziks
Institute for NET/JRF, GATE, IIT‐JAM, M.Sc. Entrance, JEST, TIFR and GRE in Physics

Q24. In a normal Zeeman Effect experiment, spectral splitting of the line at the wavelength
643.8 nm corresponding to the transition 5 1 D2  5 1 P1 of cadmium atoms is to be
observed. The spectrometer has a resolution of 0.01 nm. Minimum magnetic field needed
to observe this is me  9.1  10 31 kg , e  1.6  19 C , c  3  10 8 m / s 

(a) 0.26T (b) 0.52T (c) 2.6T (d) 5.2T


Ans: (b)
Solution: Separation of Zeeman Components
eB 2 2 eB
      
4m c c 4m
4 mc  4  3.14  9.1  1031  3  108 0.01  109
B    0.514T
e 2 1.6  1019  
2
643.8  109
Q25. The spacing between vibrational energy levels in CO molecule is found to
be 8.44  10 2 eV . Given that the reduced mass of CO is 1.14  10 26 kg , Planck’s constant

is 6.626  10 34 Js and 1 eV  1.6  10 19 J . The force constant of the bond in CO
molecule is
(a) 1.87 N/m (b) 18.7 N/m (c) 187 N/m (d) 1870 N/m
Ans: (c)
Solution: The energy of the quantum harmonic oscillator is
 1
E  h  n  , n  0,1,2,........
 2
1 k
The frequency of oscillation is   .
2 
Where k = Spring constant and  = reduced mass
h k
The energy levels are equally spaced with energy separation of E  h 
2 
2 2
 2   2  3.14 
k  E     34
 8.44  10  2  1.6  10 19   1.14  10  26  186.7 N / m
 h   6.626  10 

H.No. 40-D, Ground Floor, Jia Sarai, Near IIT, Hauz Khas, New Delhi-110016
Phone: 011-26865455/+91-9871145498
Website: www.physicsbyfiziks.com | Email: fiziks.physics@gmail.com
7
fiziks
Institute for NET/JRF, GATE, IIT‐JAM, M.Sc. Entrance, JEST, TIFR and GRE in Physics

GATE-2014
Q26. The number of normal Zeeman splitting components of 1 P  1D transition is
(a) 3 (b) 4 (c) 8 (d) 9
Ans: (a)
Solution: This is singlet transition.
Q27. The moment of inertia of a rigid diatomic molecule A is 6 times that of another rigid
diatomic molecule B . If the rotational energies of the two molecules are equal, then the
corresponding values of the rotational quantum numbers J A and J B are
(a) J A  2, J B  1 (b) J A  3, J B  1
(c) J A  5, J B  0 (d) J A  6, J B  1
Ans: (b)
J A  J A  1 I B I
Solution:   B  J A  6, J B  1
J B  J B  1 I A 6 I B

Q28. The value of the magnetic field required to maintain non-relativistic protons of energy
1MeV in a circular orbit of radius 100mm is_______Tesla
(Given: m p  1.67  1027 kg , e  1.6  1019 C )

Ans: 1.44

mv 2 1 2mE
Solution:  qvB, E  mv 2  B   1.44
r 2 qr
Q29. Neutrons moving with speed 10 3 m / s are used for the determination of crystal structure.

If the Bragg angle for the first order diffraction is 30 o the interplannar spacing of the
0
crystal is ______ A . (Given: mn  1.675 1027 kg h  6.626 1034 J .s )
Ans: 4
h 6.62  1034 0
Solution: 2d sin      2d sin 300  27
 d  4A
mv 1.67  10  10 3

Q30. The emission wavelength for the transition D2  F3 is 3122 Ǻ. The ratio of population

of the final to the initial states at a temperature 5000 K is

 h  6.626 10 34


J -s, c  3  108 m / s, k B  1.380  1023 J / K 
(a) 2.03  10 5 (b) 4.02  10 5 (c) 7.02  10 5 (d) 9.83  10 5

H.No. 40-D, Ground Floor, Jia Sarai, Near IIT, Hauz Khas, New Delhi-110016
Phone: 011-26865455/+91-9871145498
Website: www.physicsbyfiziks.com | Email: fiziks.physics@gmail.com
8
fiziks
Institute for NET/JRF, GATE, IIT‐JAM, M.Sc. Entrance, JEST, TIFR and GRE in Physics

Ans: (c)

N F  2 J f  1   kBT 5 9.227641144
hc

Solution:  e  e  7.02 105


NI  i 
2 J  1 7

GATE-2015
Q31. In a rigid rotator of mass M , if the energy of the first excited state is (1 meV ), then the
fourth excited state energy (in meV ) is ____________.
Ans.: 10
Solution:  E  J  J  1 where J  0,1, 2,3..

E4 4  4  1
   E4  10 E1  10 meV , where J  0,1, 2,3..
E1 11  1
Q32. The binding energy per molecule of NaCl (lattice parameter is 0.563 nm ) is 7.956 eV .
K
The repulsive term of the potential is of the form , where K is a constant. The value
r9
of the Modelung constant is ___________ (upto three decimal places)
(Electron charge e  1.6  10 19 C ;  0  8.854  10 12 C 2 N 1 m 2 )
Ans.: 2.80
Solution: The total energy of one ion due to the presence of all others in NaCl crystal is
(considering univalent ions)
Ae 2 K
U r     n , where A is Modelung Constant.
4 0 r r

The potential energy will be minimum at the equilibrium spacing r0 .

 dU   Ae 2 Kn  Ae 2 r0n 1
Thus     n 1 
 0  K 
 dr  r  r0  4 0 r0 r0  4 0 n
2

Thus, Binding energy of molecule or lattice energy is

Ae 2 Ae 2 r0n 1  Ae 2   n  1 
U 0  U r r        
0
4 0 r0 4 0 nr0n  4 0 r0   n 
K
Given repulsive term of the potential is  n  9 
r9
Also binding energy per molecule is U 0  7.95 eV

H.No. 40-D, Ground Floor, Jia Sarai, Near IIT, Hauz Khas, New Delhi-110016
Phone: 011-26865455/+91-9871145498
Website: www.physicsbyfiziks.com | Email: fiziks.physics@gmail.com
9
fiziks
Institute for NET/JRF, GATE, IIT‐JAM, M.Sc. Entrance, JEST, TIFR and GRE in Physics

4 0 r0 n
The Modelung constant is A  U 0   and the lattice parameter is
e 2
n 1
a
a  0.563 nm . Thus the interatomic separation is r0   0.28 nm .
2

 A  7.95 1.67 10 19


J
  
4  3.14  8.85 1012 C 2 N 1m 2  0.282 109 9
1.67 10 
2
19
J 8
7.95  1.67  4  3.14  8.85  0.282  9
 A 102  A  2.80
1.67  8
Q33. Match the phrases in Group I and Group II and identify the correct option.
Group I Group II
(P) Electron spin resonance (ESR) (i) radio frequency
(Q) Nuclear magnetic resonance (NMR) (ii) visible range frequency
(R) Transition between vibrational states of a molecule (iii) microwave frequency
(S) Electronic transition (iv) far-infrared range
(a) (P-i), (Q-ii), (R-iii), (S-iv) (b) (P-ii), (Q-i), (R-iv), (S-iii)
(c) (P-iii), (Q-iv), (R-i), (S-ii) (d) (P-iii), (Q-i), (R-iv), (S-ii)
Ans.: (d)
Solution: (P) Electron spin resonance (ESR) is achieved by Microwave frequency (iii)
(Q): Nuclear magnetic resonance (NMR) is achieved by Radio frequency (i)
(R): Transition between vibrational states of a molecule is achieved by radiation of far
infrared range (iv)
(S): Electronic transition is achieved by visible radiation (ii)
Q34. The excitation wavelength of laser in a Raman effect experiment is 546 nm . If the
Stokes’ line is observed at 552 nm , then the wavenumber of the anti-Stokes’ line (in

cm 1 ) is ___________
Ans.: 18514
Solution: Raman displacement is
1 1 1 1
   AS  0   0  S or     
AS 0 0 S
where  AS , 0 ,  S are wavelength of anti-stoke, exciting & stoke line.
From above relation we can write
H.No. 40-D, Ground Floor, Jia Sarai, Near IIT, Hauz Khas, New Delhi-110016
Phone: 011-26865455/+91-9871145498
Website: www.physicsbyfiziks.com | Email: fiziks.physics@gmail.com
10
fiziks
Institute for NET/JRF, GATE, IIT‐JAM, M.Sc. Entrance, JEST, TIFR and GRE in Physics

1 1 1 1 1 2 1 1 2 S   0 0  S
          AS 
 AS 0 0 S  AS 0 S  AS 0  S 2 S   0

 AS 
 546 10 m  552 10 m 
9 9


546  552
109 m
 2  552 10 9
m  546 10 m  9
558

 AS  540.129 109 m  540.129 107 cm

1 1
Anti-stoke wavenumber is  AS    18514 cm 1
AS 540.129 107 cm

Q35. The number of permitted transitions from 2 P3 / 2  2 S1 / 2 in the presence of a weak


magnetic field is ________________
Ans. : 6
Solution: Zeeman splitting of 2 P3 / 2 and 2 S1/ 2 is shown below
MJ
3/2
2
P3/2 1/2

1/2
3/2
2
S1/ 2 1/2

1/2

The selection rule for Zeeman transactions are


M J  0, 1  0 
 0 if J  0 

There are total six transition in accordance with above selection rules.
GATE-2016
17
Q36. The molecule O2
(a) Raman active but not NMR (nuclear magnetic resonance) active.
(b) Infrared active and Raman active but not NMR active.
(c) Raman active and NMR active.
(d) Only NMR active.
Ans.: (c)
17
Solution: (i) Molecule O2 can not absorb infrared as there is no change in dipole moment
17
during vibration. Thus O2 is infrared inactive.

H.No. 40-D, Ground Floor, Jia Sarai, Near IIT, Hauz Khas, New Delhi-110016
Phone: 011-26865455/+91-9871145498
Website: www.physicsbyfiziks.com | Email: fiziks.physics@gmail.com
11
fiziks
Institute for NET/JRF, GATE, IIT‐JAM, M.Sc. Entrance, JEST, TIFR and GRE in Physics

17
(ii) Molecule O2 shows change in polaraziability during rotation. Thus it is Raman

active molecule.
17 5
(iii) The nucleus of O has spin , therefore it is NMR active.
2
Q37. There are four electrons in the 3d shell of an isolated atom. The total magnetic moment
of the atom in units of Bohr magneton is ________.
Ans.: 0
Solution: The configuration leads to S  2 and L  2
Since it is the case of less than half filled sub shell, thus according to Hund’s rules, lower
J will be in ground state. M L  2 1 0 1 2
4
  e  3d 
J  L  S  0  J   g  J .
 2m 

Thus,   0
Q38. Which of the following transitions is NOT allowed in the case of an atom, according to
the electric dipole radiation selection rule?
(a) 2 s  1s (b) 2 p  1s (c) 2 p  2s (d) 3d  2 p
Ans.: (a)
Solution: In electron dipole transition, l  1 . Thus in transition 2s  1s, l  0 . It violate the
selection rule and hence not allowed.
Q39. The number of spectroscopic terms resulting from the L.S coupling of a 3 p electron and
a 3d electron is _______.
Ans.: 12
1 1
Solution: For 3 p1 3d 1 : s1  , s2   S  0,1
2 2
l1  1, l2  2  L  1, 2,3

S  0, L  1  J  1  Term  1P1

S  0, L  2  J  2  Term  1D2

S  0, L  3  J  3  Term  1F3

S  1, L  1  J  0,1, 2  Terms  3P0 , 3P1 , 3 P2

H.No. 40-D, Ground Floor, Jia Sarai, Near IIT, Hauz Khas, New Delhi-110016
Phone: 011-26865455/+91-9871145498
Website: www.physicsbyfiziks.com | Email: fiziks.physics@gmail.com
12
fiziks
Institute for NET/JRF, GATE, IIT‐JAM, M.Sc. Entrance, JEST, TIFR and GRE in Physics

S  1, L  2  J  1, 2,3  Terms  3D1 , 3D2 , 3 D3

S  1, L  3  J  2,3, 4  Terms  3F2 , 3F3 , 3 F4


Thus total number of spectroscopic terms are 12.

GATE-2017
Q40. The wavefunction of which orbital is spherically symmetric:
(a) px (b) p y (c) s (d) d xy

Ans. : (c)
Solution: For s orbital l  0
0.5 1
Q41. The total energy of an inert-gas crystal is given by E  R    (in eV ), where R
R12 R 6
is the inter-atomic spacing in Angstroms. The equilibrium separation between the atoms
is Angstroms. (up to two decimal places)
Ans. : 1
0.5 1
Solution: Given that E  R   
R12 R 6
For equilibrium separation
dE dE 12  0.5 6 1  6 
0   13
 7  0  6  6  6  0  R  1
dR dR R R R R 
Q42. Which one of the following gases of diatomic molecules is Raman, infrared, and NMR
active?
(a) 1H - 1H (b) 12C - 16O (c) 1H - 35Cl (d) 16O- 16O
Ans. : (c)
Solution: (a) 1H  1H Infrared inactive
(b) 12C  16O NMR Inactive
(c) 1H  35Cl Raman, infrared & NMR active
(d) O  16O Infrared , Raman inactive
16

H.No. 40-D, Ground Floor, Jia Sarai, Near IIT, Hauz Khas, New Delhi-110016
Phone: 011-26865455/+91-9871145498
Website: www.physicsbyfiziks.com | Email: fiziks.physics@gmail.com
13
fiziks
Institute for NET/JRF, GATE, IIT‐JAM, M.Sc. Entrance, JEST, TIFR and GRE in Physics

Q43. Using Hund’s rule the total angular momentum quantum number J for the electronic
ground state of the nitrogen atom is
1 3
(a) (b) (c) 0 (d) 1
2 2
Ans. : (b)
Solution: N : 7 :1s 2 2 s 2 2 p 3

For p 3 : ML  1 0 1
  
2 s 1
 spectral term  LJ  4s3/ 2
Q44. Positronium is an atom made of an electron and a positron. Given the Bohr radius for the
ground state of the Hydrogen atom to be 0.53 Angstroms, the Bohr radius for the ground
state of positronium is…………Angstroms. (up to two decimal places).
Ans. : 1.06
m 
Solution: rn  a0  e 
  
me me m2 m
When    e  e
me  me 2me 2

 rn  2a0  2  0.53  1.06 A0

H.No. 40-D, Ground Floor, Jia Sarai, Near IIT, Hauz Khas, New Delhi-110016
Phone: 011-26865455/+91-9871145498
Website: www.physicsbyfiziks.com | Email: fiziks.physics@gmail.com
14
fiziks
Institute for NET/JRF, GATE, IIT‐JAM, M.Sc. Entrance, JEST, TIFR and GRE in Physics

GATE-2018

Q45. Which one of the following represents the 3 p radial wave function of hydrogen atom?
( a0 is the Bohr radius)
Rr Rr
(a) (b)

0 r / a0 0
r / a0

(c) Rr (d) Rr

0 0 r / a0
r / a0

Ans. : (b)
r
 r   3a0
Solution: 3 p radial wave function is R31 r 1  e
 6a0 
Q46. Given the following table,
Group I Group II
P: Stern-Gerlach experiment 1: Wave nature of particles
Q: Zeeman effect 2: Quantization of energy of electrons in the atoms
R: Frank-Hertz experiment 3: Existence of electron spin
S: Davisson-Germer experiment 4: Space quantization of angular momentum
Which one of the following correctly matches the experiments from Group I to their
inferences in Group II?
(a) P-2, Q-3, R-4, S-1 (b) P-1, Q-3, R-2, S-4
(c) P-3, Q-4, R-2, S-1 (d) P-2, Q-1, R-4, S-3
Ans. : (c)

H.No. 40-D, Ground Floor, Jia Sarai, Near IIT, Hauz Khas, New Delhi-110016
Phone: 011-26865455/+91-9871145498
Website: www.physicsbyfiziks.com | Email: fiziks.physics@gmail.com
15
fiziks
Institute for NET/JRF, GATE, IIT‐JAM, M.Sc. Entrance, JEST, TIFR and GRE in Physics

Q47. The expression for the second overtone frequency in the vibrational absorption spectra of
a diatomic molecule in terms of the harmonic frequency e and anharmonicity constant

xe is

(a) 2e 1  xe  (b) 2e 1  3 xe  (c) 3e 1  2 xe  (d) 3e 1  4 xe 

Ans. : (d)
2
 1  1
Solution:  V  e  v    e xe  v  
 2  2
Second overtone v  0  v  3
2 2
7 7  1
 v   v 3   v 0  e  e xe    e  e xe    3e  12e xe  3we 1  4 xe 
2 2 2 2

Q48. Match the physical effects and order of magnitude of their energy scales given below,
e2
where   is fine structure constant; me and m p are electron and proton mass,
4 0 c
respectively.
Group I Group II
P: Lamb shift 
1: ~ O  2 me c 2 
Q: Fine structure 
2: ~ O  4 me c 2 
R: Bohr energy 
3: ~ O  4 me2 c 2 / m p 
S: Hyperfine structure 
4: ~ O  5 me c 2 
(a) P-3, Q-1, R-2, S-4 (b) P-2, Q-3, R-1, S-4
(c) P-4, Q-2, R-1, S-3 (d) P-2, Q-4, R-1, S-3
Ans. : (c)
Solution:- Bohr energy E   2 me c 2

Fine structure E   4 me c 2

Lamb straight E   5 me c 2

 4 me c 2
Hyperfine structure E 
mp

H.No. 40-D, Ground Floor, Jia Sarai, Near IIT, Hauz Khas, New Delhi-110016
Phone: 011-26865455/+91-9871145498
Website: www.physicsbyfiziks.com | Email: fiziks.physics@gmail.com
16
fiziks
Institute for NET/JRF, GATE, IIT‐JAM, M.Sc. Entrance, JEST, TIFR and GRE in Physics

Q49. The intrinsic/permanent electric dipole moment in the ground state of hydrogen atom is
( a0 is the Bohr radius)

(a) 3ea0 (b) zero (c) ea0 (d) 3ea0


Ans. : (b)
Solution: For dipole moment energy is eEr cos 
E11  eEr cos   eE r cos   0 [ cos   0 ]

Q50. Which one of the following is an allowed electric dipole transition?


(a) 1S0  3 S1 (b) 2P3/ 2  2D5 / 2 (c) 2D5 / 2  2P1/ 2 (d) 3P0  5D0
Ans. : (b)
Solution: For electric dipole transition
L  0, 1  0  0  , J  0.  1 , S  0

Only option (b) satisfies above selection rules


Q51. The term symbol for the electronic ground state of oxygen atom is
(a) 1S0 (b) 1D2 (c) 3P0 (d) 3P2
Ans. : (d)
Solution: O : 1s 2 , 2s 2 , 2 p 4 M L  1 0 1
  
Here, S  1 , L  2
According to Hund’s rule, for ground state energy
J  L  S  2  2 S 1
LJ  3 P2

Q52. 4 MeV  - rays emitted by the de-excitation of 19F are attributed, assuming spherical
symmetry, to the transition of protons from 1d3/ 2 state to 1d5 / 2 state. If the contribution
 
of spin-orbit term to the total energy is written as C l  s , the magnitude of C is ______

MeV (up to one decimal place).


Ans. : 1.6
1 3 5
Solution: l  1, s  , ˆj1  , ˆj2 
2 2 2

   
j   l  s   j  l  s  2l  s  l  s 
2 2 2 
j2  l 2  s2 
2

H.No. 40-D, Ground Floor, Jia Sarai, Near IIT, Hauz Khas, New Delhi-110016
Phone: 011-26865455/+91-9871145498
Website: www.physicsbyfiziks.com | Email: fiziks.physics@gmail.com
17
fiziks
Institute for NET/JRF, GATE, IIT‐JAM, M.Sc. Entrance, JEST, TIFR and GRE in Physics

   j  j  1   l  1  s  s  1   2
l s 
2
    2
E    l  s  l s     5  7  3  5       20   2  20  C
 3/ 2  2 2 2 2 2  
5/ 2
   8  8
20 32
E  C  4MeV  C  MeV , C  1.6 MeV .
8 20
Q53. An atom in its singlet state is subjected to a magnetic field. The Zeeman splitting of its
650 nm spectral line is 0.03 nm . The magnitude of the field is ___________ Tesla (up
to two decimal places).
( e  1.60 1019 C , me  9.111031 kg , c  3.0 108 ms 1 )

Ans. : 1.52
2 eB
Solution:   
c 4 m
c 4 m 3  108 4  9.1 1031
B      
 0.03  109  1.52T
2 e  
19
650 109
2
1.6  10

GATE-2019
Q54. The spin-orbit interaction term of an electron moving in a central field is written as
 
f  r  l  s , where r is the radial distance of the electron from the origin. If an electron

moves inside a uniformly charged sphere, then


(a) f  r   constant (b) f  r   r 1 (c) f  r   r 2 (d) f  r   r 3
Ans. : (a)
Solution: The electric potential of a uniformly charged sphere at r  R is

kQ  r2 
 V
3  
2R  R2 
where Q is the electric charge on the sphere of radius R and k is a constant.
  1  V 
The interaction energy is W  f  r  l  s , where for central potential V , f  r    
r  r 
1  kQr   kQ
 f  r    3   3  constant. Thus option (a) is correct.
r R  R

H.No. 40-D, Ground Floor, Jia Sarai, Near IIT, Hauz Khas, New Delhi-110016
Phone: 011-26865455/+91-9871145498
Website: www.physicsbyfiziks.com | Email: fiziks.physics@gmail.com
18
fiziks
Institute for NET/JRF, GATE, IIT‐JAM, M.Sc. Entrance, JEST, TIFR and GRE in Physics

Q55. The ground state electronic configuration of the rare-earth ion ( Nd 3 ) is  Pd  4 f 3 5s 2 5 p 6 .

8
Assuming LS coupling, the Lande g - factor of this ion is . The effective magnetic
11
moment in units of Bohr magneton  B (rounded off to two decimal places) is
____________
Ans.: 3.62
Solution: For 4 f 3 M L  3 2 1 0 1 2 3 L  6, S  3 / 2, J  9/2

8 99 
  g J  B J  J  1   B    1
11 22 

8 9 11
   B  3.62  B
11 2 2

H.No. 40-D, Ground Floor, Jia Sarai, Near IIT, Hauz Khas, New Delhi-110016
Phone: 011-26865455/+91-9871145498
Website: www.physicsbyfiziks.com | Email: fiziks.physics@gmail.com
19
fiziks
Institute for NET/JRF, GATE, IIT‐JAM, M.Sc. Entrance, JEST, TIFR and GRE in Physics

SOLID STATE PHYSICS SOLUTIONS

GATE-2010

Q1. The valence electrons do not directly determine the following property of a metal
(a) Electrical conductivity (b) Thermal conductivity
(c) Shear modulus (d) Metallic luster
Ans: (c)
Q2. Consider X-ray diffraction from a crystal with a face-centered-cubic (fcc) lattice. The
lattice plane for which there is NO diffraction peak is
(a) (2, 1, 2) (b) (1, 1, 1) (c) (2, 0, 0) (d) (3, 1, 1)
Ans: (a)
Q3. The Hall coefficient, RH, of sodium depends on
(a) The effective charge carrier mass and carrier density
(b) The charge carrier density and relaxation time
(c) The charge carrier density only
(d) The effective charge carrier mass
Ans: (c)

Q4. The Bloch theorem states that within a crystal, the wavefunction, ψ( r ), of an electron has
the form

 
(a)  r  u r e i k .r where u( r ) is an arbitrary function and k is an arbitrary vector

(b) r   u r ei G r
where u( r ) is an arbitrary function and G is a reciprocal lattice vector

(c)  r   u r e i G r
  
where u r  u r   ,  is a lattice vector and G is a reciprocal
lattice vector

    
(d)  r  u r e i k .r where u r  u r   ,  is a lattice vector and k is an arbitrary
vector
Ans: (d)

H.No. 40-D, Ground Floor, Jia Sarai, Near IIT, Hauz Khas, New Delhi-110016
Phone: 011-26865455/+91-9871145498
Website: www.physicsbyfiziks.com | Email: fiziks.physics@gmail.com
1
fiziks
Institute for NET/JRF, GATE, IIT‐JAM, M.Sc. Entrance, JEST, TIFR and GRE in Physics

Q5. In an experiment involving a ferromagnetic medium, the following observations were


made. Which one of the plots does NOT correctly represent the property of the medium?
(TC is the Curie temperature)

(a) (b)

1 / TC 1/ T

(c) (d)

TC T TC T
Ans: (c)
Q6. The thermal conductivity of a given material reduces when it undergoes a transition from
its normal state to the superconducting state. The reason is:
(a) The Cooper pairs cannot transfer energy to the lattice
(b) Upon the formation of Cooper pairs, the lattice becomes less efficient in heat transfer
(c) The electrons in the normal state lose their ability to transfer heat because of their
coupling to the Cooper pairs
(d) The heat capacity increases on transition to the superconducting state leading to a
reduction in thermal conductivity
Ans: (d)
Q7. For a two-dimensional free electron gas, the electronic density n, and the Fermi energy
EF, are related by

2mE F  2 mE F  3
3 1 1
mE F mE F 2 3
(a) n  (b) n  (c) n  (d) n 
3 2  3  2 2 2 
Ans: (b)
Solution: For two dimensional gas, the number of possible k-states between k and k+dk is

H.No. 40-D, Ground Floor, Jia Sarai, Near IIT, Hauz Khas, New Delhi-110016
Phone: 011-26865455/+91-9871145498
Website: www.physicsbyfiziks.com | Email: fiziks.physics@gmail.com
2
fiziks
Institute for NET/JRF, GATE, IIT‐JAM, M.Sc. Entrance, JEST, TIFR and GRE in Physics

2 2
 L   L 
g k dk    2 k dk  2  2 k dk it is multiplied by 2 for electron gas
 2   2 

2mE 2m 2m
Since k 2  2
 2k dk  2 dE  2 k dk  2 dE
  
2
 L  2m
 g E dE  2   2 dE
 2  
The total number of electrons at T  0 0 K is
EF EF 2 EF
2m  L  2m L2
N   g E dE  F E    g E dE  2  2    dE  2  2  2  EF
0 0
  2  0
 4

m  2  N   2 mE F
N  L EF  EF 
2
 2 n  n 
 2
m L  m  2
Q8. Far away from any of the resonance frequencies of a medium, the real part of the
dielectric permittivity is
(a) Always independent of frequency (b) Monotonically decreasing with frequency
(c) Monotonically increasing with frequency (d) A non-monotonic function of frequency
Ans: (a)
dipolar
Solution:
 1 ionic
 2
electronics

frequency

H.No. 40-D, Ground Floor, Jia Sarai, Near IIT, Hauz Khas, New Delhi-110016
Phone: 011-26865455/+91-9871145498
Website: www.physicsbyfiziks.com | Email: fiziks.physics@gmail.com
3
fiziks
Institute for NET/JRF, GATE, IIT‐JAM, M.Sc. Entrance, JEST, TIFR and GRE in Physics

GATE-2011

Q9. The temperature (T) dependence of magnetic susceptibility (χ) of a ferromagnetic


substance with a Curie temperature (Tc) is given by
C C
(a) , for T  Tc (b) , for T  Tc
T  Tc T  Tc

C C
(c) , for T  Tc (d) , for all temperatures
T  Tc T  Tc

where C is constant .
Ans: (b)
Q10. The order of magnitude of the energy gap of a typical superconductor is
(a) 1 MeV (b) 1 KeV (c) 1 eV (d) 1 meV
Ans: (d)
Q11. For a three-dimensional crystal having N primitive unit cells with a basis of p atoms, the
number of optical branches is
(a) 3 (b) 3p (c) 3p – 3 (d) 3N – 3p
Ans: (c)
Q12. For an intrinsic semiconductor, me* and mh* are respectively the effective masses of
electrons and holes near the corresponding band edges. At a finite temperature the
position of the Fermi level
(a) depends on me* but not on mh* (b) depends on mh* but not on me*
(c) depends on both me* and mh* (d) depends neither on me* nor on mh*
Ans: (c)
Ec  Ev 3  mh* 
Solution: The Fermi level for intrinsic semicondutor is E F   k B T ln * 
2 4  me 
Q13. A metal with body centered cubic (bcc) structure show the first (i.e. smallest angle)
diffraction peak at a Bragg angle of θ = 30o. The wavelength of X-ray used is 2.1 Ǻ. The
volume of the PRIMITIVE unit cell of the metal is
(a) 26.2 (Ǻ)3 (b) 13.1(Ǻ)3 (c) 9.3 (Ǻ)3 (d) 4.6 (Ǻ)3
Ans: (b)

H.No. 40-D, Ground Floor, Jia Sarai, Near IIT, Hauz Khas, New Delhi-110016
Phone: 011-26865455/+91-9871145498
Website: www.physicsbyfiziks.com | Email: fiziks.physics@gmail.com
4
fiziks
Institute for NET/JRF, GATE, IIT‐JAM, M.Sc. Entrance, JEST, TIFR and GRE in Physics

a
Solution: According to Bragg’s law 2d sin    where d 
h2  k 2  l 2
For BCC structure the first diffraction peak appear for (110) plane.
a 2a
d   sin 30 0    2a sin 30 0  2.1A 0
2 2
1
 2 a  2.1A 0  a  2  2.1A 0  a  2.97 A 0 .
2

The volume primitive unit cell of BCC is volume 


a 3 26.2 0
2

2
A  
3
 
 13.1 A 0
3

Common Data for Questions 14 and 15:


The tight binding energy dispersion (E-k) relation for electrons in a one-dimensional
array of atoms having lattice constant a and total length L is
E = E0 – β – 2γ cos (ka),
where E0, β and γ are constants and k is the wave vector.
Q14. The density of states of electrons (including spin degeneracy) in the band is given by
L L L L
(a) (b) (c) (d)
 a sin ka  2 a sin ka  2 a coska   a coska 
Ans: (a)
 L  1  L  2 1 2 L
Solution: D  E   2  2    dE / dk  2  2   2a sin ka  2 a sin ka
 2       
Q15. The effective mass of electrons in the band is given by
2 2 2 2
(a) (b) (c) (d)
 a 2 coska  2 a 2 coska   a 2 sin ka  2 a 2 sin ka 
Ans: (b)
2 2 2
Solution: Effective mass m *   
 d 2E  2a 2  coska  2a 2 coska 
 2 
 dk 

H.No. 40-D, Ground Floor, Jia Sarai, Near IIT, Hauz Khas, New Delhi-110016
Phone: 011-26865455/+91-9871145498
Website: www.physicsbyfiziks.com | Email: fiziks.physics@gmail.com
5
fiziks
Institute for NET/JRF, GATE, IIT‐JAM, M.Sc. Entrance, JEST, TIFR and GRE in Physics

GATE-2012
Q16. For an ideal Fermi gas in three dimensions, the electron velocity VF at the Fermi surface
is related to electron concentration n as,
(a) V F  n 2 / 3 (b) V F  n (c) VF  n1 / 2 (d) V F  n1 / 3
Ans: (d)
Solution: V F 

m

3 2 n 
1/ 3

Q17. The total energy, E of an ideal non-relativistic Fermi gas in three dimensions is given by
N 5/3
E  2 / 3 where N is the number of particles and V is the volume of the gas. Identify the
V
CORRECT equation of state (P being the pressure),
1 2 5
(a) PV  E (b) PV  E (c) PV  E (d) PV  E
3 3 3
Ans: (b)
Q18. Which one of the following CANNOT be explained by considering a harmonic
approximation for the lattice vibrations in solids?
(a) Deby’s T3 law (b) Dulong Petit’s law
(c) Optical branches in lattices (d) Thermal expansion
Ans: (d)
Solution: Thermal expansion in solid can only be explained if solid behave as a anharmonic
oscillator.
Q19. A simple cubic crystal with lattice parameter a c undergoes transition into a tetragonal

structure with lattice parameters at  bt  2ac and ct  2a c , below a certain temperature.


The ratio of the interplanar spacing of (1 0 1) planes for the cubic and the tetragonal
structure is

1 1 3 3
(a) (b) (c) (d)
6 6 8 8
Ans: (c)
a ac
Solution: For Cubic Lattice d c  
h2  k 2  l 2 2

a 2a c dc 3
For Tetragonal lattice d t   . Therefore, the ratio is 
h k
2
l 2 2
3 dt 8
2
 2
a c
H.No. 40-D, Ground Floor, Jia Sarai, Near IIT, Hauz Khas, New Delhi-110016
Phone: 011-26865455/+91-9871145498
Website: www.physicsbyfiziks.com | Email: fiziks.physics@gmail.com
6
fiziks
Institute for NET/JRF, GATE, IIT‐JAM, M.Sc. Entrance, JEST, TIFR and GRE in Physics

Q20. Inverse susceptibility (1/χ) as a function of temperature, T for a


1 600 K
material undergoing paramagnetic to ferromagnetic transition is 
given in the figure, where O is the origin. The values of the O T
Curie constant, C, and the Weiss molecular field constant, λ, in  2  10 4
CGS units, are (CGS unit)

(a) C  5  10 5 ,   3  10 2 (b) C  3  10 2 ,   5  10 5

(c) C  3  10 2 ,   2  10 4 (d) C  2  10 4 ,   3  10 2
Ans: (c)
1 T  TC 1
Solution:  and TC  C . Here TC  600 K and  2  10 4
 C 
Thus C  3  10 2 and   2  10 4 .

Common Data for Questions 21–22


The dispersion relation for a one dimensional monoatomic crystal with lattice spacing a,
which interacts nearest neighbour harmonic potential is given by
Ka
  A sin
2
where A is a constant of appropriate unit.
Q21. The group velocity at the boundary of the first Brillouin zone is

Aa 2 1 Aa 2
(a) 0 (b) 1 (c) (d)
2 2 2
Ans: (a)
Solution: At the first Brillouin zone the frequency is maximum and the group velocity which is
the derivative of the angular frequency is zero.
Q22. The force constant between the nearest neighbour of the lattice is (M is the mass of the
atom)
MA 2 MA 2
(a) (b) (c) MA2 (d) 2MA2
4 2

Ans: (a)
4  ka  4 MA2
Solution:   sin    A   
M  2  M 4

H.No. 40-D, Ground Floor, Jia Sarai, Near IIT, Hauz Khas, New Delhi-110016
Phone: 011-26865455/+91-9871145498
Website: www.physicsbyfiziks.com | Email: fiziks.physics@gmail.com
7
fiziks
Institute for NET/JRF, GATE, IIT‐JAM, M.Sc. Entrance, JEST, TIFR and GRE in Physics

GATE-2013
Q23. A phosphorous doped silicon semiconductor (doping density: 1017/cm3) is heated from
100C to 200C. Which one of the following statements is CORRECT?
(a) Position of Fermi level moves towards conduction band
(b) Position of dopant level moves towards conduction band
(c) Position of Fermi level moves towards middle of energy gap
(d) Position of dopant level moves towards middle of energy gap
Ans: (c)
Solution: Phosphorous doped silicon semiconductors behave as a n-type semiconductor. In
n-type semiconductor Fermi level lies near conduction band and moves toward middle of
the band gap upon heating. At a very high temperature the Fermi level is near the middle
of the band gap and semiconductor behaves as intrinsic semiconductor.
Q24. Considering the BCS theory of superconductors, which one of the following statements is
NOT CORRECT? ( h is the Plank’s constant and e is the electronic charge)
(a) Presence of energy gap at temperature below the critical temperature
(b) Different critical temperature for isotopes
h
(c) Quantization of magnetic flux in superconduction ring in the unit of  
e
(d) Presence of Meissner effect
Ans: (c)
 h
Solution: Quantization of magnetic flux in superconduction ring in the unit of  
 2e 
Q25. Group I contains elementary excitations in solids. Group II gives the associated field with
these excitations. MATCH the excitations with their associated field and select your
answer as per codes given below.
Group I Group II
(P) phonon (i) photon + lattice vibration
(Q) plasmon (ii) electron +elastic deformation
(R) polaron (iii) collective electron oscillations
(S) polariton (iv) elastic wave
Codes
(a) P  iv , Q  iii , R  i , S  ii  (b) P  iv , Q  iii , R  ii , S  i 
(c) P  i , Q  iii , R  ii , S  iv  (d) P  iii , Q  iv , R  ii , S  i 

H.No. 40-D, Ground Floor, Jia Sarai, Near IIT, Hauz Khas, New Delhi-110016
Phone: 011-26865455/+91-9871145498
Website: www.physicsbyfiziks.com | Email: fiziks.physics@gmail.com
8
fiziks
Institute for NET/JRF, GATE, IIT‐JAM, M.Sc. Entrance, JEST, TIFR and GRE in Physics

Ans: (b)
Solution: Phonon: Quantum of energy of the elastic wave in solid, produced due to the vibration
of atoms in solid.
Plasmon: Quantum of energy of the wave produced due to the oscillation of plasma,
which contains charged particles (positive ions and negative electrons or ions).
Polaron: A charge placed in a polarizable medium will be screened. The induced
polarization will follow the charge carrier when it is moving through the medium. The
carrier together with the induced polarization is considered as one entity, which is called
a polaron.
Polariton: A polariton is a quasiparticle resulting from the mixing of a photon with
phonon.
Q26.

 
  
 
A lattice has the following primitive vector in Å  : a  2 ˆj  kˆ , b  2 kˆ  iˆ , c  2 iˆ  ˆj .
The reciprocal lattice corresponding to the above lattice is
 
(a) BCC lattice with cube edge of  Å -1
2
(b) BCC lattice with cube edge of 2 Å -1

 
(c) FCC lattice with cube edge of  Å -1
2
(d) FCC lattice with cube edge of 2 Å -1
Ans: (a)
Solution: The reciprocal lattice vectors are
 
 ˆ ˆ ˆ -1
 b c
a  2    

a  b c 
2

-i  j  k Å 
  
 ˆ ˆ ˆ -1
ca
b   2    

a b c 
2

i  j k Å 
 
 ˆ ˆ ˆ -1
 a b
c   2    
 
a  b c 2

i  j k Å 

H.No. 40-D, Ground Floor, Jia Sarai, Near IIT, Hauz Khas, New Delhi-110016
Phone: 011-26865455/+91-9871145498
Website: www.physicsbyfiziks.com | Email: fiziks.physics@gmail.com
9
fiziks
Institute for NET/JRF, GATE, IIT‐JAM, M.Sc. Entrance, JEST, TIFR and GRE in Physics

 e2 B
Q27. The total energy of an ionic solid is given by an expression E    9 where 
4 0 r r
is Madelung constant, r is the distance between the nearest neighbours in the crystal and
B is a constant. If r0 is the equilibrium separation between the nearest neighbours then
the value of B is
 e 2 r08  e 2 r08 2 e 2 r010  e 2 r010
(a) (b) (c) (d)
36 0 4 0 9 0 36 0
Ans: (a)
dE  e2 9B  e 2 r08
Solution: At r  r0 , 0   B 
dr r  r0 4 0 r02 r010 36 0

GATE-2014
Q28. The Miller indices of a plane passing through the three points having coordinates (0, 0, 1)

1, 0, 0  1 , 1 , 1  are
2 2 4
(a) (212) (b) (111) (c) (121) (d) (211)
Ans: (a)
Solution: The equation of plane is determined from following determinant:

x 1 y  0 z  0
 1  1 1   1
1 0 1  0   x  1     y      z  1     0
 2  4 2  2
1 1 1

2 2 4
x y z 1
     0  2 x  y  2 z  2  0 , hx  ky  lz  2  0 . Miller indices are  2 1 2 
2 4 2 2
Q29. The plot of specifies heat versus temperature across the superconducting transition
temperature Tc  is most appropriately represented by
(a) (b) (c) (d)
Cp Cp Cp Cp

TC T TC T TC T TC T
Ans: (a)
  
 
Solution: CV  e  2kT 

H.No. 40-D, Ground Floor, Jia Sarai, Near IIT, Hauz Khas, New Delhi-110016
Phone: 011-26865455/+91-9871145498
Website: www.physicsbyfiziks.com | Email: fiziks.physics@gmail.com
10
fiziks
Institute for NET/JRF, GATE, IIT‐JAM, M.Sc. Entrance, JEST, TIFR and GRE in Physics

Q30. The energy  k for band electrons as a function of the wave vector k in the first Brillouin

  
zone    k   of a one dimensional monoatomic lattice is shown as ( a is lattice
 a a
constant) k

k
  /a O  /a

The variation of the group velocity vg is most appropriately represented by


vg vg

(a) (b)
  /a k   /a O
k
O  /a  /a

vg
vg

  /a k
O  /a   /a k
O  /a

Ans: (b)
Solution: E   E0    cos ka  

1 dE a
Vg   sin ka
 dk 
Q31. For Nickel the number density is 8  10 23 atoms / cm 3 and electronic configuration is

1s 2 2 s 2 2 p 6 3s 2 3 p 6 3d 8 4s 2 . The value of the saturation magnetization of Nickel in its

ferromagnetic state is _____________  10 9 A / m .

(Given the value of Bohr magneton  B  9.21  10 21 Am 2 )


Ans: 4.42
Solution: Component of magnetic dipoles in a solid material are in the direction of external field.

H.No. 40-D, Ground Floor, Jia Sarai, Near IIT, Hauz Khas, New Delhi-110016
Phone: 011-26865455/+91-9871145498
Website: www.physicsbyfiziks.com | Email: fiziks.physics@gmail.com
11
fiziks
Institute for NET/JRF, GATE, IIT‐JAM, M.Sc. Entrance, JEST, TIFR and GRE in Physics

M S  (Magnetic dipole moment)  B N  0.6  9.211021  Am 2  N .

(For N i : magnetic dipole moment  0.6 , Fe :2.22, For Cu :1.2 )

 NA
N  8  1029 / m3 ,  B  9.211021 A / m
AN
M S  0.6  9.211021  8 1029  4.42 109 A / m , An  atomic weight

GATE-2015
Q31. The energy dependence of the density of states for a two dimensional non-relativistic
electron gas is given by, g E   CE n , where C is constant. The value of n
is____________
Ans.: 0
Solution: We know that
g  E   E1/ 2 for 3  D , g  E   E 0 for 2  D , g  E   E 1/ 2 for 1  D

 n  0 for 2  D
Q32. The lattice parameters a, b, c of an orthorhombic crystal are related by a  2b  3c . In
units of a the interplanar separation between the 110  planes is ____________. (Upto
three decimal places)
Ans.: 0.447
1 1 a
Solution: d hkl   d110    0.447  a  2b  3c
2
h k l 2 2
1 1 5
 2 2  0
2
a b c a  a 2
2

 
2
Q33. The dispersion relation for phonons in a one dimensional monoatomic Bravais lattice
with lattice spacing a and consisting of ions of masses M is given by

2c
 k   1  coska  , where  is the frequency of oscillation, k is the wavevector
M
and C is the spring constant. For the long wavelength modes   a  , the ratio of the
phase velocity to the group velocity is_________
Ans.: 1

2C
Solution:   k   1  cos  ka  
M 
H.No. 40-D, Ground Floor, Jia Sarai, Near IIT, Hauz Khas, New Delhi-110016
Phone: 011-26865455/+91-9871145498
Website: www.physicsbyfiziks.com | Email: fiziks.physics@gmail.com
12
fiziks
Institute for NET/JRF, GATE, IIT‐JAM, M.Sc. Entrance, JEST, TIFR and GRE in Physics

For long wavelength modes   a 

 ka  2C   ka  
2 2
C
 cos  ka   1   k   1  1   a k
2 M  2  M

 C d C v
Phase velocity vP  a and Group velocity vg  a  P 1
k M dk M vg

Q34. In a Hall effect experiment, the hall voltage for an intrinsic semiconductor is negative.
This is because (symbols carry usual meaning)
(a) n  p (b) n  p (c) n  h (d) mh*  mn*
Ans.: (c)
Solution: The Hall voltage is V H  R H JB
where J : current density, B : magnetic field and R H : Hall constant

1 p  p  n n   p  n   n  p B
2 2 2 2 2

RH 
e  n  p  2   p  n  2  2  2 B 2
n p n p

1  p  n
For intrinsic semiconductor n  p  ni  RH 
eni  p   n

In Intrinsic semiconductor  n   p , therefore Hall voltage is negative.

Q35. Given that the Fermi energy of gold is 5.54 eV , the number density of electrons is

__________ 1028 m 3 (upto one decimal place)


(Mass of electron  9.11  10 31 kg ; h  6.626  10 34 j  s; 1 eV  1.6  10 19 j )
Ans.: 5.9
Solution: Relation between electron density (n) and Fermi energy E F  is

3 2 n2 / 3  n  31 2  2m3


3/ 2
2
EF   EF3/ 2
2m
 2  9.110 kg 
31 3/ 2
1
  5.54 1.6 1019 J 
3/ 2
n 
3   3.14  1.0546 10 J -s 
2 34 3

1 2.45  1045  8.35 1028 3


n  102
m  0.59  10 29 m 3  n  5.9 1028 m 3
29.61 1.17 10

H.No. 40-D, Ground Floor, Jia Sarai, Near IIT, Hauz Khas, New Delhi-110016
Phone: 011-26865455/+91-9871145498
Website: www.physicsbyfiziks.com | Email: fiziks.physics@gmail.com
13
fiziks
Institute for NET/JRF, GATE, IIT‐JAM, M.Sc. Entrance, JEST, TIFR and GRE in Physics

Q36. Which one of the following represents the electron occupancy for a superconductor in its
normal and superconducting states?

Normal Superconducting Normal Superconducting


state state state state
(a) (b)
f E  f E  f E  f E 

E E E E

Normal Superconducting Normal Superconducting


(c) state state (d) state state

f E  f E  f E  f E 

Ans. : (d) E E E E
Solution: In normal slide, some states below Fermi levels are empty and equal number of states
above Fermi levels are filled. If material converts into a superconductor, electrons above
the Fermi Level makes cooper pair and they fall back below level Fermi level as same
energy released during cooper pair formation. Therefore, correct option is (d).
GATE-2016
Q37. Consider a metal which obeys the Sommerfield model exactly. If E F is the Fermi energy
of the metal at T  0 K and R H is its Hall coefficient, which of the following statements
is correct?
3 2

(a) R H  E 2
F (b) R H  E 3
F

3

(c) RH  E F2 (d) R H is independent of E F .


Ans.: (c)

2
3/ 2
 EF 
3/ 2
1
   2m 
2/3
Solution: RH  , where EF  3 2 n n 2   R H  EF3 / 2
ne 2m   3 2

H.No. 40-D, Ground Floor, Jia Sarai, Near IIT, Hauz Khas, New Delhi-110016
Phone: 011-26865455/+91-9871145498
Website: www.physicsbyfiziks.com | Email: fiziks.physics@gmail.com
14
fiziks
Institute for NET/JRF, GATE, IIT‐JAM, M.Sc. Entrance, JEST, TIFR and GRE in Physics

Q38. A one-dimensional linear chain of atoms contains two types of atoms of masses m1 and
m2 (where m2  m1 ), arranged alternately. The distance between successive atoms is the
same. Assume that the harmonic approximation is valid. At the first Brillouin zone
boundary, which of the following statements is correct?
(a) The atoms of mass m2 are at rest in the optical mode, while they vibrate in the
acoustical mode.
(b The atoms of mass m1 are at rest in the optical mode, while they vibrate in the
acoustical mode.
(c) Both types of atoms vibrate with equal amplitudes in the optical as well as in the
acoustical modes.
(d) Both types of atoms vibrate, but with unequal, non-zero amplitudes in the optical as
well as in the acoustical modes.
Ans.: (a)
Solution: In optical mode, at Brillouin zone boundary atom of heavier mass  m2  is at rest,

whereas in Acoustic mode, atoms of lighter mass  m1  is at rest.

 1 1 
2   
optical mode  m1 m2 

2  2  / m1
Acoustic 1  2  / m2
mode
 
2a k 2a
Q39. A solid material is found to have a temperature independent magnetic susceptibility,
  C . Which of the following statements is correct?
(a) If C is positive, the material is a diamagnet.
(b) If C is positive, the material is a ferromagnet.
(c) If C is negative, the material could be a type I superconductor.
(d) If C is positive, the material could be a type I superconductor.
Ans.: (b)

H.No. 40-D, Ground Floor, Jia Sarai, Near IIT, Hauz Khas, New Delhi-110016
Phone: 011-26865455/+91-9871145498
Website: www.physicsbyfiziks.com | Email: fiziks.physics@gmail.com
15
fiziks
Institute for NET/JRF, GATE, IIT‐JAM, M.Sc. Entrance, JEST, TIFR and GRE in Physics


Solution: Susceptibility is defined as    1 , where  and 0 are permeability of medium
0
and vacuum respectively.
(i) For Diamagnet;   0 , thus   0 i.e.  is negative

(ii) For Ferromagnet,   0 , thus   0 i.e.  is positive

(iii) For superconductor,   0 , thus   1


Thus best answer is (b)
Q40. Atoms, which can be assumed to be hard spheres of radius R , are arranged in an fcc
lattice with lattice constant a , such that each atom touches its nearest neighbours. Take
the center of one of the atoms as the origin. Another atom of radius r (assumed to be
 a 
hard sphere) is to be accommodated at a position  0, ,0  without distorting the lattice.
 2 
r
The maximum value of is ________. (Give your answer upto two decimal places)
R
Ans.: 0.41
 a 
Solution: The new atom location is  0, , 0  i.e. it is on the middle of y - axis.
 2  a
z
R

2a

Position of new r
x
 a   a 
atom  0, , 0  a new atom at  0, ,0 
figure (i)  2   2 
2
figure (ii)

If new atom of radius r fit without distorting the original lattice, then from figure (ii) we
get
a
 Rr (i)
2
4
whereas for FCC 2a  4 R  a  R  2 2R (ii)
2
H.No. 40-D, Ground Floor, Jia Sarai, Near IIT, Hauz Khas, New Delhi-110016
Phone: 011-26865455/+91-9871145498
Website: www.physicsbyfiziks.com | Email: fiziks.physics@gmail.com
16
fiziks
Institute for NET/JRF, GATE, IIT‐JAM, M.Sc. Entrance, JEST, TIFR and GRE in Physics

Thus from (i) and (ii)


2 2
2
R  Rr   
2 1 R  r 
r
R
 2  1  1.414  1  0.414

Q41. The energy vs. wave vector E  k  relationship near the bottom of a band for a solid can

be approximated as E  Aka   Bka  , where the lattice constant a  2.1 A . The


2 4 0

values of A and B are 6.3  10 19 J and 3.2 10 20 J , respectively. At the bottom of
the conduction band, the ratio of the effective mass of the electron to the mass of free
electron is _______. (Give your answer upto two decimal places)
(Take   1.05  10 34 J  s , mass of free electron  9.1 10 31 kg )
Ans.: 0.22

Solution: E  A  ka   B  ka 
2 4

E 2 E
  2 Aa 2 k  4 Ba 4 k 3 and  2 Aa 2  12 Ba 4 k 2
k k 2

At the bottom of the band k  0


 
2
2 2 1.05  1034 J  s
Thus effective mass m  2
*  
 
2
 E / k 2 2 Aa 2 2  6.3 1019 J  2.11010 m
1.1025 1068
 39
 0.01984 1029  19.84  1032 kg
55.57 10

m* 19.84  1032 kg
  31
 2.18  101  0.218  0.22
m 9.1  10 kg
Q42. The Fermi energies of two metals X and Y are 5 eV and 7 eV and their Debye
temperatures are 170 K and 340 K , respectively. The molar specific heats of these
metals at constant volume at low temperatures can be written as
CV  X   X T  AX T 3 and CV Y   Y T  AY T 3 where  and A are constants. Assuming
that the thermal effective mass of the electrons in the two metals are same, which of the
following is correct?
 X 7 AX  X 7 AX 1
(a)  , 8 (b)  , 
 Y 5 AY  Y 5 AY 8
 X 5 AX 1  X 5 AX
(c)  ,  (d)  , 8
 Y 7 AY 8  Y 7 AY

H.No. 40-D, Ground Floor, Jia Sarai, Near IIT, Hauz Khas, New Delhi-110016
Phone: 011-26865455/+91-9871145498
Website: www.physicsbyfiziks.com | Email: fiziks.physics@gmail.com
17
fiziks
Institute for NET/JRF, GATE, IIT‐JAM, M.Sc. Entrance, JEST, TIFR and GRE in Physics

Ans.: (a)
Solution: Heat capacity is defined as CV   T  AT 3
3 2 1 12 4 1 1
where   Nk B  and A  Nk B  3  234 Nk B  3
2 EF 5 D D
3 2 1
Nk B 
X 2 EFX EF 7eV 7
Thus,   Y  
Y 3 2 1
Nk B  E FX
5eV 5
2 EFY
1
234 Nk B 3
AX  D3 X   DY   340 3
  2  8
3
and    
AY 234 Nk 1  D   170 
B 3  X 
 DY

X 7 A
Thus,  and X  8
Y 5 AY
GATE-2017
Q43. The atomic mass and mass density of Sodium are 23 and 0.968 g cm 3 , respectively. The

number density of valence electrons is……………… 1022 cm 3 . (Up to two decimal

places) (Avogadro number, N A  6.022 1023 )

Ans. : 2.54
neff  M neff  NA
Solution:  n 
NA  a 3
a 3
M

where   0.968gcm 3 , N A  6.022 1023 , M  23g

0.968  6.022  1023


n  2.54 1022 cm 3
23
 2 x 
Q44. Consider a one dimensional lattice with a weak periodic potential U  x   U 0 cos  .
 a 
 
The gap at the edge of the Brillouin zone  k   is:
 a
U0 U
(a) U 0 (b) (c) 2U 0 (d) 0
2 4
Ans. : (c)

H.No. 40-D, Ground Floor, Jia Sarai, Near IIT, Hauz Khas, New Delhi-110016
Phone: 011-26865455/+91-9871145498
Website: www.physicsbyfiziks.com | Email: fiziks.physics@gmail.com
18
fiziks
Institute for NET/JRF, GATE, IIT‐JAM, M.Sc. Entrance, JEST, TIFR and GRE in Physics

 2 
Solution: U  U 0 cos  x
 a 
 2 a 
Energy at the edge of Brillouin Zone is U t  U 0 cos  . 
 a 
Energy at the k  0 is U b  U 0  Band gap U  U t  U b  2U 0

Q45. Consider a 2 - dimensional electron gas with a density of 1019 m2 . The Fermi energy of
the system is………………… eV (up to two decimal places).
( me  9.311031 kg , h  6.626 1034 Js, e  1.602  1019 C )

Ans. : 2.34

 
2
 2  1.055 1034 J  s
Solution: EF     2 n    2  3.142 1019
 2m  2  9.311031

 0.3756 1018 J  0.2345 10 eV  2.34 eV

Q46.
  a
The real space primitive lattice vectors are a1  axˆ and a2  xˆ  3 yˆ . The reciprocal
2
 
 
space unit vectors b1 and b2 for this lattice are, respectively

2  yˆ  4 2  yˆ  4
(a)
a  xˆ   and yˆ (b)
a  xˆ   and yˆ
 3 a 3  3 a 3

2 4  xˆ  2 4  xˆ 
(c) xˆ and   yˆ  (d) xˆ and   yˆ 
a 3 a  3  a 3 a  3 
Ans. (a)
  a
  
Solution: a1  axˆ , a2  xˆ  3 yˆ . Assume, a3  zˆ
2

  a2
 
3a 2
2
   a  a xˆ
Now, a1   a2  a3   axˆ.  xˆ  3 yˆ  zˆ     yˆ  3xˆ   0  3  
2  2 2 2

  
a a  3 xˆ  yˆ  a2 2  yˆ 
b1  2  2 3  2   xˆ  
a1   a2  a3  3 2 a  3
a
2
  
a3  a1 4
Similarly, b2  2     yˆ . Thus correct option is (a)
a1   a2  a3  3a

H.No. 40-D, Ground Floor, Jia Sarai, Near IIT, Hauz Khas, New Delhi-110016
Phone: 011-26865455/+91-9871145498
Website: www.physicsbyfiziks.com | Email: fiziks.physics@gmail.com
19
fiziks
Institute for NET/JRF, GATE, IIT‐JAM, M.Sc. Entrance, JEST, TIFR and GRE in Physics

GATE – 2018
Q47. For the given unit cells of a two dimensional square lattice, which option lists all the
primitive cells?

1 5

4
3
2

(a) (1) and (2) (b) (1), (2) and (3)


(c) (1), (2), (3) and (4) (d) (1), (2), (3), (4) and (5)
Ans. : (c)
Solution: For primitive cell, N eff  1

In cell (1), (2), (3) and (4) N eff  1 , these are primitive cell

Whereas in cell (5), N eff  2 , this is non-primitive cell.

Q48. At low temperatures ( T ), the specific heat of common metals is described by (with 
and  as constants)

(a)  T   T 3 (b)  T 3 (c) exp   / T  (d)  T   T 5

Ans. : (a)
Solution: C  Ce  C pn   T   T 3

Q49. The high temperature magnetic susceptibility of solids having ions with magnetic
1
moments can be described by   with T as absolute temperature and  as
T 
constant. The three behaviours i.e., paramagnetic, ferromagnetic and anti-ferromagnetic
are described, respectively, by
(a)   0,   0,   0 (b)   0,   0,   0
(c)   0,   0,   0 (d)   0,   0,   0
Ans. : (c)

H.No. 40-D, Ground Floor, Jia Sarai, Near IIT, Hauz Khas, New Delhi-110016
Phone: 011-26865455/+91-9871145498
Website: www.physicsbyfiziks.com | Email: fiziks.physics@gmail.com
20
fiziks
Institute for NET/JRF, GATE, IIT‐JAM, M.Sc. Entrance, JEST, TIFR and GRE in Physics

C
Solution: Paramagnetism:  
T
C
Ferromagnetism:  
T  TC
C
Anti-ferromagnetism:  
T  TC
Q50. The energy dispersion for electrons in one dimensional lattice with lattice parameter a is
1
given by E  k   E0  W cos ka , where W and E0 are constants. The effective mass of
2
the electron near the bottom of the band is
2 2 2 2 2
(a) (b) (c) (d)
Wa 2 Wa 2 2Wa 2 4Wa 2
Ans. : (a)
1
Solution: E  k   E0  W cos  ka 
2
dE aW d 2 E a 2W
 sin  ka   2  cos  ka 
dk 2 dk 2
2 2 2 2
 m  2  2
*
 [At bottom of the band, k  0 ]
d E aW Wa 2
cos  ka 
dk 2 2
Q51. Amongst electrical resistivity    , thermal conductivity   , specific heat  C  , Young’s

modulus Y  and magnetic susceptibility    , which quantities show a sharp change at

the superconducting transition temperature?


(a)  ,  , C , Y (b)  , C ,  (c)  ,  , C ,  (d)  , Y , 
Ans. : (b)

H.No. 40-D, Ground Floor, Jia Sarai, Near IIT, Hauz Khas, New Delhi-110016
Phone: 011-26865455/+91-9871145498
Website: www.physicsbyfiziks.com | Email: fiziks.physics@gmail.com
21
fiziks
Institute for NET/JRF, GATE, IIT‐JAM, M.Sc. Entrance, JEST, TIFR and GRE in Physics

GATE-2019
Q52. The relative magnetic permeability of a type-I super conductor is
1
(a) 0 (b) 1 (c) 2 (d)
4
Ans.: (a)
Solution: B  0  H  M   0  H   H   0 1    H   H


   0 1  x   r   1 
0
For type-I superconductor:   1
 r  1  1  0
Q53. In order to estimate the specific heat of phonons, the appropriate method to apply would
be
(a) Einstein model for acoustic phonons and Debye model for optical phonons
(b) Einstein model for optical phonons and Debye model for acoustic phonons
(c) Einstein model for both optical and acoustic phonons
(d) Debye model for both optical and acoustic phonons
Ans.: (b)
Solution: At low temperature, the optical branch phonons have energies higher than k B T and
therefore, optical branch waves are not excited. And Debye model is not suitable for
optical branch instead it is suitable for acoustical branch. Whereas Einstein model is
useful for high temperature and therefore can be applied to optical branch.
Q54. Consider a three-dimensional crystal of N inert gas atoms. The total energy is given by
   12   
6

U  R   2 N   p    q    , where p  12.13, q  14.45 and R is the nearest


  R   R  

neighbour distance between two atoms. The two constants,  and R , have the
dimensions of energy and length, respectively. The equilibrium separation between two
nearest neighbour atoms in units of  (rounded off to two decimal places)
is____________
Ans.: 1.09

H.No. 40-D, Ground Floor, Jia Sarai, Near IIT, Hauz Khas, New Delhi-110016
Phone: 011-26865455/+91-9871145498
Website: www.physicsbyfiziks.com | Email: fiziks.physics@gmail.com
22
fiziks
Institute for NET/JRF, GATE, IIT‐JAM, M.Sc. Entrance, JEST, TIFR and GRE in Physics

   12   
6

Solution: U  R   2 N   p    q   
  R   R  

dU  11
           
5

 0  2 N  12 p     2   6 q     2   0
dR  R  R   R   R  

 12 6
 12 p  6q 0
R13 R7
 12 6 12 p 6
 12 p 13
 6q 7
 R6  
R R 6q
1/ 6
 2p 
 R   given p  12.13 , q  14.45
 q 
1/ 6
 2 12.13 
   1.679    1.09 
1/ 6
R  
 14.45 
R
Thus  1.09

Q55. The energy-wavevector  E  k  dispersion relation for a particle in two dimensions is

E  Ck , where C is a constant. If its density of states D  E  is proportional to E p then

the value of p is____________


Ans.: 1
d 
 1
Solution: For E  k   k . The density of states in d - dimension is D  E   E
s s 

Given, E  Ck  s  1, d  2
2 
 1
Thus D  E   E 1 

 E1
Q56. A conventional type-I superconductor has a critical temperature of 4.7 K at zero
magnetic field and a critical magnetic field of 0.3 Tesla at 0 K . The critical field in Tesla
at 2K (rounded off to three decimal places) is__________
Ans.: 0.246

H.No. 40-D, Ground Floor, Jia Sarai, Near IIT, Hauz Khas, New Delhi-110016
Phone: 011-26865455/+91-9871145498
Website: www.physicsbyfiziks.com | Email: fiziks.physics@gmail.com
23
fiziks
Institute for NET/JRF, GATE, IIT‐JAM, M.Sc. Entrance, JEST, TIFR and GRE in Physics

  T 2    2 2 
Solution: H c T   H 0 1      0.3 1      0.3 1   0.426  
2

  Tc     4.7  

 0.3 1  0.181  0.3  0.819  0.246 Atm

Q57. A particle of mass m moves in a lattice along the x - axis in a periodic potential
V  x   V  x  d  with periodicity d . The corresponding Brillouin zone extends from

 k0 to k0 with these two k - points being equivalent. If a weak force F in the x -


direction is applied to the particle, it starts a periodic motion with the time period T .
dpcrystal
Using the equation of motion F  for a particle moving in a band, where pcrystal is
dt
the crystal momentum of the particle, the period T is found to be ( h is Planck constant)

2md 2md 2h h
(a) (b) 2 (c) (d)
F F Fd Fd
Ans. : (d)
d
Solution: E  E  Fdx  F  x 0  Fd
d

h h
Using Heisenberg uncertainty E  t  h , T  t   . Thus correct option is (d)
E Fd
Q58. In a certain two-dimensional lattice, the energy dispersion of the electrons is
  
  1
 k  2t cos k x a  2 cos k x a cos
2 2
3
kya
 

where k   k x , k y  denotes the wave vector, a is the lattice constant and t is a constant

in units of eV . In this lattice the effective mass tensor mij of electrons calculated at the

2   0 
center of the Brillouin zone has the form mij    . The value of  (rounded off
ta 2  0  
to two decimal places) is ____________
Ans.: 0.333
Solution: Effective mass tensor matrix 4

H.No. 40-D, Ground Floor, Jia Sarai, Near IIT, Hauz Khas, New Delhi-110016
Phone: 011-26865455/+91-9871145498
Website: www.physicsbyfiziks.com | Email: fiziks.physics@gmail.com
24
fiziks
Institute for NET/JRF, GATE, IIT‐JAM, M.Sc. Entrance, JEST, TIFR and GRE in Physics

 1 1   1 
m 0 
mxy   mxx
mij    
xx

 1 1   1 
   0
 myx m yy   m yy 

2 2
When mxx  and m yy 
 2 E / k x2  2 E / k y2

E  1   3 
Now  2t  a sin k x a  a sin  k x a  cos  k y a  
k x  2   2  

2 E  2 a2 1   3 
2
 2t  a cos  x 
k a  cos  k x a  cos  k y a  
dk x  2 2   2  
At the Brillouin zone centre i.e. at k x  k y  0

2 E  1
  2ta 2 1    3ta 2
k x
2
 2

E  1   3 
Similarly,  2t  3a cos  k x a  sin  k y a  
k y  2   2  

2 E  3a 2 1   3 
 2t  cos k
 x a cos  2 y  
 k a
k y2  2 2    
At the Brillouin zone centre i.e. at k x  k y  0

2 E
 3ta 2
 2y

2 2 2 2
Thus mxx   and m yy  
 2 E / k x2 3ta 2  2 E / k y2 3ta 2

 2  1 
 3ta 2 0  3 0
2
mij     2  
  2  ta 0 1
 0 3ta 2   3 
1
Thus    0.333 .
3

H.No. 40-D, Ground Floor, Jia Sarai, Near IIT, Hauz Khas, New Delhi-110016
Phone: 011-26865455/+91-9871145498
Website: www.physicsbyfiziks.com | Email: fiziks.physics@gmail.com
25
fiziks
Institute for NET/JRF, GATE, IIT‐JAM, M.Sc. Entrance, JEST, TIFR and GRE in Physics

NUCLEAR AND PARTICLE PHYSICS

GATE-2010
Q1. The basic process underlying the neutron β-decay is
(a) d  u  e    e (b) d  u  e 
(c) s  u  e    e (d) u  d  e    e
Ans: (a)
Q2. In the nuclear shell model the spin parity of 157 N is given by

1 1 3 3
(a) (b) (c) (d)
2 2 2 2
Ans: (a)
Solution: Z  7 ; s1 / 2   p3 / 2 4  p1 / 2 1
2
and N  8

1 1
l  1, J   parity   1  1,
1
spin - parity   
2 2
Q3. Match the reactions on the left with the associated interactions on the right.
(1) π+ → μ+ +  (i) Strong

(2) π0 → γ + γ (ii) Electromagnetic


(3) π0 + n → π- + p (iii) Weak
(a) (1, iii), (2, ii), (3, i) (b) (1, i), (2, ii), (3, iii)
(c) (1, ii), (2, i), (3, iii) (d) (1, iii), (2, i), (3, ii)
Ans: (a)
Q4. The ground state wavefunction of deuteron is in a superposition of s and d states. Which
of the following is NOT true as a consequence?
(a) It has a non-zero quadruple moment
(b) The neutron-proton potential is non-central
(c) The orbital wavefunction is not spherically symmetric
(d) The Hamiltonian does not conserve the total angular momentum
Ans: (d)

H.No. 40-D, Ground Floor, Jia Sarai, Near IIT, Hauz Khas, New Delhi-110016
Phone: 011-26865455/+91-9871145498
Website: www.physicsbyfiziks.com | Email: fiziks.physics@gmail.com
1
fiziks
Institute for NET/JRF, GATE, IIT‐JAM, M.Sc. Entrance, JEST, TIFR and GRE in Physics

228
Q5. The first three energy levels of Th90 are shown below
4 187 keV
2 57.5keV
0 0keV

The expected spin-parity and energy of the next level are given by
(a) (6+; 400 keV) (b) (6+; 300 keV) (c) (2+; 400 keV) (d) (4+; 300 keV)
Ans: (a)
E 2 J 2  J 2  1 E 66  1
Solution:   6   E 6  393keV
E1 J 1  J 1  1 E 4 44  1

GATE-2011

Q6. The semi-empirical mass formula for the binding energy of nucleus contains a surface
correction term. This term depends on the mass number A of the nucleus as
(a) A-1/3 (b) A1/3 (c) A2/3 (d) A
Ans: (c)
Q7. According to the single particles nuclear shell model, the spin-parity of the ground state
of 178 O is
   
1 3 3 5
(a) (b) (c) (d)
2 2 2 2
Ans: (d)
Solution: Z  8 and N  9; s1 / 2   p3 / 2 4  p1 / 2 2 d 5 / 2 1
2


5 5
l  2, J   parity   1  1, spin - parity   
2

2 2
Q8. In the β-decay of neutron n→ p + e- + e , the anti-neutrino e , escapes detection. Its
existence is inferred from the measurement of
(a) energy distribution of electrons (b) angular distribution of electrons
(c) helicity distribution of electrons (d) forward-backward asymmetry of electrons
Ans: (a)
Q9. The isospin and the strangeness of   baryon are
(a) 1, -3 (b) 0, -3 (c) 1, 3 (d) 0, 3
Ans: (b)

H.No. 40-D, Ground Floor, Jia Sarai, Near IIT, Hauz Khas, New Delhi-110016
Phone: 011-26865455/+91-9871145498
Website: www.physicsbyfiziks.com | Email: fiziks.physics@gmail.com
2
fiziks
Institute for NET/JRF, GATE, IIT‐JAM, M.Sc. Entrance, JEST, TIFR and GRE in Physics

GATE-2012

Q10. Deuteron has only one bound state with spin parity 1+, isospin 0 and electric quadrupole
moment 0.286 efm2. These data suggest that the nuclear forces are having
(a) only spin and isospin dependence
(b) no spin dependence and no tensor components
(c) spin dependence but no tensor components
(d) spin dependence along with tensor components
Ans: (d)
Q11. The quark content of   , K  ,   and p is indicated:

   uus ; K   su ;    u d ; p  uud .

In the process,    p  K     , considering strong interactions only, which of the


following statements is true?
(a) The process, is allowed because ∆S = 0
(b) The process is allowed because ∆I3 =0
(c) The process is not allowed because ∆S ≠ 0 and ∆I3 ≠ 0
(d) The process is not allowed because the baryon number is violated
Ans: (c)
Solution:    p  k    

S: 0 0 1 1 (not conserved)
1 1
I3 : 1   1 (not conserved)
2 2
For strong interaction S and I3 must conserve. Therefore this process is not allowed under
strong interaction
Q12. Which one of the following sets corresponds to fundamental particles?
(a) proton, electron and neutron
(b) proton, electron and photon
(c) electron, photon and neutrino
(d) quark, electron and meson
Ans: (a)

H.No. 40-D, Ground Floor, Jia Sarai, Near IIT, Hauz Khas, New Delhi-110016
Phone: 011-26865455/+91-9871145498
Website: www.physicsbyfiziks.com | Email: fiziks.physics@gmail.com
3
fiziks
Institute for NET/JRF, GATE, IIT‐JAM, M.Sc. Entrance, JEST, TIFR and GRE in Physics

Q13. In case of a Geiger-Muller (GM) counter, which one of the following statement is
CORRECT?
(a) Multiplication factor of the detector is of the order of 1010
(b) Type of the particles detected can be identified
(c) Energy of the particles detected can be distinguished
(d) Operating voltage of the detector is few tens of Volts
Ans: (c)
Q14. Choose the CORRECT statement from the following
(a) Neutron interacts through electromagnetic interaction
(b) Electron does not interact through weak interaction
(c) Neutrino interacts through weak and electromagnetic interaction
(d) Quark interacts through strong interaction but not through weak interaction
Ans: (d)

GATE-2013
Q15. The decay process n  p   e   v e violates
(a) Baryon number (b) lepton number (c) isospin (d) strangeness
Ans: (c)
Q16. The isospin I  and baryon number B  of the up quark is
(a) I  1, B  1 (b) I  1, B  1 / 3
(c) I  1 / 2, B  1 (d) I  1 / 2, B  1 / 3
Ans: (d)

Q17. In the  decay process, the transition 2   3  , is


(a) allowed both by Fermi and Gamow-Teller selection rule
(b) allowed by Fermi and but not by Gamow-Teller selection rule
(c) not allowed by Fermi but allowed by Gamow-Teller selection rule
(d) not allowed both by Fermi and Gamow-Teller selection rule
Ans: (c)
Solution: According to Fermi Selection Rule:

H.No. 40-D, Ground Floor, Jia Sarai, Near IIT, Hauz Khas, New Delhi-110016
Phone: 011-26865455/+91-9871145498
Website: www.physicsbyfiziks.com | Email: fiziks.physics@gmail.com
4
fiziks
Institute for NET/JRF, GATE, IIT‐JAM, M.Sc. Entrance, JEST, TIFR and GRE in Physics

I  0, Parity  No Change
According to Gammow-Teller Selection Rule:
I  0,1, Parity  No Change

In the  decay process, the transition 2   3  ,


I  1, Parity  No Change .

GATE-2014

Q18. Which one of the following is a fermions’?


(a)  -particle (b) 4 Be7 nucleus
(c) Hydrogen atom (d) deuteron

Ans: (b)
Solution: If a nucleus contains odd number of nucleons, it is fermions. If a nucleus contains even
number of nucleons, it is a boson.
Q19. Which one of the following three-quark states  qqq  denoted by X CANNOT be a

possible baryon? The corresponding electric charge is indicated in the superscript.


(a) X   (b) X  (c) X  (d) X  
Ans: (d)
Solution: X  qqq
2 2 2 6
X   uuu      2  two unit positive charge 
3 3 3 3
2 2 1 4 1
X   uud       1 single unit positive charge 
3 3 3 3 3
1 1 1
X   ddd       1 single unit negative charge 
3 3 3
X   Not possible with qqq  . So the correct option is (d)

Q20. Consider the process           . The minimum kinetic energy of the muons
  in the centre of mass frame required to produce the pion   pairs at rest is
______ MeV .
Ans: 81.7
H.No. 40-D, Ground Floor, Jia Sarai, Near IIT, Hauz Khas, New Delhi-110016
Phone: 011-26865455/+91-9871145498
Website: www.physicsbyfiziks.com | Email: fiziks.physics@gmail.com
5
fiziks
Institute for NET/JRF, GATE, IIT‐JAM, M.Sc. Entrance, JEST, TIFR and GRE in Physics

Solution: Use conservation of energy and momentum in relativistic form.


m  105 MeV / c 2 and m  140 MeV / c 2

m   c
2 2

   m  c 2  m   m 2
 280 
2
MeV   210  MeV
2

E   E   163.3 MeV
2m 2  105
163.3
For pair it will be MeV  81.7 MeV
2
Q21. A nucleus X undergoes a first forbidden  -decay to nucleus Y . If the angular
7
momentum I  and parity P  , denoted by I P as for X , which of the following is a
2
possible I P value for Y ?
1 1 3 3
(a) (b) (c) (d)
2 2 2 2
Ans: (c)
Solution: For first forbidden  -decay; I  0,1 or 2 and Parity does change.

GATE-2015
Q22. The decay    e    is forbidden, because it violates
(a) momentum and lepton number conservations
(b) baryon and lepton number conservations
(c) angular momentum conservation
(d) lepton number conservation
Ans.: (d)
Solution:    e    . In this decay lepton number is not conserved.
Q23. A beam of X - ray of intensity I 0 is incident normally on a metal sheet of thickness

2 mm . The intensity of the transmitted beam is 0.025 I 0 . The linear absorption

 
coefficient of the metal sheet in m 1 is _______________ (upto one decimal place)
Ans.: 1844.4
x 1  I0  1  I0  1
Solution: I  I 0 e  ln    3
ln   3
ln  40 
x  I  2  10  0.025 I 0  2  10
2.303
3 
 log10 40  1.151 103  2  0.3010  1  1844.4 m 1
2  10
H.No. 40-D, Ground Floor, Jia Sarai, Near IIT, Hauz Khas, New Delhi-110016
Phone: 011-26865455/+91-9871145498
Website: www.physicsbyfiziks.com | Email: fiziks.physics@gmail.com
6
fiziks
Institute for NET/JRF, GATE, IIT‐JAM, M.Sc. Entrance, JEST, TIFR and GRE in Physics

Q24. The mean kinetic energy of a nucleon in a nucleus of atomic weight A varies as An ,
where n is____________(upto two decimal places)
Ans.: -0.67
R 2  d 2 1 d  2 2
   2  4 r dr
2 R
0 2m  dr r dr   4   2  2  dr  4  4 R
Solution: T   2m 0
 2 m
R R
4 R 3 / 3
 4 r 2 dr   2
4 r dr
0 0

2
1 1 1  2
 T  2
 2
 2
 A 3
 n    0.667  0.67
R  1
 3
R A
 0 
3 A3
 
152 152
Q25. The atomic masses of 63 Eu, 62 Sm, 11 H and neutron are 151.921749, 151.919756,

1.007825 and 1.008665 in atomic mass units (amu), respectively. Using the above
3
information, the Q - value of the reaction 152
63 62 Sm  p is ___________  10
Eu  n 152
amu (upto three decimal places)
Ans.: 2.833
Solution: Q  152.930414  152.927581  2.833  103 a.m.u.

1  
Q26. In the nuclear shell model, the potential is modeled as V r   m 2 r 2  L  S ,   0 .
2
The correct spin-parity and isospin assignments for the ground state of 136C is
1  1 1  1 3 1 3  1
(a) ; (b) ; (c) ; (d) ;
2 2 2 2 2 2 2 2
Ans.: (a)
2 4 1
      1
Solution: C6 , N  7, Z  6 , for N  7 ; 1S 1 
13
1P3   P1   j  and l  1
 2  2  2 2

1
Thus spin- parity is   .
2

H.No. 40-D, Ground Floor, Jia Sarai, Near IIT, Hauz Khas, New Delhi-110016
Phone: 011-26865455/+91-9871145498
Website: www.physicsbyfiziks.com | Email: fiziks.physics@gmail.com
7
fiziks
Institute for NET/JRF, GATE, IIT‐JAM, M.Sc. Entrance, JEST, TIFR and GRE in Physics

GATE-2016
Q27. In the SU 3 quark model, the triplet of mesons   ,  0 ,    has
(a) Isospin  0 , Strangeness  0 (b) Isospin  1 , Strangeness  0
1 1
(c) Isospin  , Strangeness  1 (d) Isospin  , Strangeness  1
2 2
Ans.: (b)
Solution:   ,  0 ,   are not strange particle thus strangness  0
Since meson group contain 3 particles, thus I  1

25 Mn  e  24 Cr  X . The particle X is
Consider the reaction 54 54
Q28.

(a)  (b)  e (c) n (d)  0


Ans.: (b)
Q29. Which of the following statements is NOT correct?
(a) A deuteron can be disintegrated by irradiating it with gamma rays of energy 4 MeV .
(b) A deuteron has no excited states.
(c) A deuteron has no electric quadrupole moment.
(d) The 1 S 0 state of deuteron cannot be formed.
Ans.: (c)
Q30. According to the nuclear shell model, the respective ground state spin-parity values of
15 17
8 O and 8 O nuclei are

1 1 1 5 3 5  3  1
(a) , (b) , (c) , (d) ,
2 2 2 2 2 2 2 2
Ans.: (b)
N  7 :  s1/ 2   p3/ 2   p1/ 2 
2 4 1
8 O ; Z  8 and N  7 ;
Solution: 15

1 1
 j  and l  1 . Thus spin and parity   
2 2

N  9 :  s1/ 2   p3/ 2   p1/ 2   d5/ 2 


2 4 2 1
17
8 O ; Z  8 and N  9 ;

5 5
 j  and l  2 . Thus spin and parity   
2 2

H.No. 40-D, Ground Floor, Jia Sarai, Near IIT, Hauz Khas, New Delhi-110016
Phone: 011-26865455/+91-9871145498
Website: www.physicsbyfiziks.com | Email: fiziks.physics@gmail.com
8
fiziks
Institute for NET/JRF, GATE, IIT‐JAM, M.Sc. Entrance, JEST, TIFR and GRE in Physics

GATE-2017
Q31. Which one of the following conservation laws is violated in the decay         
(a) Angular momentum (b) Total Lepton number
(c) Electric charge (d) Tau number
Ans. : (d)
Solution:        
q  1 1  1  1 conserved
L  1 1  1  1 conserved
L  1 0 0 0 Not conserved

1 1 1
spin = 1 conserved
2 2 2 2
Tau number is not conserved
Q32. Electromagnetic interactions are:
(a) C conserving
(b) C non-conserving but CP conserving
(c) CP non-conserving but CPT conserving
(d) CPT non-conserving
Ans. : (a)
Solution: In electromagnetic interaction C is conserved
CPT: Conserved in all interaction
CP: Conserved in EM and Strong interactions
13.6
En   eV 
n2
For n  1 , E1  13.6 eV Ground state

For n   , E  0 Highest state


Thus, correct option is (a)
Q33. In the nuclear reaction 13C6  e  13N 7  X , the particle X is
(a) an electron (b) an anti-electron
(c) a muon (d) a pion
Ans. : (a)
H.No. 40-D, Ground Floor, Jia Sarai, Near IIT, Hauz Khas, New Delhi-110016
Phone: 011-26865455/+91-9871145498
Website: www.physicsbyfiziks.com | Email: fiziks.physics@gmail.com
9
fiziks
Institute for NET/JRF, GATE, IIT‐JAM, M.Sc. Entrance, JEST, TIFR and GRE in Physics

Solution: 13C6  e  13N 7  X  13C6  13N 7  X  e

Le  0 0 1 1

To conserve the Lepton number Le , X should be e 

Q34. J P for the ground state of the 13 C6 nucleus is

3 3 1
(a) 1 (b) (c) (d)
2 2 2
Ans. : (d)

Solution: 13 C6 : Z  6, N  7 , N  7 :  s1/ 2   p3/ 2   p1/ 2 


2 4 1


1 1
 j  and l  1 . Thus spin and parity   
2 2
Q35. The   decays at rest to   and v . Assuming the neutrino to be massless, the

momentum of the neutrino is…………….. MeV / c . (up to two decimal places)


( m  139 MeV / c 2 , m  105 MeV / c 2 )

Ans. : 29.84

Solution: E 
m 
2

 m2 c 2
 pc
2m

So p
m 
2

 m2 c

19321  11025 29.84
  MeV 
2m 2  139 c c

H.No. 40-D, Ground Floor, Jia Sarai, Near IIT, Hauz Khas, New Delhi-110016
Phone: 011-26865455/+91-9871145498
Website: www.physicsbyfiziks.com | Email: fiziks.physics@gmail.com
10
fiziks
Institute for NET/JRF, GATE, IIT‐JAM, M.Sc. Entrance, JEST, TIFR and GRE in Physics

GATE-2018
Q36. The elementary particle  is placed in the baryon decuplet, shown below, at
0



Strangeness
Q

P R

S
3rd component of isospin

(a) P (b) Q (c) R (d) S


Ans. : (c)
 0  
0
  
 
Q
 0
 
P R


Q37. In the decay,    e    e  X , what is X ?
(a)  (b)  e (c)   (d)  

Ans. : (d)
Solution:- u   e   e  u

Lu :  1 0 0 1

Le : 0 1 1 0

Q38. For nucleus 164


Er , a J   2 state is at 90 keV . Assuming 164
Er to be a rigid rotor, the

energy of its 4 state is ___________ keV (up to one decimal place)


Ans. : 300
Solution: EJ  hcBJ  J  1 _________ 4

E2  hc B 2  2  1 and E4  hc B 4  4  1 _________ 2

E4 20 20
Then,   E4   90 keV  300 keV
E2 6 6

H.No. 40-D, Ground Floor, Jia Sarai, Near IIT, Hauz Khas, New Delhi-110016
Phone: 011-26865455/+91-9871145498
Website: www.physicsbyfiziks.com | Email: fiziks.physics@gmail.com
11
fiziks
Institute for NET/JRF, GATE, IIT‐JAM, M.Sc. Entrance, JEST, TIFR and GRE in Physics

Q39. Inside a large nucleus, a nucleon with mass 939 MeVc 2 has Fermi momentum 1.40 fm 1
at absolute zero temperature. Its velocity is Xc , where the value of X is__________ (up
to two decimal places).
( c  197 MeV -fm )
Ans. : 0.29
Solution: Here, fermi – momentum or fermi radius, k F  1.40 fm 1 and c  197 Mev – fm
Now, Fermi velocity –
P k F  c  k F  c 197   1 40  c 275  8c
VF       0.29c
m m mc2 939 939

Q40. An  particle is emitted by a 230


90 Th nucleus. Assuming the potential to be purely
Coulombic beyond the point of separation, the height of the Coulomb barrier is________
MeV (up to two decimal places).
e2
(  1.44 MeV -fm, r0  1.30 fm )
4 0
Ans. : 25.995
Solution: The height of coulomb barrier for  particle from

90 Th 230  88 X 226  2 He 4 (  - particle)

1  2 ze 2 
VC   
4 0  R 
e2
Here, R0  1.3 fm,  1.44 MeV fm
4 0

And R  R0 A1 / 3
Here, we consider pure Coulombic interection
 A1X/ 3  A1 / 3  226   4   6.09  1.58  7.67
1/ 3 1/ 3 1/ 3
ATh

R  R0 ATh
1/ 3
 1.37.67 

 e2  2  90 180  1.44 MeV


Hence, VC    
 4 0  1.37.67  1.3  7.67 fm
VC  25.995 MeV

H.No. 40-D, Ground Floor, Jia Sarai, Near IIT, Hauz Khas, New Delhi-110016
Phone: 011-26865455/+91-9871145498
Website: www.physicsbyfiziks.com | Email: fiziks.physics@gmail.com
12
fiziks
Institute for NET/JRF, GATE, IIT‐JAM, M.Sc. Entrance, JEST, TIFR and GRE in Physics

GATE-2019
Q41. Considering baryon number and lepton number conservation laws, which of the
following process is/are allowed?
(i) p   0  e   ve
(ii) e   ve     v
(a) both (i) and (ii) (b) only (i) (c) only (ii) (d) neither (i) nor (ii)
Ans. : (c)
Solution: (i) P   0  e  e

B : 1 0 0 0 : Not conserved
Therefore, this is not an allowed process
(ii) e  e    
q: 1 0 1 0 : conserved
spin : 1/ 2 1/ 2 1/ 2 1/ 2 : conserved
Le : 1 1 0 0 : conserved

L : 0 0 1 1 : conserved

Since neutrino is involve, therefore parity is violated. This is allowed through weak
interaction
Q42. A massive particle X in free space decays spontaneously into two photons. Which of the
following statements is true for X ?
(a) X is charged
(b) Spin of X must be greater than or equal to 2
(c) X is a boson
(d) X must be a baryon
Ans. : (c)
Solution: X  rr
q: 0 0 0
spin : 0,1, 2 1 1
Thus spin of X can be either 0,1 or 2 . (integer)
Therefore, option (b) is wrong while option (c) is correct.

H.No. 40-D, Ground Floor, Jia Sarai, Near IIT, Hauz Khas, New Delhi-110016
Phone: 011-26865455/+91-9871145498
Website: www.physicsbyfiziks.com | Email: fiziks.physics@gmail.com
13
fiziks
Institute for NET/JRF, GATE, IIT‐JAM, M.Sc. Entrance, JEST, TIFR and GRE in Physics

40
Q43. The nuclear spin and parity of 20 Ca in its ground state is

(a) 0 (b) 0 (c) 1 (d) 1


Ans.: (a)
Solution: 40
20 Ca is an even-even nuclei, therefore I  0, P   ve

 Spin-parity  0
Q44. Low energy collision ( s - wave scattering) of pion (   ) with deuteron ( d ) results in the
production of two proton (    d  p  p ). The relative orbital angular momentum (in
units of  ) of the resulting two-proton system for this reaction is
(a) 0 (b) 1 (c) 2 (d) 3
Ans.: (b)
Solution:   d  p p

Parity:  1   1 (1)l  p p

  1  p p  1
l

Since  p  1   1l  1

Thus, l  1 .
Q45. A radioactive element X has a half-life of 30 hours. It decays via alpha, beta and
gamma emissions with the branching ratio for beta decay being 0.75 . The partial half-life
for beta decay in unit of hours is ____________
Ans.: 40
Solution: Branching ratio is the fraction of particles (here  ) which decays by an individual
decay mode with respect to the total number of particles which decays
 dN 
 
 dt  x T1/ 2  x T1/ 2  x 30
BR    T1/ 2      40 hours
 dt  T1/ 2  BR 0.75
 
 dt  

H.No. 40-D, Ground Floor, Jia Sarai, Near IIT, Hauz Khas, New Delhi-110016
Phone: 011-26865455/+91-9871145498
Website: www.physicsbyfiziks.com | Email: fiziks.physics@gmail.com
14

You might also like